X



トップページ数学
1002コメント338KB
分からない問題はここに書いてね440
レス数が1000を超えています。これ以上書き込みはできません。
0002132人目の素数さん垢版2018/01/20(土) 14:21:38.24ID:2JsoyV0W
x=-1で最小となりx=-3のときy=5、x=2のときy=15である2次関数の式を求めよ
教えて下さい
0003132人目の素数さん垢版2018/01/20(土) 14:22:05.05ID:2JsoyV0W
すれたてお疲れ様でした
0004132人目の素数さん垢版2018/01/20(土) 15:02:45.18ID:BdHhmenA
[前スレ.998]

固有値λが固有方程式のk重根の場合、固有ベクトルは存在しますが(独立に)k個あるとは限りません。
しかし、一般固有空間{x≠o|(A-λI)^k・x = o}はk次元で、k個の基底ベクトルがあります。

・基底の求め方
固有ベクトルの1つ x_1 から
 (A-λI)x_j = x_(j-1)
により定まります。
0006132人目の素数さん垢版2018/01/20(土) 15:34:30.70ID:gB7kB20f
>>4
スレ立て乙。

それはごもっともです。
これは最小多項式が重根を持たない場合ですよ。
前スレ998の例では rank(A-I) = 1 なので固有値1の固有空間の次元2と同じ数だけ固有ベクトルを取れると思います。
0007132人目の素数さん垢版2018/01/20(土) 16:55:49.62ID:XURtOrzK
オズワルド・ヴェブレンと野口聡一はどっちの方が頭が良いですか?
0008132人目の素数さん垢版2018/01/20(土) 17:05:23.72ID:bEiI/N73
統計学の問題なのですが、問4(2)のやり方、答えがわかりません。教えてください。

https://i.imgur.com/2tYtqzV.jpg
0011132人目の素数さん垢版2018/01/20(土) 18:22:59.12ID:2JsoyV0W
>>5
X=-1で最小となりx=-3のときy=5、x=2のときy=15である2次関数の式を求めよ
Y=a(x+1)+b…1
5=a(-3+1)+b@
15=a(2+1)+bA
@とAで連立ですか?
0012132人目の素数さん垢版2018/01/20(土) 18:34:33.02ID:U5FWk8HG
a,bは区間[-1,1]を動く独立な実数である。

(1)c=a+bが区間[-1,1]に含まれる確率を求めよ。
(2)実数a,bが有理数の場合を考えたとき、cが区間[-1,1]に含まれる確率は(1)と同じになるか。
0017132人目の素数さん垢版2018/01/20(土) 19:49:13.90ID:BdHhmenA
[前スレ.982]

C > 90°のときは、B <{A+B,90゚-A}< 90°だから

cos(B)+ cos(90゚)≦ cos(A+B)+ cos(90゚-A), ←凸性

cos(A)+ cos(B)+ cos(C)
= cos(A)+ cos(B)- cos(A+B)
≦ cos(A)+ cos(90゚-A)
≦ 2cos(45゚)   ← 凸性
= √2,
0018132人目の素数さん垢版2018/01/20(土) 20:02:23.91ID:3l6AVmW2
>>15
まずここは数学板だ、化学板で聞くのが妥当
(一応回答すると、弱酸の遊離反応であることは確かだけど、
「バリウムイオンと塩化物イオン」が反応するわけじゃないでしょ。
塩酸由来の水素イオンと、炭酸バリウム由来の炭酸イオンが反応する
⇒だからなかなか中和しない、という話なんだから)
0019132人目の素数さん垢版2018/01/20(土) 20:05:09.06ID:3l6AVmW2
>>17
なるほどね。ありがとう(C≦90°の場合だけ書いていた者より)
0021132人目の素数さん垢版2018/01/20(土) 20:27:39.15ID:UPa9uKwZ
円環の線積分で、なぜdr=adθeθになるか前に先輩に聞いて

r↑=Re↑_r
だから
dr↑=d(Re↑_r)
=dR(e↑_r)+Rd(e↑_r)
いまR=a(一定)だからdR=0
d(e↑_r)=dθ(e↑_θ)より
=adθe↑_θ
∴dr↑=adθe↑_θ

とノートに書いたのですが、de↑_rの向きって接線方向ですよね?
でも、r方向って接線に垂直な方向じゃないんですか?

教えてください


https://i.imgur.com/SzmDWxR.jpg
https://i.imgur.com/TaRSgUa.jpg
0022132人目の素数さん垢版2018/01/20(土) 20:48:28.06ID:bEiI/N73
間違って化学の質問をして申し訳ないです

>>17

> [前スレ.982]
> C > 90°のときは、B <{A+B,90゚-A}< 90°だから
> cos(B)+ cos(90゚)≦ cos(A+B)+ cos(90゚-A), ←凸性

従属関係があって読み取りづらいが、此れは
a<x<b, a<y<bで関数f(x)が閉区間[a,b]で上に凸ならば
f(a)+f(b)≦f(x)+f(y)
が成り立つということだろうか?
だとすれば誤りだ

例えばf(x)=cosx, [a,b]=[30°,90°], (x,y)=(60°, 75°)のとき、
(左辺)=f(a)+f(b)=cos30°+cos90°=(1/2)√3≒0.8660
(右辺)=f(x)+f(y)=cos60°+cos75°=(1/2)+(1/4)(√6-√2)≒0.7588
で(左辺)>(右辺)となり、成立しない

前スレで示された不等式は、x,yがa,bに従属するときにだけ成り立つものではないだろうか
0023132人目の素数さん垢版2018/01/20(土) 20:50:00.26ID:bEiI/N73
変になったのでもう一度レスさせてください

> [前スレ.982]
> C > 90°のときは、B <{A+B,90゚-A}< 90°だから
> cos(B)+ cos(90゚)≦ cos(A+B)+ cos(90゚-A), ←凸性

従属関係があって読み取りづらいが、
此れはa<x<b, a<y<bで関数f(x)が閉区間[a,b]で上に凸ならば、f(a)+f(b)≦f(x)+f(y)
が成り立つということだろうか?

だとすれば誤りだ

例えば、f(x)=cosx, [a,b]=[30°,90°], (x,y)=(60°, 75°)のとき、
(左辺)=f(a)+f(b)=cos30°+cos90°=(1/2)√3≒0.8660
(右辺)=f(x)+f(y)=cos60°+cos75°=(1/2)+(1/4)(√6-√2)≒0.7588
で(左辺)>(右辺)となり、成立しない

前スレで示された不等式は、x,yがa,bに従属するときにだけ成り立つものではないだろうか
0025132人目の素数さん垢版2018/01/20(土) 20:55:53.48ID:0zviVFKV
前スレ801です

>>828
1週間ほど考えました
解けないです限界です
文字式で解くとどうしても冗長になってしまって証明したい等式までの道がまったく見えないです
0026132人目の素数さん垢版2018/01/20(土) 20:59:11.22ID:bEiI/N73
化学の質問や連投とスレ汚し何度も申し訳ないです...

相変わらず、なぜか表示がおかしいので今度は画像で。
https://i.imgur.com/2Np4PXw.jpg
0027132人目の素数さん垢版2018/01/20(土) 21:15:54.25ID:3l6AVmW2
>>8
(1)は、帰無仮説が真の下で
t=(標本平均-0.3)/√(標本不偏分散)/100
が自由度99のt分布に従うことから、両側検定ならば
|t|>1.66(大体このくらい)のとき仮説を棄却するようにすればよい。

母平均が0.4のとき、上記tは非心母数約2,分散1の非心t分布になるから、
検出力つまり|t|>1.66なる確率を数値計算すると、だいたい65%ぐらいかな
(※非心母数=(0.4-0.3)/√(p=0.4のベルヌーイ分布の分散=0.4*0.6)/100)
非心母数は標本数の平方根に比例するので、例えばn=400ぐらいで
検出力は99%ぐらいまで改善されるみたい
0028132人目の素数さん垢版2018/01/20(土) 21:34:20.65ID:bEiI/N73
>>27
(1)母比率の検定なので普通に正規分布使って考えたんですが、それではダメなのでしょうか?
0029132人目の素数さん垢版2018/01/20(土) 22:15:04.29ID:3l6AVmW2
>>28
標本数が100しかないからt検定使うのが普通かと思ったけど
あなたの(1)の解答次第で(2)も変わってくるから、書いてみて
0030132人目の素数さん垢版2018/01/20(土) 22:25:42.39ID:zwkNe933
>>10
はぁ
0031132人目の素数さん垢版2018/01/20(土) 22:47:50.44ID:bEiI/N73
>>29
(1)を正規分布でやって、両側検定の時、37.4以上22.6以外を棄却するとなったのですが、この場合の(2)の答えを良ければ教えてください
0032132人目の素数さん垢版2018/01/20(土) 22:48:27.27ID:3l6AVmW2
>>25
H_nのほうだけ。行列は適当にカンマで区切って書いた
a,b,p,q,rは0と1の間の実数で、
a+b=p+q+r=1は仮定した
A=((a,b),(p,r)),Bは、B^(-1)*A*B=((λ,0),(0,μ))
なる2次正方行列で、B=((c,d),(e,f)),B^(-1)=((g,h),(i,j))
とでもおく
H_n=(A^(n-1))(q,0)転置 の第1成分で、
実際に計算すると
H_n=(ch*λ^(n-1) + dj*μ^(n-1))q
u=chとおくと、BとB^(-1)の積の計算からdj=-u
∴H_n=uq*λ^(n-1) - uq*μ^(n-1)
一般に、αが絶対値1未満の実数なら
∴Σ_[n=1~∞]n(α^(n-1))=1/(1-α)^2
∴Σ_[n=1~∞]nH_n=uq*((1/(1-λ)^2) - (1/(1-μ)^2))
=uq*(λ-μ)*(1-λ-μ)/(λμ-λ-μ+1)^2
λ,μは次の方程式の2根だから、
 x^2 -(a+r)x +(ar-bp)=0
λ+μ=a+r, λμ=ar-bp
またB^(-1)*A*B=((λ,0),(0,μ))
の(1,2)-成分より u*(λ-μ)=b
以上代入して、a+b=p+q+r=1より
(上式)=bq*(2-a-r)/(ar-bp-a-r+1)^2
=(b+p+q)/bq
0033132人目の素数さん垢版2018/01/20(土) 22:48:34.94ID:bEiI/N73
授業で使うt分布表に自由度99の場合が書かれてないので、先生はt分布を使うことを想定してはいないと思います
0034132人目の素数さん垢版2018/01/20(土) 23:11:41.15ID:3l6AVmW2
>>31
そしたら、二項分布 Bi(100,0.4) がその範囲(37.4以上22.6以外)
に落ちる(=r:0.3が正しく棄却される)確率を求めればよい。
正規分布近似して雑に見積もると70%ぐらい?
計算は任せます
0036132人目の素数さん垢版2018/01/20(土) 23:53:51.10ID:BdHhmenA
>>19

[前スレ.983-984]だな。 C≧90°でトチったんぢゃ?

>>22-23

x+y = 135゚ > 120゚ = a+b だから成立しないのでは?

>>17 では
線分(B,sin(B))−(90゚,0)は線分(A+B,cos(A+B))−(90゚-A,cos(90゚-A))より下側にあり、

かつ、それらの中点の横座標は等しい。 (← これが重要)
        ̄ ̄ ̄ ̄ ̄ ̄ ̄ ̄ ̄ ̄
これから >>17 が成立ちます…
0037132人目の素数さん垢版2018/01/20(土) 23:54:50.24ID:nJB107o6
分からないふりの質問者がしたり顔で回答者を問い詰めるスレではないのだが
0038132人目の素数さん垢版2018/01/21(日) 00:04:29.14ID:ZhQ7EiVu
証明ならネットに落ちてるじゃないか。
定期テストの過去問か?
母校の試験対策委員だとしたら承知しないぞ
003936垢版2018/01/21(日) 00:14:04.48ID:SUkk+U6n
>>22-23 (続き)

[補題]
a<x<b,a<y<b で関数 f(x)が閉区間[a,b]で上に凸でかつ a+b=x+y ならば
                                  ̄ ̄ ̄ ̄ ̄ ̄
f(a)+ f(b)≦ f(x)+ f(y)
が成り立つ。

(略証)
fは上に凸だからJensenにより
{(b-x)f(a)+(x-a)f(b)}/(b-a)≦ f(x),
{(b-y)f(a)+(y-a)f(b)}/(b-a)≦ f(y),
辺々たして計算する。
0040132人目の素数さん垢版2018/01/21(日) 00:16:26.81ID:Eb4DuKOz
線形代数の証明についてわからないところがたくさんあります
この質問だけでなく他の質問ものぞいてもらえると助かります

「shoichi_0313 知恵袋」で検索していただきたいのですが
0041132人目の素数さん垢版2018/01/21(日) 00:34:44.86ID:x19IolHw
zが実数となる最小のnを教えてください!すみません、よろしくお願いいたします。
https://i.imgur.com/Bxg5LpB.jpg
0042132人目の素数さん垢版2018/01/21(日) 00:44:20.19ID:m68ScmO7
>>40
質問を書きさえせず、検索して質問を見つけて
答えろというのは、あきれた態度だが。
最近の教育は、こういう生徒を量産しているのか?
0043132人目の素数さん垢版2018/01/21(日) 00:50:50.10ID:o/Xlu9i+
>>35
めんどくさい奴
0044132人目の素数さん垢版2018/01/21(日) 01:01:48.67ID:m68ScmO7
>>35
自明ではないか?→自明どころか反例がある!
というのは、なんの飛躍もない十分一般的な論理だが。
0045132人目の素数さん垢版2018/01/21(日) 01:02:09.21ID:F9K6I4je
>>41
いい方法が見つからない
捨て問でしょ
0047132人目の素数さん垢版2018/01/21(日) 01:10:19.06ID:Oz5ttA7q
これが数学板の実力です
専門板なのに異常にレベルが低い
せいぜい数学の少しできる高校生レベル
0048132人目の素数さん垢版2018/01/21(日) 01:10:44.93ID:9qeWWzMP
>>41
n=0とか
0050132人目の素数さん垢版2018/01/21(日) 01:33:00.74ID:Eb4DuKOz
>>42
最近の教育どうこうではなく、自分がクソなだけなので、、、
0051132人目の素数さん垢版2018/01/21(日) 01:34:42.53ID:x19IolHw
{1−(√2−1)i}/(1+i)
=1−√2i/(1+i)
=1−i(1−i)/√2
=1−(1+i)/√2
=1−e^(iπ/4)

z={1−e^(iπ/4)}^n
2項定理より
z=Σ(k=0→n)nCk(−1)^ke^(ikπ/4)

z∈R⇒Im(z)=0
∴Σ(k=0→n)nCk(−1)^ksin(kπ/4)

k≡0,4 mod8⇒sin(kπ/4)=0
k≡1,3 mod8⇒sin(kπ/4)=1/√2
k≡2 mod8⇒sin(kπ/4)=1
k≡5,7 mod8⇒sin(kπ/4)=−1/√2
k≡6 mod8⇒sin(kπ/4)=−1

色々検討しましたがこれと言ったものがないです
0052132人目の素数さん垢版2018/01/21(日) 01:36:56.79ID:F9K6I4je
{1-(√2-1)i}/(1+i)
={(√2-1)-i}/√2
=√(2-√2){√(2-√2)/2-i√(2+√2)/2}
=√(2-√2){cos(3π/8)-isin(3π/8)}
なので、n乗して偏角がπの整数倍となるためにはn=8

これが模範回答やな
0053132人目の素数さん垢版2018/01/21(日) 01:40:52.80ID:xXFyQU5T
>>40
質問がたくさんあって暇潰しにはちょうど良さそうだが、その中でも特にこれが知りたい!ってのはないの?
0054132人目の素数さん垢版2018/01/21(日) 01:45:44.58ID:Eb4DuKOz
>>53
しいて言えば
・A^(-1)v=β^(-1)v
・複素数(一番古い質問)
・(V(α)の次元)≦(C^nの次元)
です!
0055132人目の素数さん垢版2018/01/21(日) 01:51:53.24ID:8SyDLltC
やっぱ1/8πシリーズの三角比も覚えておいた方がいいんだろうなぁ
滅多に使い道なさそうだけど

この問題なら、とりあえず二乗してみて綺麗になったよ。やったね。
って感じの解法をとってしまいそう。
0056132人目の素数さん垢版2018/01/21(日) 01:59:37.38ID:SUkk+U6n
>>41 (蛇足)

>>51 より
ζ = 1 - e^(iπ/4)= -2i sin(π/8)e^(iπ/8),

ζ^2 = -2{1 - cos(π/4)}e^(iπ/4)= -(2-√2)e^(iπ/4),

ζ^4 = 2(√2 -1)^2 i

ζ^8 = -4(√2 -1)^4 ∈ Q[√2]⊂ R
0057132人目の素数さん垢版2018/01/21(日) 02:14:45.90ID:iJo2/zRH
()内が 1-e^(iπ/4) であることさえ気づければ、
1-e^(iπ/4) の偏角は複素数平面で図示することによっても分かるね
0060132人目の素数さん垢版2018/01/21(日) 02:43:40.89ID:Eb4DuKOz
>>59
上から順に

・対角化可能、固有ベクトル、基底の存在性の証明
・ユニタリ行列、直交系の証明について
・正則行列、ユニタリ行列、上三角行列、一意性の証明
・固有空間と次元について
・シュヴァルツ不等式、三角不等式(複素数)の証明
・特性根、ベキ零行列の証明について
0061132人目の素数さん垢版2018/01/21(日) 02:49:52.54ID:o/Xlu9i+
>>60
本読んだら全部書いてあるけど
0062132人目の素数さん垢版2018/01/21(日) 02:58:29.22ID:x19IolHw
3/8π...覚えてないです

結局一番上手い方法ってなんでしょう
0063132人目の素数さん垢版2018/01/21(日) 03:06:07.12ID:h6hbZk2i
存命中で世界最高の数学者って誰ですか?
マキシム・コンツェビッチとか望月新一とかリチャード・テイラーとかアラン・コンヌとか
アンドリュー・ワイルズとかグレゴリー・ペレルマンとかテレンス・タオあたり?
0064132人目の素数さん垢版2018/01/21(日) 03:12:39.49ID:o/Xlu9i+
>>51
arg(a+bi)=arctan2(a,b)
arg((a+bi)^2)=arctan2(a^2-b^2,2ab)
arctan2(2√2-2,2-2√2)=-(1/4)π
arctan2(1,1-√2)=-(1/8)π
arg(a/b)=arga-argb=-(1/8)π-(1/4)π=-(3/8)π
arg((a/b)^8)=-3π
n=8
0066132人目の素数さん垢版2018/01/21(日) 09:09:18.88ID:TGpBI6pd
耳栓をしたら世界が変わってワロタ
0067132人目の素数さん垢版2018/01/21(日) 09:25:41.09ID:Eb4DuKOz
>>61
どういうことですか?
0068132人目の素数さん垢版2018/01/21(日) 09:43:17.08ID:Eb4DuKOz
>>61
どういう風に書いてあるか教えてください
0069132人目の素数さん垢版2018/01/21(日) 10:22:23.72ID:x19IolHw
>>39
ようやく「凸性」の正体が現れたね
x,yがa+b=x+yという従属関係にあれば、f(a)+f(b)≦f(x)+f(y)は正しい
(なお線分による説明ではx=yのときが微妙になるので、補題を用いた方がよい。
その後の計算 cosA+cos(90°-A)≦2cos45° で困ることになります。)

これで、cos曲線の凸性を認めて補題を使えば、加法定理を用いなくても最大値を示すことができるという道筋を追うことができた。
その道筋では確かに加法定理は使われていないんだが、前提となっているcos曲線の凸性を示すのにcosの微分を用いているのだとすれば、そこに加法定理が関わっている可能性があるので、問題設定上の矛盾を感じなくもない

なお一般に凸性とは☆の不等式が成り立つことを指し、凸性から導かれる不等式には別の呼び方をしたり、その式を明示したりする。
補題の不等式を用いる場合は、「凸性」とは言わずに、その式を明示するのがよい。


閉区間[a,b]で定義された実数値関数f(x)が、0≦t≦1(*)を満たす任意の実数tと、a≦x<y≦bを満たす任意の実数x,yに対して、

f(tx+(1-t)y)≧tf(x)+(1-t)f(y) …☆

が成り立つとき、閉区間[a,b]で関数f(x)は上に凸な関数と定義されます。
0070132人目の素数さん垢版2018/01/21(日) 10:32:26.57ID:Eb4DuKOz
>>62
2{cos(π/8)}^2-1=cosπ/4=√2/2
{cos(π/8)}^2=(2+√2)/4
cos(π/8)=√(2+√2)/2
sin(π/8)=√(2-√2)/2
cos(π/2-π/8)=√(2-√2)/2
sin(π/2-π/8)=√(2+√2)/2
0071132人目の素数さん垢版2018/01/21(日) 11:01:51.08ID:Eb4DuKOz
>>62
ちなみに{1-(√2-1)i}/(1+i)=(2-√2)/2-√2/2i
これをc(a+bi), a^2+b^2=1という形にしたいのでc(a+bi)=(2-√2)/2-√2/2iとして
ac=(2-√2)/2, bc=-√2/2
(ac)^2+(bc)^2=c^2=2-√2
c>0とすると(c<0で計算してもよい)
c=√(2-√2) よってa=√(2-√2)/2
b=√(2+√2)/2とすればいいと思います
0072132人目の素数さん垢版2018/01/21(日) 11:59:45.27ID:u//BWKFO
m*n行列Aの1行目1列目を除いた(m-1)*(n-1)行列をA_11とするとき
rank(A)-rank(A_11)の最大値、最小値を求めろ
また、そのときの行列Aをそれぞれ具体的に書け
0075132人目の素数さん垢版2018/01/21(日) 13:26:15.57ID:Kk6ixi2r
6x^2+5x-6=0 のとき (6x^2+5x)×√12x^2+10x+24 の値を求めよ

これどうやればよいか教えてください
ルートは 24 までです
0076132人目の素数さん垢版2018/01/21(日) 13:33:06.01ID:PeAwZJ66
6x^2+5x-6=0だから、6x^2+5x=6
2倍して12x^2+10x=12
この2つを代入すると、
6×√(12+24)=36
0078132人目の素数さん垢版2018/01/21(日) 14:40:32.70ID:xk8qFx1I
0°≦θ≦180°とする。3sinθ=√2のとき,cosθ=ア
Tanθ=イ教えて下さい
0079132人目の素数さん垢版2018/01/21(日) 18:32:25.14ID:xk8qFx1I
次に示す集合A.BについてA∩BとA∪Bを求めよ A={x|xは6の正の約数} B={x|xは12の約数|
どう解くのでしょうか?全然わかりません
0080132人目の素数さん垢版2018/01/21(日) 18:46:15.91ID:xXFyQU5T
>>78
(sinθ)^2+(cosθ)^2=1とtanθ=sinθ/cosθを使う
>>79
AとBの要素を列挙してその共通集合や合併集合を求めれば良い
0081132人目の素数さん垢版2018/01/21(日) 18:53:01.80ID:FV4MdDLa
これの(3)なんですが
解答が言ってることは最後まで理解できるんですけど、「」で括ったところはどうやったら思い付けるんでしょうか
0<={(a_n+1)-c}/{(a_n)-c}<1となるところまでは自力で思いついたんですけどそこで詰まりました
初見で完解するとしたら(an+c)/2と1の間に定数rをとったら証明できるという見通しが立つもんなんですか?

https://i.imgur.com/5G2sWr1.jpg
https://i.imgur.com/Zwz7i0I.jpg
0082132人目の素数さん垢版2018/01/21(日) 18:58:31.99ID:xk8qFx1I
>>80
{x|xは6の正の約数} B={x|xは12の約数|
1.2.3.6       1.2.3.4.6.12
A∩B=1.2.3.6
A∪B=1.2. 3.4.6.12
が答ですか?
0083132人目の素数さん垢版2018/01/21(日) 19:04:58.75ID:CilVwVvR
>>81
2次方程式解いて見当つける
y=1/2(x^2+b)のグラフ書いたら
収束しそうと分かる
差を取ったらきれいに因数分解
縮小写像だと示せそう
あとは頑張る
0084132人目の素数さん垢版2018/01/21(日) 19:14:59.62ID:Y5Z5kKbp
AB=4、BC=6、CA=5である三角形ABCにおいて、角Aの二等分線と辺BCの交点をDとする。
(1)BDの長さ
(2)三角形ABCの外接円の半径R
(3)三角形ABCの外接円において、点Aを含まない弧BC上に点Eをとる。
辺EDが角BECの二等分線になるとき、三角形ABCと三角形EBCの面積比は?

(1)(2)はともかく、(3)が解き方からわかりません。(3)について解法を含めて教えていただけると幸いです。
どなたかよろしくお願いします。
0085132人目の素数さん垢版2018/01/21(日) 20:26:06.09ID:nJMudanV
f(x,y)=e^{(x^2)-(y^2)}を最大にする点(x,y)及び最小にする点(x,y)をそれぞれすべて求めよ

どなたか計算過程も含めてよろしくお願い致します。
0088132人目の素数さん垢版2018/01/21(日) 20:52:28.23ID:sysCdItI
■ピタゴラスの定理の短い二辺の長さ

3 4 

5 12 

8 15 から

3+5=8
3+12=15
3x4=12
3x5=15
5+15=8+12

なぜここまで美しい関係性があるのか?
0090132人目の素数さん垢版2018/01/21(日) 21:07:29.67ID:nRJOOFFL
X^3+2=0 mod (p^2+q~2)  ただし p, q はキ素数
0092132人目の素数さん垢版2018/01/21(日) 21:41:44.44ID:nRJOOFFL
f(ax+by+cz) for a+b+c=0
009384垢版2018/01/21(日) 22:19:58.47ID:Y5Z5kKbp
>>87
返信ありがとうございます。
しかしそのヒントだけではわかりませんでした。すみません。

良ければもう少し詳細に教えて頂けるとありがたいです。
009584垢版2018/01/21(日) 22:44:24.78ID:Y5Z5kKbp
>>94
ありがとうございます。正しく出すことが出来ました。
とても助かりました。
0096132人目の素数さん垢版2018/01/22(月) 00:08:51.46ID:EFMpeTLH
>>69

Jensenの式
f(ta+(1-t)b)≧ t f(a)+(1-t)f(b) … ☆

で、t =(b-x)/(b-a)、t =(b-y)/(b-a)とおいたものが >>39 の2つの式
0097132人目の素数さん垢版2018/01/22(月) 01:27:33.26ID:MdX5CUh0
苦から逃れたい。
無になってもう二度と有になりたくない。
0098132人目の素数さん垢版2018/01/22(月) 02:31:43.65ID:EFMpeTLH
>>79 >>82
Aでは「6の正の約数」と云い、Bでは「12の約数」と言っています。
        ̄ ̄
B ={±1,±2,±3,±4,±6,±12}
だと思いましょう。

>>81
(1)から、
 c/2 ≦ r_n ≦(1+c)/2 = r
とおく。題意から、
 0≦b<1、0≦c<1,r<1

>>84
余弦定理から
cos(A)= 1/8,
cos(A/2)= 3/4,
cos(B)= 9/16,
cos(C)= 3/4,
∴∠DAC = ∠C
∴ AD = CD
CD = 10/3,
BD = 8/3,
正弦定理から
R = 8/√7,
△ABC =(15/4)√7,
0100132人目の素数さん垢版2018/01/22(月) 10:17:17.92ID:EqBRgif4
>>99
円(x-a)^2+(y-b)^2=a^2+b^2の
内部に点(x,y)があるとき(x-a)^2+(y-b)^2<a^2+b^2となる
外部に点(x,y)があるとき(x-a)^2+(y-b)^2>a^2+b^2となる
点Aが内部かつ点Bが外部の場合と
点Aが外部かつ点Bが内部の場合とについて
それぞれ不等式を立てて解く
0101132人目の素数さん垢版2018/01/22(月) 10:39:12.94ID:EqBRgif4
補足
・円(x-a)^2+(y-b)^2=a^2+b^2の中心は(a,b)で、半径は√(a^2+b^2)となる
 円の中心から原点までの距離と、円の半径が等しいので、この円は中心がどこにあっても原点を通る
・点(x,y)が円の内部にあるとは、円の周より点(x,y)のほうが円の中心に近いということ
 つまり点(x,y)が円(x-a)^2+(y-b)^2=a^2+b^2の内部にあるとは、原点より点(x,y)のほうが点(a,b)に近いということ
 同様に点(x,y)が円(x-a)^2+(y-b)^2=a^2+b^2の外部にあるとは、原点より点(x,y)のほうが点(a,b)から遠いということ
0104132人目の素数さん垢版2018/01/22(月) 13:11:00.65ID:ABwLSc8N
xの範囲が0≦x≦a(aは正の定数)である二次関数y=x^2-6x+11の最大値をM最小値をmとする。1)0<a<3のときM=ア、m=イ 2)3≦a≦6のときM=ウ、m=エ どう解くのでしょうか?お願いします
0105132人目の素数さん垢版2018/01/22(月) 13:56:57.92ID:EFMpeTLH
>>99-100

・点A(0,2)が円の内部にあるとき
(0-a)^2 +(2-b)^2 < aa+bb,
 4(1-b)< 0,
 b>1,
,
・点A(0,2)が円の外部にあるとき
 b<1

・点B(3,-1)が円の内部にある解き
(3-a)^2 +(-1-b)^2 < aa+bb,
 3(3-2a)+1+2b < 0,
 b < 3a-5,

・点B(3,-1)が円の外部にある朱鷺
 b > 3a-5,

>>104
 y =(x-3)^2 + 2,
 軸は x=3
 0<x<3 で単調減少、3<x で単調増加
 1)0<a≦3 のとき M=11,m=aa-6a+11,
 2)3≦a≦6 のとき M=11,m=2,
 3)6≦a のとき   M=aa-6a+11,m=2,
0110132人目の素数さん垢版2018/01/22(月) 19:05:08.25ID:CRqGHbc8
どや顔で2番歌う奴ぅwww
0111132人目の素数さん垢版2018/01/22(月) 19:28:49.78ID:X4OXGWnG
夜更け過ぎに雪江と変わるだろう
0112132人目の素数さん垢版2018/01/22(月) 20:27:28.97ID:jaEpfB5w
実数を、素数を基底とした、素因数分解後の指数の無限次元のベクトルと考えた時、
このベクトル和って何か決まった関係があるのでしょうか。
また、それらはどういった数学の分野になりますでしょうか。
0114132人目の素数さん垢版2018/01/22(月) 21:10:48.07ID:CRqGHbc8
線形じゃないでしょ

2=a↑
3=b↑
2+3=5=c↑
0115132人目の素数さん垢版2018/01/22(月) 21:51:03.05ID:EI7sv2wc
|x+2|-|x-1|>x を解くとき、どう場合分けすればいいのですか?教えてください。
0116132人目の素数さん垢版2018/01/22(月) 21:56:04.60ID:jaEpfB5w
用語が不明瞭ですいません。

整数を素因数分解
6=2*3=2^1*3^1
で指数部をベクトル表記すると[1 1]T
60=2^2 * 3^1 * 5^1
で指数部をベクトル表記すると[2 1 1]T

実数は、指数部が負をとることを許容すると表記可能。

などとした時に、これらのベクトルについての体系的な数学の分野ってありますでしょうか。
0117132人目の素数さん垢版2018/01/22(月) 21:58:06.95ID:Ezo/MJwc
>>116
>実数は、指数部が負をとることを許容すると表記可能。

本当に?無理数も?
0118132人目の素数さん垢版2018/01/22(月) 22:04:44.27ID:IGcpJF29
>>115
x<-2
-2≦x<0
0≦x<1
1≦x

かなぁ…(脳筋)
0119132人目の素数さん垢版2018/01/22(月) 22:59:35.03ID:fbEiot6U
>>117 有理数ですね
0120132人目の素数さん垢版2018/01/22(月) 23:10:46.31ID:tJ4xjYN8
帝釈天とグレゴリー・ペレルマンはどっちの方が賢いですか?
0121132人目の素数さん垢版2018/01/22(月) 23:19:58.69ID:uDem0Ezi
イエスキリストです
0122132人目の素数さん垢版2018/01/22(月) 23:24:08.37ID:tJ4xjYN8
イエス・キリストと大日如来はどっちの方が凄いですか?
0123132人目の素数さん垢版2018/01/22(月) 23:27:29.39ID:uDem0Ezi
イエスキリストですね
0124132人目の素数さん垢版2018/01/22(月) 23:28:39.70ID:tJ4xjYN8
>>123
理由を教えてください。
0125132人目の素数さん垢版2018/01/22(月) 23:43:03.99ID:uDem0Ezi
イエスキリストだからです
0126132人目の素数さん垢版2018/01/22(月) 23:44:26.12ID:tJ4xjYN8
>>125
それはどういうことですか?
もう少し詳しく教えてください。
0129132人目の素数さん垢版2018/01/23(火) 01:04:39.80ID:G4Zn7LtG
>>116

p-adic numbers をしらべたら
0130132人目の素数さん垢版2018/01/23(火) 01:12:39.58ID:Nv6hHyQu
>>116
ベクトルの和が数の積、スカラー倍が定数乗で与えられるだけで
普通の線型代数学でそれなりに広く賄える気がするけど
0131132人目の素数さん垢版2018/01/23(火) 01:29:58.91ID:HBnI6seA
>>128
あー。0いらないな
0133132人目の素数さん垢版2018/01/23(火) 01:52:35.11ID:9xWmKawn
>>115

| |の中が負の場合と非負の場合とに分ける

x<-2 のとき、(左辺)= -3
-2≦x<1 のとき、(左辺)= 2x+1
1≦x のとき、(左辺)= 3

答 x<-3,-1<x<3

かなぁ…(成り済し筋)


>>111
サイレンナイト、ホーリーナイト
0134132人目の素数さん垢版2018/01/23(火) 04:14:58.52ID:YqZeQcev
F1(x)=1(|x|≦1/2),0(1/2<|x|)から始めて、
Fn+1(x)=(Fn*F1)(x)=∫[s=-∞〜∞]Fn(s)F1(x-s)dsのように畳み込み積分で関数を作っていくと
F2(x)=1-|x|(|x|≦1),0(1<|x|)
F3(x)=3/4-|x|^2(|x|≦1/2),9/8-(3/2)|x|+(1/2)|x|^2(1/2<|x|≦3/2),0(3/2<|x|)
F4(x)=2/3-|x|^2+(1/2)|x|^3(|x|≦1),4/3-2|x|+|x|^2-(1/6)|x|^3(1<|x|≦2),0(2<|x|)
のようになるんだけど、一般のnでFn(x)を表すにはどうしたらいいですか?
なんかどんどん複雑になって法則が見えないです
0135132人目の素数さん垢版2018/01/23(火) 05:39:40.34ID:1imRO4DQ
>>134
n→∞のときが問題.中心極限定理.
0136132人目の素数さん垢版2018/01/23(火) 07:07:05.45ID:YqZeQcev
問題は極限がどうなるかではなくnが有限のときの一般式なのですが
0137132人目の素数さん垢版2018/01/23(火) 07:51:45.83ID:tWfi7Rgr
>>134
法則ないよ
0138132人目の素数さん垢版2018/01/23(火) 09:10:17.74ID:1imRO4DQ
>>134
F1をフーリエ変換すれば F1~ = (exp(iω/2)-exp(-iω/2))/iω = sin(ω/2)/(ω/2). これを sinc(ω/2)と書くこともある.
Fnは F1のn重たたみこみだから,そのフーリエ変換は Fn~ =( F1~)^n.
これから Fnの一般式は Fn(x) = ∫(sinc(ω/2))^n exp(iωx)dω.
こんなの,閉じた式にはならない.中心極限定理より,正規分布の曲線を折れ線
近似したものだ.
0139132人目の素数さん垢版2018/01/23(火) 09:28:31.52ID:1imRO4DQ
F1は -1/2<x<1/2の値をとる一様乱数の確率密度関数 xとみなすことができる.
すると,F2は y = x+x の確率密度,F3は y = x+x+x の確率密度….
一般にFn は上記の乱数 n個の和の確率密度.一方,これは中心極限定理より
正規分布に近づくことが知られている.この方法で実用的に正規乱数を生成する
こともできる.6個の和 F6 を作ると標準偏差 1となって,N(0,1)として
使用できるのは有名.
0140139垢版2018/01/23(火) 09:36:15.40ID:1imRO4DQ
一様乱数 12個の和,F12 だた.
0141132人目の素数さん垢版2018/01/23(火) 16:02:59.80ID:9xWmKawn
まづ、n個の小区間に分けて考えましょう。
区間の境界を b_L = -n/2,-n/2+1,…,n/2-1,n/2 とします。
これらの外側では F_n(x)= 0 です。

F_n は C^(n-2)級、つまり(n-2)階微分まで連続です。
また F_n は各区間内では n-1次の多項式ですから、
区間の境界で a_L・(x-b_L)^(n-1)だけ増減します。

F_n(x)= 納L=0,n]a_L g(x-b_L)
ここに、
 a_L = (-1)^L・C[n,L]/{(n-1)!},
 b_L = L - n/2, (L=0,1,…,n)
 g(x)= x^(n-1)(x≧0), 0(x≦0)

納k=-∞,∞]F_n(x+k)= 1,

数セミ・エレ解 問題1(出題:2016年4月号、解説:7月号)

>>139-140
F_n(x)の分散は n/12
0142132人目の素数さん垢版2018/01/23(火) 16:18:32.94ID:LQ6yH08u
自然数aを8で割ると5余り、自然数bを8で割ると4余る。a+b+abを8で割った時の余りを求めよ。

これどうやるか教えてください。お願いします
0144132人目の素数さん垢版2018/01/23(火) 16:24:57.81ID:9xWmKawn
>>141
畳み込み、というのは一種のボカシ(移動平均)操作。
角がとれて丸くなり、C-級が1つ上がる。
0145132人目の素数さん垢版2018/01/23(火) 16:43:49.21ID:AOjlEoOm
行列Aが小行列B,Cを用いてA=[[B,0],[0,C]]と表されるとき、rankA=rankB+rankCを示せ
直感では分かるのですが証明の仕方が分からないのでお願いします
0146132人目の素数さん垢版2018/01/23(火) 16:54:28.66ID:LQ6yH08u
>>143
a=8n+5, b=8m+4とおいて
a+b+ab=(8n+5)+ (8m+4)+ (8n+5) (8m+4)
=8{(n+m)+8mn+(4n+5m)}+29
よってa+b+abを8で割った時の余りは29

となったのですが答えを見ると5なんですよね
どこが間違ってるのか教えてください
0149132人目の素数さん垢版2018/01/23(火) 17:28:57.13ID:VwfYa9v9
学校で出された問題なのですがヒントもなくよく分かりません、お願いします

いまAさんは15歳で、お父さんの年齢は44歳です。
何年か経つと、Aさんの年齢を2倍した数が、お父さんの年齢以上になることを、中間値の定理を用いて説明しなさい。
0150132人目の素数さん垢版2018/01/23(火) 17:34:08.41ID:Y2nznxrT
年齢は整数値をとるから不連続関数なんだけど、中間値の定理は適用できるの?
0152132人目の素数さん垢版2018/01/23(火) 21:35:46.86ID:VAUy+8Se
2/5x=1/3(x+3/2)
これの分母をはらう工程を教えてください。
0153132人目の素数さん垢版2018/01/23(火) 21:56:33.35ID:y0AZLXJD
私文で厨房の家庭教師やってるんだけど数学わからん
なんで4倍になるの?って聞かれて考えたけど誰かわかりますか?
https://imgur.com/a/dDjwI.jpg
0154132人目の素数さん垢版2018/01/23(火) 21:57:21.99ID:ZhvKN5Lo
>>153
見れません
0155132人目の素数さん垢版2018/01/23(火) 22:00:11.81ID:SXlTxUnV
>>142
それ、4ステップだろ
そのページの上のところにやり方書いてなかったか?

ついでに29はまだ8で割れる
0157132人目の素数さん垢版2018/01/23(火) 22:07:45.94ID:ZhvKN5Lo
>>156
△BCDは△BCQの2倍
平行四辺形は△BCDの2倍
合わせて4倍ですね
0158132人目の素数さん垢版2018/01/23(火) 22:08:33.44ID:Y2nznxrT
>>153
カテキョやっててそれかい、って思ったけど文科じゃしょうがないか…
・底辺と高さが同じなら平行四辺形の面積は三角形の2倍
・底辺が同じなら三角形の面積は高さに比例する
・三角形の高さは頂点から底辺(またはその延長)に下ろした垂線の長さ。ただし比率をみるだけなら斜めでもかまわない
これで説明できる?
0160132人目の素数さん垢版2018/01/23(火) 22:10:35.45ID:y0AZLXJD
普通にCD底辺にしたら自己解決したわ。スマン
0161132人目の素数さん垢版2018/01/23(火) 22:18:20.75ID:pfW7rO4C
アラブの石油王とマサチューセッツ工科大学の超天才数学者はどっちの方が凄いですか?
0162132人目の素数さん垢版2018/01/23(火) 22:20:51.52ID:ZhvKN5Lo
イエスキリストです
0163132人目の素数さん垢版2018/01/23(火) 22:22:26.77ID:pfW7rO4C
イエス・キリストとゼウスはどっちの方が凄いですか?
0164132人目の素数さん垢版2018/01/23(火) 22:22:36.40ID:ZhvKN5Lo
イエスキリストです
0165132人目の素数さん垢版2018/01/23(火) 22:24:06.87ID:VwfYa9v9
大学の数学って実数から始めますけど、あれをやらずに高校のままの数の定義だと何処で困るんですか?
広義積分ですか?
0166132人目の素数さん垢版2018/01/23(火) 22:24:20.04ID:pfW7rO4C
イエス・キリストと望月新一はどっちの方が凄いですか?
0167132人目の素数さん垢版2018/01/23(火) 22:32:59.02ID:tWfi7Rgr
>>165
どこで困った?
0168132人目の素数さん垢版2018/01/23(火) 23:28:19.51ID:MBTxmQ9w
2次方程式x^2-3x-1=0の解をa.bとするとき(b+1)/a,(a+1)/bを解にもつ2次方程式を求めよ。
教えて下さい
0169132人目の素数さん垢版2018/01/23(火) 23:38:08.18ID:9xWmKawn
>>145

行列Aの任意の小行列A’は
 A’=[[B',0'],[0',C']]
 B’は B の小行列(または B自身)、
 C’は C の小行列(または C自身)
と一意的に表わされ、その行列式は
 |A'|=|B'||C'|
つまり
 A’が正則 ⇔ B’も C’も正則

rank(A)= a,rank(B)= b,rank(C)= c と略す。

A のa次の正則な小行列 A’が存在する。
これを分解してできる Bの小行列B',Cの小行列C’も正則。
a = dim(A')= dim(B')+ dim(C')≦ rank(B) + rank(C),

B のb次の正則な小行列 B’,C のc次の正則な小行列 C’が存在する。
これらを並べてできる A’は Aの正則な小行列。
b + c = dim(B')+ dim(C')= dim(A')≦ rank(A),

これらより、a = b + c.
0170132人目の素数さん垢版2018/01/23(火) 23:59:59.11ID:tWfi7Rgr
>>145
<B>と<C>の基底を取ってずらして並べたら<A>の基底になることを証明するだけ
あるいはImB⊕C=ImB⊕ImCを言うだけ
0171132人目の素数さん垢版2018/01/24(水) 00:23:14.59ID:9ooAixL8
>>168

xx-3x-1 =(x-a)(x-b)= xx -(a+b)x +ab,
a+b=3,ab=-1

c+d =(b+1)/a + (a+1)/b
={b(b+1)+ a(a+1)}/(ab)
=(a+b)(a+b+1)/(ab)-2
=(3・4)/(-1)-2
= -14,

cd ={(b+1)/a}・{(a+1)/b}
= 1 +(a+b+1)/(ab)
=(-1+3+1)/(-1)
= -3,

(x-c)(x-d)= xx -(c+d)x +cd
= xx+14x-3.
0173132人目の素数さん垢版2018/01/24(水) 03:57:51.52ID:sYCbbjiO
https://i.imgur.com/65i17zi.jpg
なんでこれの係数行列がこうなるの?
この連立方程式の係数行列の右のほうが全て0になるってのが想像つかない
0176132人目の素数さん垢版2018/01/24(水) 15:49:14.16ID:VB7iGd8Q
>>168
これ数Uですか?
0177132人目の素数さん垢版2018/01/24(水) 18:42:12.52ID:QMksidP5
>>168
解の公式でa、bを求める
(b+1)/a,(a+1)/bに代入する
{x-(b+1)/a}{x-(a+1)/b}を計算する

強引だけどこんな感じでやってみれば?
0178132人目の素数さん垢版2018/01/24(水) 19:45:58.09ID:JUUlgt+A
https://i.imgur.com/a5g89Gr.jpg

行列の固有ベクトルの問題
赤字がテキストの解答なんですが
どこからこの値が出てきたのか分かりません
0179132人目の素数さん垢版2018/01/24(水) 19:55:37.52ID:GIQ0Lkp8
1から100までの整数のうちで、2,3,5の倍数でない数がいくつあるか教えてください。
式もお願いします。
0180132人目の素数さん垢版2018/01/24(水) 20:36:30.88ID:hyx9rJmE
100個くらい自分で調べろよ
0181132人目の素数さん垢版2018/01/24(水) 21:15:20.40ID:r1+h+8M5
亜保なことを
0182132人目の素数さん垢版2018/01/24(水) 21:29:49.84ID:CMOfC/O8
7 11 13 17 23 27 29 31 37 41 43 47 57 59 61 67 71 73 79 83 87 89 91 97
0183132人目の素数さん垢版2018/01/24(水) 21:34:13.92ID:eWVm4f7A
(プラスマイナスk-√3/2)X+2マイナスプラスak=0
任意のXについて成り立つことから

k-√3/2=0かつ2-ak=0
-k+√3/2=0かつ2+ak=0
これより
k=√3/2 a=4√3/3
k=-√3/2 a=-4√3/3

なんでこうなるか教えてください
0184132人目の素数さん垢版2018/01/24(水) 22:15:21.04ID:aSRphMzl
>>178
L1 ノルムなんだろ。
0185132人目の素数さん垢版2018/01/24(水) 22:18:01.26ID:+KTAzAIB
1, 7, 11, 13, 17, 19, 23, 29, 31, 37, 41, 43, 47, 49, 53, 59, 61, 67, \
71, 73, 77, 79, 83, 89, 91, 97

の26個だね
0188132人目の素数さん垢版2018/01/24(水) 23:06:46.04ID:W6bU7EHl
>>179
2の倍数:50
3の倍数:33
5の倍数:20
2と3の公倍数:16
2と5の公倍数:10
3と5の公倍数:6
2と3と5の公倍数:3
100-{50+33+20-16-10-6+3}=26
0190132人目の素数さん垢版2018/01/24(水) 23:27:24.53ID:+KTAzAIB
>>186

この問題は自由度が多すぎる。 つまり解はたくさんある。
とりあえず
1, 3, 4, 9, 10, 12, 13, 27, 28, 30, 31, 81, 82, 84, 85, 243, 244, \
246, 247, 729, 730, 732, 733, 2187, 2188, 2190, 2191, 6561, 6562, \
6564, 6565, 19683, 19684, 19686, 19687, 59049, 59050, 59052, 59053, \
177147, 177148, 177150, 177151, 531441, 531442, 531444, 531445, \
1594323, 1594324, 1594326, 1594327, 4782969, 4782970, 4782972, \
4782973, 14348907, 14348908, 14348910, 14348911, 43046721, 43046722, \
43046724, 43046725, 129140163, 129140164, 129140166, 129140167, \
387420489, 387420490, 387420492, 387420493, 1162261467, 1162261468, \
1162261470, 1162261471, 3486784401, 3486784402, 3486784404, \
3486784405, 10460353203, 10460353204, 10460353206, 10460353207, \
31381059609, 31381059610, 31381059612, 31381059613, 94143178827, \
94143178828, 94143178830, 94143178831, 282429536481, 282429536482, \
282429536484, 282429536485, 847288609443, 847288609444, 847288609446, \
847288609447, 2541865828329, 2541865828330, 2541865828332, \
2541865828333}

から100番目の
2541865828329 を答えとする。

あまりいい問題とはいえない。
0191132人目の素数さん垢版2018/01/24(水) 23:35:00.07ID:PaCxyogK
答えは891です
0192132人目の素数さん垢版2018/01/24(水) 23:38:37.35ID:Sk+od6EP
>>184
おっしゃる通りでした…他の問題も全部L1でした
ありがとうございます
0193171垢版2018/01/24(水) 23:43:22.39ID:9ooAixL8
>>168
2次方程式 xx -(s)x +(t)= 0(t≠0)の解を a,b とするとき

(b+1)/a,(a+1)/b を解にもつ2次方程式は
xx +{2 - s(s+1)/t}x +{1 +(s+1)/t}= 0,

>>172
仰るとおりです。

>>177
それは御免蒙ります。
0194132人目の素数さん垢版2018/01/24(水) 23:49:35.60ID:+KTAzAIB
答えは3^26です
0197132人目の素数さん垢版2018/01/25(木) 00:34:01.15ID:XbXfE70D
>>190

とりあえず
1,3,4,9, 10,12,13,27, 28,30,31,36, 37,39,40,81,
82,84,85,90, 91,93,94,108, 109,111,112,117, 118,120,121,243,
244,246,247,252, 253,255,256,270, 271,273,274,279, 280,282,283,324,
325,327,328,333, 334,336,337,351, 352,354,355,360, 361,363,364,729,
730,732,733,738, 739,741,742,756, 757,759,760,765, 766,768,769,810,
811,813,814,819, 820,822,823,837, 838,840,841,846, 847,849,850,729,
973,975,976,981, …

から100番目の 981 を答えとする。

あまりいい問題とはいえない。
0199132人目の素数さん垢版2018/01/25(木) 00:48:38.25ID:XbXfE70D
>>196

OA=OB=OC=OD=r,

正弦定理で △ =(1/2)rr sin(中心角)

△OAB = △OBC =(1/2)rr sin(2π/3)=(√3)/4 rr,
△OCD =(1/2)rr sin(π/2)=(1/2)rr,
△ODA =(1/2)rr sin(π/6)=(1/4)rr,
◇ABCD =(3+2√3)/4 rr = 16(3+2√3),

∴5
0200132人目の素数さん垢版2018/01/25(木) 00:58:05.64ID:9jep5JTM
>>190

とりあえず
1,3,4,9, 10,12,13,27, 28,30,31,36,1,3,4,9, 
10,12,13,27, 28,30,31,36 ,1,3,4,9, 10,12,13,27, 28,30,31,36,1,3,4,9, 10,12,13,27, 28,30,31,
361,3,4,9, 10,12,13,27, 28,30,31,361,3,4,9
, 10,12,13,27, 28,30,31,36
,981,1,3,4,9, 10,12,13,27, 28,30,31,36]

から100番目の 10 を答えとする。あほ問題とはいえる。

漫才だね
0201132人目の素数さん垢版2018/01/25(木) 01:06:01.07ID:oR1Nttvx
100 = 91 + 6 + 3; 91 = 13(13+1)/2; 6 = 3(3+1)/2; 3 = 2(2+1)/3
(3^13 -1)/2 + (3^3-1)/2 + (3^2-1)/2
= (1594323 - 1)/2 + (27-1)/2 + (9-1)/2
= 797161 + 13 + 4
= 797178
0205132人目の素数さん垢版2018/01/25(木) 11:32:19.00ID:EFuBapDh
N≡39 mod97
N≡6 mod83
N≡25 mod29
これらをみたす自然数Nを全て求めよ

とっかかりもわからないのでよろしくおねがいします
0206132人目の素数さん垢版2018/01/25(木) 11:49:09.94ID:4dXuSK1x
>>205
中国人余剰定理
0207132人目の素数さん垢版2018/01/25(木) 15:31:08.18ID:9jep5JTM
なにもしらない小学生でも解けるね
0208132人目の素数さん垢版2018/01/25(木) 17:14:39.43ID:YJOWUDHL
https://i.imgur.com/D3hodEk.jpg
これが重複順列になる理由がわかりません。
わかりやすく教えていただけませんか?
0209132人目の素数さん垢版2018/01/25(木) 19:48:51.50ID:PSlKswQ5
>>208
玉に順序をつけることができ(順列)、かつ
それぞれの箱に何個でも玉を容れて良い(重複)からです
0210132人目の素数さん垢版2018/01/25(木) 21:27:54.08ID:9jep5JTM
3この箱に8このタマをいれるのは
{{8}, {7, 1}, {6, 2}, {6, 1, 1}, {5, 3}, {5, 2, 1}, {4, 4}, {4, 3,
1}, {4, 2, 2}, {3, 3, 2}}

だが どの箱に入れるかも問題にするのかね?
0211132人目の素数さん垢版2018/01/25(木) 22:18:15.95ID:02yEZNOH
https://i.imgur.com/n7xQV5p.jpg

この計算過程、どこかで間違えてるはずなんですがなにがいけないのですか?
0212132人目の素数さん垢版2018/01/25(木) 22:35:27.46ID:ikFKaeuq
>>205
N≡39 mod97
N≡6 mod83
N≡25 mod29

97*r+(83*29)*a=1 なる整数a(☆)を求めてA=(83*29)*aとおくと
Aは97で割ると1余り、かつ83でも29でも割り切れる、つまり
N≡1 mod97
N≡0 mod83
N≡0 mod29 の解の一つ
同じ要領でB,Cを求め、
39*A+6*B+25*C と置けば与えられた合同方程式の解になる。

(☆)の求め方:いわゆるユークリッドの互除法
83*29=2407=97*24+79, 97=79*1+18, 79=18*4+7,
18=7*2+4, 7=4*1+3, 4=3*1+1
余りが1になったところから、計算を全部さかのぼると、
1=4-3*1=4-(7-4*1)= (18-7*2)-(7-(18-7*2))
=18*2-7*5=(97-79)*2-(79-18*4)*5=2*97-7*79+20*(97-79)
=22*97-27*79=22*97-27*(83*29-97*24)=670*97-27*(83*29)
…残りの計算や、解の吟味はまかせた(NとN+k*(97*83*29)は
同じ連立合同方程式を満たす)
0213132人目の素数さん垢版2018/01/25(木) 22:42:00.83ID:ikFKaeuq
>>211
a(t) はkごとに異なるから、シグマを外してn倍するところが
おかしいよ
0214132人目の素数さん垢版2018/01/26(金) 00:30:48.09ID:MpPHqpFL
この漸化式で表される数列は、一般項を求められますか
a_(n+3)=2a_(n)+5
a_1=1,a_2=2,a_3=3
0215132人目の素数さん垢版2018/01/26(金) 01:04:51.71ID:sc1f2f+a
られます。
項番が3で割って0,1,2の系列ごとに
nの式で書けば良いだけ。うまくやればきれいに
まとめられるかもしれない。
0216132人目の素数さん垢版2018/01/26(金) 01:07:13.06ID:043/+7HS
とりあえず1つの式にできそうな気はする
3次方程式だからなんとかなりそう
トラップあったらしらん
虚数入ってても文句言わないこと
0217132人目の素数さん垢版2018/01/26(金) 01:18:38.89ID:UTYSLzDJ
aとrを固定
nをaで割った余りがrのときf(n)=1、それ以外のときf(n)=0となるような
nの初等関数f(n)は存在するか?
0218132人目の素数さん垢版2018/01/26(金) 01:20:50.83ID:SFTUqpd2
>>217
初等関数とは?
0219132人目の素数さん垢版2018/01/26(金) 01:26:34.42ID:sc1f2f+a
初等関数(しょとうかんすう、英: Elementary function)とは、
実数または複素数の1変数関数で、代数関数、指数関数、対数関数、
三角関数、逆三角関数および、それらの合成関数を作ることを
有限回繰り返して得られる関数のことである。
初等関数 - Wikipedia

これでは?周期性持たせるのに三角関数いじるんだろうけど
すぐには思いつかんなあ
0220132人目の素数さん垢版2018/01/26(金) 01:48:14.34ID:sc1f2f+a
>>217
rだけ平行移動するのはすぐできるからr=0として考えて良い
aが素数の時は f(n)=sin(pi*(n^(a-1) -1)/a)/(-sin(1/a)) が
条件を満たす
aが相異なる素数の積のときはそれぞれ掛けるだけ
aが素数べきで割れるときは…保留
0221132人目の素数さん垢版2018/01/26(金) 02:31:51.13ID:dGgp4j0R
>>215
j =(n mod 3)とすると

a_n = a_(3m+j)=(5+a_j)・2^m - 5, (j=0,1,2)

本問では
 a_0 = -1,a_1 = 1,a_2 = 2,a_3 = 3,

>>216

生成関数
 Σ[n=0,∞]a_n z^n =(-1 +2z +z^2 +3z^3)/{(1-z)(1-2z^3)}
0222132人目の素数さん垢版2018/01/26(金) 02:44:41.03ID:dGgp4j0R
>>217

sin とか exp とかの超越関数は実は分かってないし…
ラグランジュの補間多項式で…

f(n)= Π_(0≦k<a,k≠r)(a{n/a}- k)/(r-k),
{x}= x -[x]
0223199垢版2018/01/26(金) 03:45:18.36ID:dGgp4j0R
>>196 >>203

2π/3 = 120°

π/2 = 90°

π/6 = 30°

とします。 正弦定理じゃなかったし...orz
0226132人目の素数さん垢版2018/01/26(金) 13:28:49.54ID:u5owDkso
みなさん、灘中の算数解いたことありますか?
小学生があんな問題、解ける必要あるんですかね?
大学入試に出てもおかしくないような問題が山盛りですよ。
0228132人目の素数さん垢版2018/01/26(金) 14:22:39.43ID:mVneyCBt
おら、小学校の時は解けたが、もう解けない。悲しい
0231132人目の素数さん垢版2018/01/26(金) 19:41:42.85ID:XSVQ9V/s
>239
後からどうにでも実力伸ばせるのだから、小学生が大学入試レベルの問題を解かなきゃ行けない出題ってむしろ有害な感じがするんですよね。数学スレの皆さんだって優秀なのに、小学生の時に大学入試レベルの問題なんて解けなかったでしょう?
0232132人目の素数さん垢版2018/01/26(金) 19:52:30.20ID:9Efg2B9V
灘中に行く必要はないし解ける必要はないよ
行きたい人が解ければいい
0234132人目の素数さん垢版2018/01/26(金) 20:59:05.02ID:WWlQq7Zx
>>231
問題書かないと何にもならない
0235132人目の素数さん垢版2018/01/26(金) 22:28:22.12ID:1rHuUO/S
停止時刻の問題で分からないのがあるんだけど詳しい人いますか?
0237132人目の素数さん垢版2018/01/26(金) 23:08:19.98ID:dO6YTpKd
外から調子に乗るクソガキはいい加減に誹謗中傷を繰り返すのをやめろ。
文句があったら面と向かって言えと言っているだろ。

しつこいんだよ。

何も解決しない、言ったら言いっぱなし、意味不明な言葉を
ずっと年中その嫌がらせを受けて暮らさなければならないとは
なんていう国というか社会だと思うよ。

こいつらの目的が分からない。
0238132人目の素数さん垢版2018/01/26(金) 23:55:34.08ID:0yXze6P2
>>235
とりあえず書いてみたら?誰かわかるかもしれない
伊藤の公式とかSDEとか懐かしいなあ
0239132人目の素数さん垢版2018/01/26(金) 23:57:00.29ID:0yXze6P2
>>233
こちとら数学と無縁のアラサーやぞ。
暇なときに回答して遊んでる
0243132人目の素数さん垢版2018/01/27(土) 02:56:33.89ID:cbONqMLa
T(ω)!=T(ω’) だと変だよね、と証明すれば良い。
0244132人目の素数さん垢版2018/01/27(土) 03:09:05.45ID:0MhzVKsq
誰か今日の12時までにお願いします
複素数平面の垂直の問題です

2点 z1=3+4i z2=-1-5i を考える
次の問題に答えよ

(1) 線分z1z2の垂直2等分線L(直線)をガウス平面に図示し、L上の任意のz3=a+biをすべて求めなさい ただし、aとbは実数である
(z4を中点と考えて解く問題だそうです)

(2) 底辺をz1z2とすると三角形z1z2z3が、線分z1z3となる線分z2z3が等しく高さが√13となる二等辺三角形になるようにz3をもとめなさい (1)を利用します
0245132人目の素数さん垢版2018/01/27(土) 05:31:58.87ID:D06Jx1kP
>>244
複素数平面の要素殆どない
p+qiを点(p,q)に直してみろ、ただの座標の問題になるだろ
解答の最後で出てきた点(x,y)をx+yiに直すだけ、直線の式も同様
0246132人目の素数さん垢版2018/01/27(土) 05:51:00.36ID:gbbm6MqT
>>245
いやいや…複素平面で出題してるんだからそういうやり方したらきっと減点食らうんやろなあ
0247132人目の素数さん垢版2018/01/27(土) 06:09:13.57ID:0MhzVKsq
>>245
以前そのやり方で解いたところ複素数平面の考えで解いてくださいと書かれていたので今回、回答をお願いしました

わかる方お願いします
0250132人目の素数さん垢版2018/01/27(土) 06:17:31.45ID:G5V4uSu9
z_3=(z_4)+k((z_2)-(z_1))i

とか
0251132人目の素数さん垢版2018/01/27(土) 06:20:29.59ID:0MhzVKsq
>>249
はい
解いていただきたいです
答えだけでなく手順もお願いします
0252132人目の素数さん垢版2018/01/27(土) 06:23:09.19ID:0MhzVKsq
>>250
ありがとうございます
実際に数値をいれてみるとどういう答えになりますか?
0253132人目の素数さん垢版2018/01/27(土) 06:24:50.41ID:G5V4uSu9
そんくらい自分でやれ
なめとんのか
0254132人目の素数さん垢版2018/01/27(土) 06:30:26.61ID:GcXmOoNg
座標平面の問題としてなら解けるってことのようだからそのやり方で解いて
解答の中のぜんぶの座標を>>245みたいに複素数に直すだけで十分だと思う

複素数の積が座標平面上の一次変換に対応することを利用しない問題なら
複素数平面なんて座標平面上で平面ベクトル考えてるのと何も変わらんわけで
0255132人目の素数さん垢版2018/01/27(土) 06:36:55.23ID:0MhzVKsq
>>254
ありがとうございます
自分で考えて何がなんだかわからなくなってしまっています

出来れば手順も書いて欲しいです
お願いします
0257132人目の素数さん垢版2018/01/27(土) 06:58:04.76ID:0MhzVKsq
>>256
わかりました
考えていただきありがとうございました
0258132人目の素数さん垢版2018/01/27(土) 07:23:32.00ID:0MhzVKsq
皆さんのお陰で(1)は答えまでたどり着けたと思います
(2)のほうは全くわかりません
できる方お願いします
0259132人目の素数さん垢版2018/01/27(土) 07:49:16.54ID:x31JH3ww
>>258
底辺をz1z2とするときの高さが√13→線分z3z4が底辺z1z2の垂線上にあるのでz3z4の長さが高さに等しい→|z3-z4|=√13
辺z1z3と辺z2z3の長さが等しい→z3は線分z1z2の垂直2等分線L上にある→(1)で求めたz3とz4を利用
0260132人目の素数さん垢版2018/01/27(土) 07:55:37.61ID:0MhzVKsq
>>259
ありがとうございます
そのやり方で1度挑戦してみます

もし良ければ計算もしてください
0261132人目の素数さん垢版2018/01/27(土) 08:15:04.09ID:0MhzVKsq
計算してみたところ√13が残って本当に合っているのかが不安です
誰か計算してもらえると嬉しいです
0262132人目の素数さん垢版2018/01/27(土) 11:40:28.89ID:gbbm6MqT
問題を写し間違えてるんじゃなければ√13は残るのが正解なんじゃないかな
0263132人目の素数さん垢版2018/01/27(土) 12:08:23.39ID:tghXOZVX
z2-z1=-4-9i
これをπ/2回転させてt倍したものをz4に足せば垂直二等分線上の点を表す
(1-(1/2)i)+t(-4-9i)i=(1+9t)+(-(1/2)-4t)i

|t(-4-9i)i|=√13⇔|t||9-4i|=√13⇔t=±√(13/97)
z3=(1±9√(13/97))+(-(1/2)±4√(13/97))i (複号同順)
0264132人目の素数さん垢版2018/01/27(土) 12:16:37.55ID:pqZRjVA5
mが整数のとき、ガウス記号について、実数αに対して
[α±m] = [α]±m
は明らかにいえますか?
0265132人目の素数さん垢版2018/01/27(土) 12:36:51.04ID:0MhzVKsq
>>263
ありがとうございます
これは(1)の方ですね?
0266132人目の素数さん垢版2018/01/27(土) 12:39:49.87ID:0MhzVKsq
>>263
(2)もありますね
勘違いしてごめんなさい
0269132人目の素数さん垢版2018/01/27(土) 13:16:41.25ID:GuICzJgB
複号は逆順やね
すまんな
0270132人目の素数さん垢版2018/01/27(土) 14:23:14.31ID:pqZRjVA5
えっ

[α±m] = [α]干m

ということですか?
0271132人目の素数さん垢版2018/01/27(土) 14:46:30.89ID:X12ydps4
ポーカーの役が出る確率の問題なのですが・・・
http://www.kumamotokokufu-h.ed.jp/kokufu/math/game_p.html
こちらのサイトだと、単純に山札から5枚ランダムに選んだ場合の確率を示しています。
しかし、最もポピュラーなテキサスホールデムポーカーだと、ランダムに選ばれた5枚と自分の手札2枚から、一番強くなるように5枚のカードを選んで役を作ります。
前者と後者のルールで、役が揃う確率は違うのでしょう?
0272132人目の素数さん垢版2018/01/27(土) 14:54:21.88ID:MA43eYRM
ぜーんぜん違おう
0273132人目の素数さん垢版2018/01/27(土) 14:55:07.67ID:Vqh38PFk
>>269

>>263
0274132人目の素数さん垢版2018/01/27(土) 19:11:56.28ID:D06Jx1kP
複素数平面上の2点A(α)、B(β)に対し、原点Oから下ろした垂線の足をH(γ)とする。
点P(αβ)とHの距離が1以下かつ、OとHの距離が1以下であるとき、αとβが満たすべき条件を求めよ。
0275132人目の素数さん垢版2018/01/28(日) 01:40:45.00ID:ru4HDAPy
>>274

A(α)= a’+ia”
B(β)= b’+ib”
H(γ)= h’+ih”
とおくと
P(αβ)=(a’b’-a”b”)+i(a’b”+a”b’),
AB^2 =(b’-a’)^2 +(b”-a”)^2,
また
γ ={(a’b”-a”b’)/AB^2}(α-β)i
よって
h’={(a’b”-a”b’)/AB^2}(b”-a”)
h”={(a’b”-a”b’)/AB^2}(a’-b’)
0276132人目の素数さん垢版2018/01/28(日) 07:56:17.34ID:8UL7hOGH
子供の算数の問題がありました。なんか納得いきません。

四捨五入して百の位までの数にしたとき、1600になる整数のはんいは、
□□□□から□□□□までです。

答え 1550から1649

ええんか?
0277132人目の素数さん垢版2018/01/28(日) 09:01:55.38ID:tvdqz0JU
>>32
ああ、確率だから記号は0から1の間ってことを見落としていました…
整理できないはずです(-ω-;)


ん?確率記号と固有値記号見間違えてました
これって固有値が絶対値1以下って証明する必要ありますよね?
この部分が迷子なんですけど、B^-1ABの1,2成分が=0という意味であってますよね?
固有値の差の式が出てこないです…

またB^(-1)*A*B=((λ,0),(0,μ))
の(1,2)-成分よりu*(λ-μ)=b
0281132人目の素数さん垢版2018/01/28(日) 10:30:36.55ID:eCAAOzs4
「無」に勝るものはありますか?
0283132人目の素数さん垢版2018/01/28(日) 14:59:54.14ID:MgDMHoIt
>>281
「神」です
0285132人目の素数さん垢版2018/01/28(日) 15:22:01.04ID:Zb2yPUWu
>>283
「神」より「無」の方が強いです。
0288132人目の素数さん垢版2018/01/28(日) 17:05:11.78ID:mJwzA7SX
(2000+1/2)^n+(1999+1/2)^n ガ整数になる正整数nをすべてもとめよ
0289132人目の素数さん垢版2018/01/28(日) 17:27:09.79ID:mJwzA7SX
>>284

-1
0290132人目の素数さん垢版2018/01/28(日) 17:30:30.07ID:dGk/uttR
>>285
いいえ、「神」が最強です
0294132人目の素数さん垢版2018/01/28(日) 19:06:00.01ID:qZQ2ZqMA
>>284
通分したときの分子は
ca(c-a)+ab(a-b)+bc(b-c)
acc-aac+aab-abb+bbc-bcc
=(a-b)cc-(aa-bb)c+aab-abb
=(a-b)cc-(a+b)(a-b)c+ab(a-b)
=(a-b)(cc-(a+b)c+ab)
=(a-b)(c-a)(c-b)
=-(a-b)(b-c)(c-a)
よって-1
0296132人目の素数さん垢版2018/01/28(日) 20:31:02.87ID:ZRhv114K
解き方がわからないならいいが、計算すれば出来る問題を教えろとか本物のアホなんだな
0299132人目の素数さん垢版2018/01/28(日) 21:58:55.79ID:FAImHS9N
>>277
おー、久しぶり。細かい論点は無視したから、
必要なら自分で行間を埋めてください。

>A=((a,b),(p,r)),Bは、B^(-1)*A*B=((λ,0),(0,μ))
>なる2次正方行列で、B=((c,d),(e,f)),B^(-1)=((g,h),(i,j))
>u=chとおくと、BとB^(-1)の積の計算からdj=-u
>またB^(-1)*A*B=((λ,0),(0,μ)) の(1,2)-成分より u*(λ-μ)=b

ここは「B*((λ,0),(0,μ))*B^(-1)=A の(1,2)-成分より u*(λ-μ)=b」
と書くべきでした。わるい
0300132人目の素数さん垢版2018/01/28(日) 23:00:14.98ID:1dlaUMtq
>>277
0≦a,b,p,q,r≦1やa+b=p+q+r=1の条件を使うと固有値が1以下なのは証明できる
固有値=1の可能性はあるのでその場合はは別で考える必要がある
か、もしくは矢印が引いてあるところは非0と画面の外で言ってるのかもしれないが
0301132人目の素数さん垢版2018/01/28(日) 23:07:29.75ID:mJwzA7SX
>>288 をおねがいします。
0302132人目の素数さん垢版2018/01/28(日) 23:14:53.63ID:5rZ2InGQ
http://rmc-oden.com/blog/archives/5397

図3で赤と緑の三角形が相似。
売上高と安全余裕額が対応するのは理解できるのですが、営業利益と限界利益が対応するのが理解できません。
わかる方解説願えますでしょうか。
0304132人目の素数さん垢版2018/01/28(日) 23:30:53.63ID:FAImHS9N
>>302
総費用線と変動費線は平行だから、
これら2直線とでかい赤三角の2辺(底辺以外の2辺)
とを考えると、売上高線が、損益分岐点のところで、
(営業利益):(営業利益ー限界利益)
の比に内分されることがわかる
0305132人目の素数さん垢版2018/01/28(日) 23:41:17.44ID:ypZsP/7u
>>304
ありがとうございます!
斜めの三角形で考えるんですね、スッキリしました。
0306132人目の素数さん垢版2018/01/29(月) 00:46:07.41ID:ZektEyMe
>>303

そこまではいくのですが、nはいくらになりますか?
0307132人目の素数さん垢版2018/01/29(月) 02:07:43.54ID:BjS1YFQQ
>>288
(A+B)^n=A^n+C[n,1]A^(n-1)B+C[n,2]A^(n-2)B^2+...+C[n,n-1]AB^(n-1)+B^n
だから、(A+B)^n+(A-B)^n を展開したとき、Bの指数が奇数の項は消える

nが偶数の時
(A+B)^n+(A-B)^n
=2{A^n + C[n,2]A^(n-2)B^2+...+C[n,n-2]A^2B^(n-2)+B^n}
A=2000=2^4*5^3、B=2^(-1)のとき、検討すべき項は、2B^n
これは、2^(1-n) なので、n=0の時のみ、整数

nが奇数の時
(A+B)^n+(A-B)^n
=2{A^n + C[n,2]A^(n-2)B^2+...+C[n,n-3]A^3B^(n-3)+C[n,n-1]AB^(n-1)}
検討すべき項は、2*C[n,n-1]*AB^(n-1)で
これは、 2n*2^4*5^3*2^(1-n)=n*5^3*2^(6-n)
n=1,3,5の時のみ整数

正整数に限るなら、n=1,3,5のときだけ整数になる
0308132人目の素数さん垢版2018/01/29(月) 02:49:17.39ID:hyZJ2mMS
>>288 >>306
 n=1,3,5

2項展開すると
(M+1/2)^n + (M-1/2)^n = 2Σ[k=0,[n/2]]C[n,2k]M^(n-2k)(1/2)^(2k)

n=1  2M^1,
n=2  2M^2 +(1/2),
n=3  2M^3 + (3/2)M,
n=4  2M^4 +(6/2)M^2 +(1/8),
n=5  2M^5 +(10/2)M^3 +(5/8)M,
n=6  2M^6 +(15/2)M^4 +(15/8)M^2 +(1/32),
n=7  2M^7 +(21/2)M^5 +(35/8)M^3 +(7/32)M,


本題では
M = 2000 =(2^4)(5^3)
0309132人目の素数さん垢版2018/01/29(月) 04:22:09.66ID:+S6MB9w8
>>292
「無」というのは無いことだから、破壊できないのです。
そもそも「破壊」というのは「有」の概念なのです。
何かがあって初めて成り立つ概念なのです。
0310132人目の素数さん垢版2018/01/29(月) 05:11:19.88ID:T6by0vl6
>>309
一度しか言わないので確と肝に銘じてください
「無」は脆弱です
「無」に何かひとつでもモノが生じれば「無」ではなくなります
つまり「無」は容易に「破壊」されます
「無」はモノを産み出す能力があります。これは観測事実です
したがって「無」は脆弱です
0311132人目の素数さん垢版2018/01/29(月) 05:35:21.04ID:+S6MB9w8
>>310
全然分かってないですね。
「有」が「無」になるのは敗北を意味しますが、
「無」が「有」になるのはOKなのです。
それに、「有」=「無」では「無い」。
「全(=「有」の全て)」=「無」では「無い」。
つまり、「全て」は「無」「無」は「全て」なのです。
「無」ってのは「どうなってもいい」ものなのです。
それぐらい強いのです。
0312132人目の素数さん垢版2018/01/29(月) 05:37:29.51ID:+S6MB9w8
ようするに、「無」が「有」になるのはОKと言ったが、
そもそも「全て」は「無」なのだから、
「無」が「有」になるなんてことはありえない。
0313132人目の素数さん垢版2018/01/29(月) 05:43:19.17ID:+S6MB9w8
それに、今言ってる「無」ってのは、
現象的な、外側から見た場合の「無」だけでなく、
精神的な、内側の本質的な「無」のことも言ってます。
「無」はやはり最強でしょう。
「有」は制約があるが、「無」はそもそも何も無いわけだから、当然それが無い。
0314132人目の素数さん垢版2018/01/29(月) 05:54:47.31ID:+S6MB9w8
いや、「有」が「無」になるのは敗北を意味するって言ったが、それはちょっと違うな。
なんか自分でも、何が言いたいのかよく分からなくなってきた。
0317132人目の素数さん垢版2018/01/29(月) 13:24:47.67ID:BZ/BRTjp
大事なことは1度だけ…まあ2度くらいまで言えば必要十分です
何度も何度も同じ投稿をするから飽きられる
0318132人目の素数さん垢版2018/01/29(月) 15:30:42.07ID:bEB/hmj6
f(x)=(sin)^3をx=0におけるテイラー展開をn次まで求め、一般項がよくわかるよう、例えばsinxのx=0におけるテイラー展開であれば
sinx=x-x^3/6+x^5/120+…+{x^(2n+1)*(-1)^n}/(2n+1)!+(剰余項)
のように答えよ
0319132人目の素数さん垢版2018/01/29(月) 17:17:56.82ID:tcjrlyZs
www.wolframalpha.com使え
ココで聞くより間違いなく速くて正確
sin^3(x)なのか、sin(x^3)なのかははっきりさせてくれとおもったけど
どっちでも割と初歩的な問題だな。

taylor series sin^3(x)
0320132人目の素数さん垢版2018/01/29(月) 17:43:08.87ID:FARWhayU
「肯定する証拠がない」から「ゆえに否定される」は導けません

「否定する証拠がない」から「ゆえに肯定される」は導けません

論理構造
0321132人目の素数さん垢版2018/01/29(月) 17:46:52.49ID:MO3RU3e7
証拠が”ない”ことを主張するなら、まず証拠という疑念を定義しないと駄目
0322132人目の素数さん垢版2018/01/29(月) 22:19:39.35ID:ZektEyMe
>>307−308

ありがとうございます。
助かります。
0323132人目の素数さん垢版2018/01/30(火) 03:06:47.54ID:Z5rBZZAJ
>>288 >>301 >>306 >>322

(M+1/2)^n +(M-1/2)^n = a_n,
とおくと
 a_0 = 2,
 a_1 = 2M,
漸化式は
 a_{n+1}= 2M・a_n -(MM - 1/4)a_{n-1},

M=2000 のとき分母の2の指数がどうなるか…
0324132人目の素数さん垢版2018/01/30(火) 03:28:47.03ID:Z5rBZZAJ
>>318

マクローリン展開
sin(x)= Σ[k=0,∞]{(-1)^k /(2k+1)!}x^(2k+1)
を使えば

{sin(x)}^3 ={3sin(x)-sin(3x)}/4
=(3/4)Σ[k=0,∞]{(-1)^k /(2k+1)!}x^(2k+1)-(1/4)Σ[k=0,∞]{(-1)^k /(2k+1)!}(3x)^(2k+1)
=(3/4)Σ[k=0,∞]{(-1)^k /(2k+1)!}x^(2k+1)-(3/4)Σ[k=0,∞]{(-9)^k /(2k+1)!}x^(2k+1)
=(3/4)Σ[k=0,∞]{(-1)^k(1 - 9^k)/(2k+1)!}x^(2k+1)

*) 奇関数なので、偶数次の係数は 0
0325132人目の素数さん垢版2018/01/30(火) 06:30:17.29ID:xcx/0UYn
ヘンペルのカラス
0326132人目の素数さん垢版2018/01/30(火) 09:13:35.22ID:87c+4/dj
相異なる正の整数a,b,cがこの順に等比数列をなすとき、次の各問いに答えよ。
1.bが素数のとき、(a-1)(c-1)の値を求めよ
2.a,cが互いに素のとき、(a-1)(c-1)は8の倍数であることを示せ。
0327132人目の素数さん垢版2018/01/30(火) 09:42:02.85ID:tDXdTeaU
>>326
1. {a,b,c}={1,p,p^2} (a-1)(c-1)=0
2. {a,b,c}={1,r,r^2} (a-1)(c-1)=0
0328132人目の素数さん垢版2018/01/30(火) 09:48:47.26ID:tDXdTeaU
>>327
>2. {a,b,c}={1,r,r^2} (a-1)(c-1)=0
{a,b,c}={m^2,mn,n^2} (a-1)(c-1)=(m^2-1)(n^2-1)
{m,n}={2k,2k+1} (m^2-1)(n^2-1)=(4k^2-1)4k(k+1)
2|k(k+1) 8|4k(k+1)
0329132人目の素数さん垢版2018/01/30(火) 09:50:08.92ID:tDXdTeaU
>>328
>{m,n}={2k,2k+1}
m or n=2k+1
0330132人目の素数さん垢版2018/01/30(火) 12:14:30.39ID:bVow/qUC
陰関数 (x^3+x^2 y-y^2)^10=0 の微分
0331132人目の素数さん垢版2018/01/30(火) 12:29:46.60ID:udnegqvU
微分と導関数をわざとごちゃまぜにするのって最近の流行りなの?
0333132人目の素数さん垢版2018/01/30(火) 14:09:14.51ID:oYVJlAGI
おら、よく区別わからないんだけど、微分は動詞、導関数は名刺でOK?
0336132人目の素数さん垢版2018/01/30(火) 16:57:04.49ID:nqAQN3qR
この 2 問の解き方をどなたか教えてください
解法は中学数学の範囲でお願いします
https://i.imgur.com/V1rEJaM.jpg
0338132人目の素数さん垢版2018/01/30(火) 19:45:23.00ID:pt37UQM4
ああああああああああああああああああああああああああああああああああああああああああああああああああああああああああああああああああああああああああああ
0339132人目の素数さん垢版2018/01/30(火) 19:59:52.87ID:/cBV7J36
>>336
(ア)小さい方の2つの形を直線のとこで背中合わせにくっつけて大きいほうの形の窪みに納める
(イ)AOとBCに補助線を引く
0341132人目の素数さん垢版2018/01/30(火) 20:36:11.96ID:YJGOa4vQ
無でも有でもない『虚』の可能性を除外して議論している
ので誤謬になります

by論理学
0342132人目の素数さん垢版2018/01/30(火) 20:36:22.76ID:tDXdTeaU
>>330
d(x^3+x^2y-y^2)^10=10(x^3+x^2y-y^2)^9d(x^3+x^2y-y^2)=10(x^3+x^2y-y^2)^9(3x^2dx+2xydx+x^2dy-2ydy)=0
>>330
微分とはdf(x,y)すなわちf_x(x,y)dx+f_y(x,y)dyのこと
0343132人目の素数さん垢版2018/01/30(火) 20:37:12.35ID:tDXdTeaU
>>333
NG
どちらも名詞
0344132人目の素数さん垢版2018/01/30(火) 21:26:48.95ID:YJGOa4vQ
「物理的な今が何であるのか」
どうして分からないのでしょう、実に不愉快です

意識(あるいは純粋理性)とは
物理的にはどのような事態なのでしょう
これも分からない

人間は偶然発生的な理屈(力学とか数学とか宇宙論)によって
世界を構成しているのだなぁ、と
そのことに人間自身が気づくことが可能であり
(哲学とはそもそもそのようなものだから)

その能力のありようを「限りなく普遍的な問題」として
設定できる(すなわち超越論的)
がしかし、残念ながらこの気付きを「厳密の学」として
信頼できる根拠がない
0345132人目の素数さん垢版2018/01/30(火) 21:49:53.52ID:nqAQN3qR
>>339
ありがとうございます。
高校受験レベルだと普通それは証明不要で大丈夫なんでしょうか?
0348132人目の素数さん垢版2018/01/31(水) 03:43:41.30ID:LQM3BDJg
円Cの内部に、Cの中心とは異なる定点Pをとる。
Pを通り互いに直交する二本の直線をl,mとすると、lとmによりCの内部は4つの領域に分割される。
lの位置が色々変わるとき、以下が成り立つようなlの位置はいくつあるか。

「4つの領域をD1,D2,D3,D4と名付けるとき、DiとDjの面積が等しくなるような(i,j)の組(i≠j)が少なくとも1組存在する」
0350132人目の素数さん垢版2018/01/31(水) 07:51:23.46ID:6zFDJIsp
「微分する」は動詞だぞ
0351132人目の素数さん垢版2018/01/31(水) 09:09:09.67ID:ybzssN0Z
時空の哲学と計算複雑性理論ってどっちの方が難易度高い?
0356132人目の素数さん垢版2018/01/31(水) 15:49:36.43ID:LQM3BDJg
この積分が何日考えても計算できません
工学系の学部二回生です
どうやったらいいんでしょうか、図書館の本にもなく困っています、よろしくお願いします
∫[1~∞] exp(-x)ln(x) dx
0358132人目の素数さん垢版2018/01/31(水) 16:17:14.29ID:LQM3BDJg
区間[a,b]におけるf(x)の最小値はf(a)で最大値はf(b)である。
この区間でf(x)はf(a)からf(b)までのすべての実数値をとり、かつx1<x2⇒f(x1)<f(x2)が成りたつ。
このときf(x)は区間[a,b]において連続であると言えるか。連続であるならば証明し、そうでなければ反例を挙げよ。
0360132人目の素数さん垢版2018/01/31(水) 16:33:24.23ID:LQM3BDJg
358は俺の考えた新しい連続の定義だ
旧来の連続の定義と等価である、どうだ
0361132人目の素数さん垢版2018/01/31(水) 17:11:08.59ID:7vHguFE6
>工学系の学部二回生です
>工学系の学部二回生です
>工学系の学部二回生です
どうも糞もない(笑)
0362132人目の素数さん垢版2018/01/31(水) 17:12:41.71ID:A6yP4Whm
>>360
単調増加でない連続関数については説明できてないですね
0363132人目の素数さん垢版2018/01/31(水) 17:25:01.23ID:LQM3BDJg
p1,p2,...,pnは平方数でない自然数で、どの2つも相異なる。
α=Σ[k=1~n](√pk)とおく。
f(x)を整数係数の整式とするとき、αが方程式f(x)=0の解となるためには、f(x)の次数が少なくともいくつ以上であることが必要か。
0364132人目の素数さん垢版2018/01/31(水) 17:27:45.99ID:g2m5dpXv
>>362
そして新しくもない
0366132人目の素数さん垢版2018/01/31(水) 17:44:58.32ID:PV8Xa2H+
「言葉は慣習である」などという命題は、
生活環境がその命題を可能とできるほどに安定している、
そのことを言っているに過ぎないのであって、慣習そのものは
記号が言葉となるための仕組みとは一切関係ない
言葉の習得(あるいは成立)には生活環境での反復を要する
その現状報告を述べているだけのことである
しかしそうすると、言葉とはいったい何なのか
0367132人目の素数さん垢版2018/01/31(水) 18:09:36.12ID:64/FZFfL
あああああ
0368132人目の素数さん垢版2018/01/31(水) 18:14:19.03ID:64/FZFfL
全宇宙を微分するとどうなりますか?
0369132人目の素数さん垢版2018/01/31(水) 18:16:01.18ID:A6yP4Whm
神界になります
0370132人目の素数さん垢版2018/01/31(水) 18:45:38.66ID:bGodstjA
神界の広さはどれくらいですか?
0371132人目の素数さん垢版2018/01/31(水) 19:11:11.62ID:y5iWMmMu
>>356
x=e^tだと?
0372132人目の素数さん垢版2018/01/31(水) 19:15:17.36ID:y5iWMmMu
>>365
どうでもいいけどその定義で何が証明できるか
旧来のに帰着させないでうまいのある?
0373132人目の素数さん垢版2018/01/31(水) 19:28:34.43ID:+CmVJce/
神界の広さは広義神界です
0374132人目の素数さん垢版2018/01/31(水) 19:31:38.18ID:A6yP4Whm
中間値の定理は簡単に証明できそうですね
0375132人目の素数さん垢版2018/01/31(水) 19:33:05.76ID:PIjNH4VJ
広義神界とはどれくらいの広さのことでしょうか?
0377132人目の素数さん垢版2018/01/31(水) 19:59:21.41ID:PV8Xa2H+
■存在

日本語では「○○がある」と「□□である」とでは、
表現も違うし内容にも違いがあります
日本語で「存在」というとき、
それは普通、「○○がある」に相当するように思います

しかし哲学辞典で「存在」を引きますと、
それはドイツ語で「Sein」となっており
これは英語だと「Being」です
ですから西洋では、“存在”は“があること”でもあり、
また“であること”でもあると

なるほど、いろいろ面倒くさい
0379132人目の素数さん垢版2018/01/31(水) 20:25:55.29ID:A6yP4Whm
>>377
がある、の「ある」は存在を表す本動詞であり、である、の「ある」は断定を表す助詞「だ」に補助動詞「ある」をつけたものです
すなわち、本質は「ある」と「だ」の違いにあると言えます
がある、の「が」は対格の助詞ですから、「がある」は他動詞的であり、「である」は自動詞的な役割を果たすわけです
前者は存在を表しますが、後者は通常は「は」によってマークされる主題に対する何らかの評価を与えていると考えられます
英語で言えば、existとbeの違いと言えるでしょう
0380132人目の素数さん垢版2018/01/31(水) 20:58:06.38ID:p21aLfIw
英語のbeは主語と状態を結びつける動詞であって、状態が場所なら「〜がある」状態が名詞なら「〜である」になる
日本語だって動詞は「ある」であって意味の違いは助詞でつけている
動詞だけ取り出して云々するのはナンセンス
0381132人目の素数さん垢版2018/01/31(水) 21:01:55.86ID:A6yP4Whm
細かいことですけど、である、の主体はやはり「だ」なんです
これはリンゴだ、という文が許容されるわけですから

まあ動詞だけで議論するのはナンセンスなのは確かでしょうね
0384132人目の素数さん垢版2018/01/31(水) 23:28:37.28ID:KF1wSIJc
a pen is there
0385132人目の素数さん垢版2018/02/01(木) 00:01:13.70ID:y3lGqruL
>>363
ちょうど、[Q(√p1,...,√pn):Q]
場合分けがめんどくさそう…
2,8,...,2^(2n-1) だったら =2だしね
0386132人目の素数さん垢版2018/02/01(木) 00:19:59.88ID:YjwFQWoR
テレビで糞芸人が必至に私を馬鹿にするのは
私の研究分野の人間が完全敗北して悔しいから
そのベクトルが発生するのでしょうか?

よろしく、回答をお願いいたします。
0387132人目の素数さん垢版2018/02/01(木) 00:37:43.21ID:9NvJxwVa
>>356

部分積分で
∫e^(-x)ln(x)dx = -e^(-x)ln(x)+ ∫ e^(-x)/x dx
 = -e^(-x)ln(x)+ Ei(-x),

1<x<∞ で積分すると -Ei(-1)= 0.2193839344
0388132人目の素数さん垢版2018/02/01(木) 00:56:57.36ID:9NvJxwVa
>>358

a<c<b とする。
0 < ε < min{f(c)- f(a),f(b)- f(c)}なる任意の正数εに対して、
 f(c-δ1)= f(c)-ε,
 f(c+δ2)= f(c)+ε,
となる δ1 >0 と δ2 >0 が存在する。
δ= min{δ1,δ2}とおけば
 |x-c| < δ ⇒ |f(x)- f(c)| < ε
よってf(x)はcで連続。
0389132人目の素数さん垢版2018/02/01(木) 01:02:24.17ID:IwtHqysQ
>>342
ありがとうございます。
0390132人目の素数さん垢版2018/02/01(木) 01:05:35.87ID:9NvJxwVa
>>358

・c-δ<x<c のとき
 c-δ1 < x < c,
 f(c)- ε < f(x)< f(c),
・x=c のときは f(x)= f(c),
・c<x<x+δ のとき
 c < x < c+δ2,
 f(c)< f(x)< f(c)+ ε,

いずれの場合も
 |f(x)- f(x)|< ε
0391132人目の素数さん垢版2018/02/01(木) 11:15:29.90ID:rgI+cAka
A君とB君はくじ引きゲームをすることにしました。
A君は1/10で大当たりするくじを、B君は1/20で大当たりするくじを引きます。
この時、どちらかが当たりを引くまで同時にくじを引くとして、B君がA君より先に大当たりを引く確率はいくつですか?
ただし、A君とB君が同時に大当たりをひいた場合はB君が先に大当たりを引いたものとしてカウントします。
はずれくじを引いた場合は、くじを戻してもう一度それぞれ1/10、1/20で大当たりするくじを引くことにします。
0394132人目の素数さん垢版2018/02/01(木) 14:36:10.60ID:ozAVSfgN
>>393
求める確率をxとする
1回目でBが先に当たりを引く確率(B1)は1/20
1回目で決着がつかない確率(D1)が9/10×19/20=171/200
2回目以降も1回目と同じ条件なので、1回目で決着がつかない場合に、Bが先に当たりを引く事後確率はxに等しい
xは1回で結果がわかる場合と2回以上かかる場合の合計に等しいのでx=B1+D1x
xで解いてx=B1/(1−D1)
0395132人目の素数さん垢版2018/02/01(木) 15:24:28.82ID:VENBaWyO
https://i.imgur.com/eZTVlI8.jpg
1-sin^3θがなんで偶関数でもないのに偶関数の性質を積分範囲に適応してるのかがわかりません。
積分範囲を-π/2からπ/2までのまま計算したら答えも合いませんし。
お手上げですどうか教えて下さい。
0396132人目の素数さん垢版2018/02/01(木) 15:26:12.69ID:CdkDeLqt
眠い
0399132人目の素数さん垢版2018/02/01(木) 15:41:12.17ID:1ckznX22
この問題なら対称性に注意して最初から 0 から π/2 で計算する(そして2倍する)のがふつうじゃないかね
0405132人目の素数さん垢版2018/02/01(木) 16:46:54.68ID:xonqEH2T
すみません。5列×5列ビンゴで数字は1から75まで、真ん中穴の普通のビンゴで、数字が24個発表されたときに3列ビンゴになる確率が知りたいです
0406132人目の素数さん垢版2018/02/01(木) 16:55:33.17ID:2M7atQPm
質問です
直角三角形の斜辺が2016でその他の辺が252とxの場合にxの値はどうやって出せばよいですか?
力技で756√7は一応出せたのですけれども絶対出題意図的にはもっとスマートにやってほしいと思われるのですが
0407132人目の素数さん垢版2018/02/01(木) 17:20:38.28ID:rOWhXTs4
>>406
2016と252の最大公約数pを求める
3辺をpで割っても三平方の定理は成り立つので、求めるxをt=x/pとし、2016/p=c、252/p=bとおく。
b^2+t^2=c^2
c^2-b^2=t^2
(c+b)(c-b)=t^2
求めたtをp倍すればxが得られる

これ以上何かあるか…
0409132人目の素数さん垢版2018/02/01(木) 18:11:16.47ID:2M7atQPm
すいませんもうひとつお願いします

斜辺AB=8の直角三角形ABCに半径1の円が内接している
三角形の面積を求めよ

AB=a, BC=b, CA=c と置いて高さ1の三角形3つと考えるのかなとは思うのですが
0410132人目の素数さん垢版2018/02/01(木) 18:40:03.98ID:CdkDeLqt
345
0413132人目の素数さん垢版2018/02/01(木) 19:18:29.23ID:MLuWeH6P
>>409
その考え方でおk

b^2+c^2=64 より
(b+c)^2=2bc+64 … @
bc/2 = 8/2 + b/2 + c/2 より
bc=(b+c)+8 … A

これを解いて b+c=10, bc=18
△ABC=bc/2=9

ちなみに3辺は 5−√7, 5+√7, 8 となる
0414132人目の素数さん垢版2018/02/01(木) 19:41:04.86ID:YjwFQWoR
何故お前ら糞ガキは外から「小林にも挨拶しないで。」
どうのこうの下らないヤジを俺に聞かせて何がしたいわけ。

うるさい以外の何物でもないわけだが。
文句があるんだったら、面と向かって言ってみろ。

残念でした。とかつまらないことしか言えない無能だったら
黙ってろ。

てめーらみたいなアホの声を聞いてもなんの意味もない。

男の風上にもおけないカスは黙ってろ。
0418132人目の素数さん垢版2018/02/01(木) 21:55:27.83ID:C0yreNuI
計算の指針(特にフビニの定理の使いどころにたどり着くまで)がわかりません。助けてください

フビニの定理を(※)式に用いて、以下の値Iを求めよ

1.I=∫(0、∞)exp(-x^2)dx
(※)∫(0、∞)・dy・∫(0、∞)x・exp(-x^2・(1+y^2))・dx

2.I=∫(0、∞)(sin(x)/(x・exp(x)))
(※)∬(0、∞)×(0、∞)exp(-x・(y+1))・sin(x)・dx・dy
0419132人目の素数さん垢版2018/02/01(木) 22:01:03.29ID:ozAVSfgN
>>412
内接円の中心から斜辺の両端にむけて補助線を2本引く
できた底辺x高さ1の小さい直角三角形と、底辺1高さ8-xの小さい直角三角形とを、それぞれの斜辺の両端でひっくり返すとあら不思議
8×1の長方形と、一辺1の正方形に早変わりしましたとさ
0421132人目の素数さん垢版2018/02/01(木) 22:21:00.46ID:Uh1pL2rv
>>418
1.I=∫(0、∞)exp(-x^2)dx
(※)∫(0、∞)・dy・∫(0、∞)x・exp(-x^2・(1+y^2))・dx
∫(0、∞)x・exp(-x^2・(1+y^2))・dx
=∫(0、∞)(1/2)* exp(-u・(1+y^2))・du (u=x^2, du/dx=2x)
=(1/2)*(1/(1+y^2))*[-exp^(-u)]_u=0 -> ∞
=1/(2*(1+y^2))
よって
(※)∫(0、∞)・dy・∫(0、∞)x・exp(-x^2・(1+y^2))・dx
=(1/2)*∫(0、∞)1/(1+y^2)dy
=(1/2)*[Arctan(y)]_y=0 -> ∞
=(1/2)*(pi/2) (pi=π、念のため)
フビニの定理(の一つ)は、積分の順序の交換を許すものだから
(定理の十分条件が成り立っていることはちゃんと確認すること)、
(※)=∫(0、∞)・dx・∫(0、∞)x・exp(-x^2・(1+y^2))・dy
でもある、これを計算すると、
∫(0、∞)x・exp(-x^2・(1+y^2))・dy
=(x*exp(-x^2)) * ∫(0、∞)exp(-(x^2)*(y^2))・dy
=(x*exp(-x^2))*∫(0、∞)(1/x)*exp(-v^2)・dv (v=x*y, dv/dy =x)
=exp(-x^2)*∫(0、∞)exp(-v^2)・dv
=exp(-x^2)* I (積分変数に何の文字を使っても同じ)
だから、
(※)=∫(0、∞)・dx・exp(-x^2)・I
 = I^2
よって I^2 = pi/4 ∴I=(√pi)/2

こんな感じで 2. も考えてみてください
0423132人目の素数さん垢版2018/02/01(木) 22:38:55.11ID:Uh1pL2rv
2.は、とりあえず(※)をyで(1重)積分してみてよ
そっちのほうがちょろそうでしょ?
まあ計算が最後まで済んだらそれにこしたことはないけど
0424132人目の素数さん垢版2018/02/01(木) 22:43:49.34ID:CKvgO9ry
√2の少数部分をaとするときax+y/1-a=aとなるような有理数x,yの値を求めよ。
これを教えてください
0425132人目の素数さん垢版2018/02/01(木) 22:53:37.52ID:Uh1pL2rv
>ax+y/1-a=a
↑どこまでが分母でどこまでが分子かわかんないよ
あと小数な 以下(ax+y)/(1-a)=aのことだと解釈する
a=-1+√2
(-1+√2)*x+y=(1-a)*a=(-2-√2)*(-1+√2)
=2-2-√2=√2
∴ y-x=(1+x)*√2
∴ y-x=1+x=0 ∴x=-1,y=-1
0426132人目の素数さん垢版2018/02/01(木) 23:02:19.72ID:Uh1pL2rv
まちがえた
(-1+√2)*x+y=(1-a)*a=(2-√2)*(-1+√2)
=-2-2+3*√2=-4+3*√2
∴ y-x+4=(-x+3)*√2
∴ y-x+4=-x+3=0 ∴x=3,y=-1
0427132人目の素数さん垢版2018/02/01(木) 23:05:14.31ID:C0yreNuI
>>420>>421
ありがとうございますー
2.を解く前に1.の解答をきっちり清書してやっていることを理解できるようやってみます
0428132人目の素数さん垢版2018/02/01(木) 23:09:57.82ID:CKvgO9ry
>>426
説明不足ですいません。
ありがとうございます
0429132人目の素数さん垢版2018/02/02(金) 03:21:24.43ID:VtO0MSKR
>>412
その考え方でおk

(1+x)^2 + (9-x)^2 = 8^2 より

xx -8x +9 = 0,

x = 4±√7,

僊BC = AC・BC/2 =(25-7)/2 = 9,

ちなみに{AC,BC}= 5±√7 となる。
0434132人目の素数さん垢版2018/02/02(金) 10:05:32.86ID:+w0FHFWV
differential (n., adj.)
differentiation (n.)
differentiate (v.)
導関数:derivative, derived function, differential

integral (n., adj.)
integration (n.)
integrate (v.)
原始関数:integral
0435132人目の素数さん垢版2018/02/02(金) 10:15:39.91ID:K8QwpSpk
√2の少数部分をaとするときax+y/1-a=aとなるような有理数x,yの値を求めよ。
これを教えてください
0436132人目の素数さん垢版2018/02/02(金) 12:31:21.99ID:U+VMG2jU
ax-y=2a
y=(x-2)a

a 無理数ー>(x−2)a 無理数

よって (x,y)は存在しない。
0438132人目の素数さん垢版2018/02/02(金) 14:26:36.07ID:bpO5KMeW
>>433
でもベクトル場X,Yだと▽_xYは共変微分だから微分じゃん
df/dx=df(d/dx)に相当するものなのに
df/dxもベクトル場d/dxに沿った微分ってことでいいんじゃね
0440132人目の素数さん垢版2018/02/02(金) 17:44:10.18ID:8tY4gdrs
f=x^3+4x^2+2x+1に対して、x^10(mod f)の求め方を教えていただけないでしょうか
0441132人目の素数さん垢版2018/02/02(金) 18:31:54.28ID:r6mC+IGT
>>440
x^10 =
(x^7 - 4 x^6 + 14 x^5 - 49 x^4 + 172 x^3 - 604 x^2 + 2121 x - 7448)
*(x^3+4x^2+2x+1)+(26154 x^2 + 12775 x + 7448)
ほんとに問題合ってる??
0442132人目の素数さん垢版2018/02/02(金) 18:43:38.58ID:r6mC+IGT
>>439
(3)の解答で FI' = 4てなってるけど3じゃない?
ほんとに解答合ってる?
0444132人目の素数さん垢版2018/02/02(金) 18:58:46.21ID:iEzezT/M
>>442
あれほんとだそうですね
実は答えしか手元になくてこの数字だとしたらこうかなと私が書いたのですがそれも違ったようです

お手数ですが誰か解いていただけないでしょうか?
0445132人目の素数さん垢版2018/02/02(金) 20:37:55.12ID:r6mC+IGT
>>443
乙、この調子でがんばれ
一か所惜しいところは、フビニの定理の前提条件を
確認するところで |sin(x)|<=|x| としてるけど、
x=0付近でもほんとにそうか?
(修正のヒント:√x とかを考える)
0446132人目の素数さん垢版2018/02/02(金) 20:45:58.41ID:r6mC+IGT
>>444
丸投げよくない
まず、あなたの出した√5は正しくない。なぜなら、
断面は辺AEを横切らないはずだから。
図をちゃんと描いて(線対称な六角形になる)、4通りの
長さ比べをすればよい
0447132人目の素数さん垢版2018/02/02(金) 20:48:49.81ID:r6mC+IGT
>>444
ごめん>>446はまちがいだった。線対称な六角形じゃない
0450132人目の素数さん垢版2018/02/02(金) 22:14:32.38ID:VAmbcUJT
おらには無理
0451132人目の素数さん垢版2018/02/02(金) 22:16:31.14ID:r6mC+IGT
さすがに教科書見ろといいたくなるが
マジレスすると(高校生?中学生?)、
y+xy'=(xy)'=0
∴xy=C(定数) ∴y=C/x

定石としては「変数分離形」の手法を使う
y'/y=-1/xより、両辺の原始関数を求めて
log(y)=-log(x)+C ∴xy=C~ あとはいっしょ
0454132人目の素数さん垢版2018/02/02(金) 22:35:44.40ID:r6mC+IGT
>>452 じゃ訂正よろ
0455132人目の素数さん垢版2018/02/02(金) 22:36:38.37ID:r6mC+IGT
>>453 …俺は気づいたゾ
0456132人目の素数さん垢版2018/02/02(金) 22:43:06.48ID:17AZMh5k
>>451
ありがとうございました。
y'/y=-1/x, ∫1/y*dy/dx=∫-1/x, ∫1/y dy=∫-1/x dx, log(y)=-log(x)+C,
log(y)+log(x)=C, log(xy)=C, xy=e^C=定数、y=定数/x. ですね。
0457132人目の素数さん垢版2018/02/02(金) 22:46:50.60ID:r6mC+IGT
>>456
大学生(理系全員・経済学部生等も)なら100%習う分野だと
思うので、あなたの正体が気になる
趣味で微分方程式勉強しようとしてる人?
0458132人目の素数さん垢版2018/02/02(金) 22:52:30.24ID:17AZMh5k
>>457
お察しの通りです。
文系出身なので微分方程式は習ったことがないです。
大学一年の数学では微分方程式はやりませんでした。
二年以降はそもそも数学をとっていませんでしたね。
良い教科書があれば教えて下さい。
0459132人目の素数さん垢版2018/02/02(金) 23:08:21.80ID:r6mC+IGT
>>458
書籍が星の数ほどあるので、これという具体名は挙げられないけど、
ちょっと大きめの書店に行って、数学の棚(「解析学」あるいは
「微分方程式」のところ)を覗いてみてください。
「解法・テクニック」に重きをおいたものと(殆どがこっち)、
理論の厳密さ(たとえば、どういう条件の下で解が一意に存在するか、
等の問題)に重きをおいたものの、大きく2種類あります。
数学科の人は普通後者を読んでるので、問題を「解く」能力は
工学部あたりの人のほうが(平均値をとると)上かもしれないですね。

ネットで読める入門編としては↓などはいかがでしょう
(この人は物理出身ですね)
http://eman-physics.net/math/differential01.html
0460132人目の素数さん垢版2018/02/02(金) 23:16:54.59ID:VAmbcUJT
経済学部って数3必要なのけ? まじやばい。
0461132人目の素数さん垢版2018/02/02(金) 23:32:07.45ID:qiykJdq/
経済学部だと簡単な数学の授業か説明があるんじゃないの?

数3不要だったり、数学不要で入学できる理系大学はどうしてるんだろ。
0462132人目の素数さん垢版2018/02/02(金) 23:47:51.06ID:r6mC+IGT
>>460
場合にもよるけど(例えばマルクス経済学のみを
徹底的に研究するだとか)、自分の知る限り、経済学部生の
99.9%は、それこそ九九のように数3程度の微分・積分・行列の
知識を使いこなせないといけないように思われる。
といっても、先生もそのことはわかってて、
そのための補習講義だとか補習資料だとかは用意されてると思うし、
数3なんてあくまで道具なんだから(九九がそうであるように)、
あまり物怖じする必要はないと思う。本業の経済への関心を高めておくべき。
0463132人目の素数さん垢版2018/02/03(土) 00:16:34.17ID:1SQv0bX4
>>438
微分はdf
df/dxはdfの値
0464132人目の素数さん垢版2018/02/03(土) 00:25:11.64ID:9ZCv/9LW
>>463
df/dxはベクトル場d/dxに沿ったfの微分
色々省略して慣習的にdf/dxをfの微分といったりもする
0465132人目の素数さん垢版2018/02/03(土) 01:14:23.65ID:y1+58cEe
X,Yは共に自然数のとき
X^2+Y^2=n このnの規則性ってあったりする?
0466132人目の素数さん垢版2018/02/03(土) 01:17:52.33ID:wxVyyiZU
0が自然数に入るなら、
n:(4k+3)-型の素因数の指数が偶数
が必要十分
0467132人目の素数さん垢版2018/02/03(土) 01:23:03.48ID:y1+58cEe
>>466
ありがとう、ただもう少し馬鹿にも分かるように教えてください
0469132人目の素数さん垢版2018/02/03(土) 01:41:23.07ID:wxVyyiZU
素数は、2か、A:4で割って1余る数か、B:3余る数かのいずれか
(4で割って0または2余るのなら偶数だから)
Aグループは、5,13,17,29,など
Bグループは、3,7,11,19,など
nを素因数分解して、Bグループの素数が一個もでてこないか、
もしくはでてきても指数が偶数だったら、二つの平方数の和で表せる。
しかもその逆も成り立つ。
0470132人目の素数さん垢版2018/02/03(土) 01:55:27.89ID:y1+58cEe
>>468 >>469
なるほど!!!ありがとう!!!助かりました!
0471132人目の素数さん垢版2018/02/03(土) 02:13:02.28ID:wxVyyiZU
>>470
お、おう…そんなに急を要する(深刻な)問題とは思わなかったよ
0472132人目の素数さん垢版2018/02/03(土) 02:30:55.05ID:QGLDsXik
>>461
数3受けずに数学科入ってきたやつおるけど、必修科目で0点取ってたよ。

理系と経済系なら数学3は絶対に勉強すべきだろうね。
0473132人目の素数さん垢版2018/02/03(土) 03:47:01.11ID:xvl288yy
>>288 >>306

>>323 より

(M+1/2)^n +(M-1/2)^n - 2 M^n = b_n,
とおくと
 b_0 = b_1 = 0,
漸化式は
 b_{n+1}= 2M・b_n -(MM - 1/4)b_{n-1}+(1/2)M^(n-1),

小数部は
n=1, 0
n=2, 1/2
n=3, 0
n=4, 1/8
n=5, 0
n=6, 1/32
n=7, 1/2
n=8, 1/128
n=9, 5/8
n=10, 1/512
n=11, -1/32
n=12, 1537/2048
n=13, 89/128
n=14, 3073/8192
n=15, 339/512
0474132人目の素数さん垢版2018/02/03(土) 09:00:53.37ID:y4OwCID0
私より頭がいい人がこの世に存在することはおかしいと思いませんか?
0475132人目の素数さん垢版2018/02/03(土) 09:24:31.39ID:qR5+L9Q3
nを自然数、kを1≤k≤n-1である自然数とする。
nとn+iが2以上の共通の約数を持つような自然数i(ただし1≤i≤k)はどのような数かを述べよ。
0476132人目の素数さん垢版2018/02/03(土) 12:14:50.94ID:xZGSghvc
https://i.imgur.com/CYvQKQh.jpg

この問一なんですが、答えは45度みたいなんですけど何でそうなるんですか?
0478132人目の素数さん垢版2018/02/03(土) 13:32:38.22ID:AK5x2W0M
>>477
頭悪くて申し訳ないのですが、なんでPBDではなくPBCの直角二等辺三角形を証明するのですか?
0481132人目の素数さん垢版2018/02/03(土) 16:02:40.17ID:60v3ONEF
>>480やはりPBDの方ですよね

BDPが90度になるのは、何故でしょうか?
0482132人目の素数さん垢版2018/02/03(土) 16:14:12.59ID:SRNC+iev
>>481
やっぱりそこで引っ掛かってるのね…
辺BDが面ACDの垂線だから、というのが理由だけど、もっと説明いる?
0483132人目の素数さん垢版2018/02/03(土) 16:16:27.58ID:C5w5yM46
空間に直線lを中心軸とし底面の半径が1の円柱Cがある。ただし円柱Cは直線lの方向に十分長いものとする。
この空間内で一辺の長さが2√2である正四面体Dを動かすとき、CとDの共通部分の体積Vの最大値を求めよ。
0484132人目の素数さん垢版2018/02/03(土) 16:28:58.75ID:PI5ZigKi
普通に「じゅーぶんおおきー」が答えじゃないの?
0485132人目の素数さん垢版2018/02/03(土) 16:33:08.16ID:C5w5yM46
xy平面上の曲線C:y=sinx上の、0≤x≤πの範囲を動く点Pがある。点Pのx座標をpとおく。

(1)PにおけるCの接線lpが、Pとは異なる点QでCと交わる。またそのような点Qはただ1つであるという。このようなpの範囲を求めよ。

(2)(1)のとき、Cおよびlpによって囲まれる面積の最大値を求めよ。
0486132人目の素数さん垢版2018/02/03(土) 16:37:12.27ID:C5w5yM46
(1)n,kを自然数とする。2^n-1=3^kを満たすn,kが無数に存在するかどうか、理由をつけて答えよ。
(2)m,n,kを自然数とする。2^m+3^n-1=6^kを満たすm,n,kが無数に存在するかどうか、理由をつけて答えよ。
0487132人目の素数さん垢版2018/02/03(土) 16:40:32.20ID:C5w5yM46
6で割って1余る素数で、n以下のものの個数をpnとする。
また、6で割って5余る素数で、n以下のものの個数をqnとする。
極限lim[n→∞] pn/qn を求めよ。
0488132人目の素数さん垢版2018/02/03(土) 16:54:58.69ID:TSEB3dIY
0<β<α を満たす任意の α, β について
u∈H^α ⇒ {(‖u‖_α)^(β/α)}{(‖u‖)^(1-β/α)}
を示せ。(H^αはハーディ空間)

をどなたかお願いします...
ヘルダーの不等式を使うのはなんとなくわかるんですけど...
0489132人目の素数さん垢版2018/02/03(土) 16:55:00.33ID:C5w5yM46
方程式x=2sinxの正の実数解αの値を小数点以下第一位まで求めよ。第二位以下は切り捨てよ。
0490132人目の素数さん垢版2018/02/03(土) 22:46:23.43ID:8oGhrs9l
>>489
y = x-π/2 として,cos(y)の級数展開を 2次まで使って方程式
2(1-y^2/2) = y + π/2 を解く. y = (1/2)(-1 + √(9-2π)).
x = y+π/2 を評価すれば x = 1.89493. 真の解は x = 1.89549.
0491132人目の素数さん垢版2018/02/04(日) 00:40:04.50ID:FHnZ0MZe
>>482
はい

なんでBDがACDの垂線だと、BDPが90度になるんですか?
0492132人目の素数さん垢版2018/02/04(日) 01:12:47.84ID:5XKbohJi
順列と組合せで質問なんですが
重複を嫌うのはどちらですか?
問題で、例えばじゅず順列は2で割ったり、部屋割りも割ったり
ここで
nCr × r! = nPr
から、nCrをr!していると言うことは、nCrは重複を含まない値であると推測したのです
つまり、順列は重複を許す、つまり区別する
組合せは重複を嫌う、つまり区別しない
ってことですか?
0493132人目の素数さん垢版2018/02/04(日) 01:22:34.79ID:5XKbohJi
あーまじで場合の数が分からない
左右対称とかなんやねん、積分愛してる
0494132人目の素数さん垢版2018/02/04(日) 01:31:07.28ID:+/8ZTz7g
>>492
組合せは順列と比べると順序を問わないだけ
重複を許す「重複順列」「重複組合せ」というのが「順列」「組合せ」とは別にある
つまり単に「順列」「組合せ」と言った場合は通常は重複を許さないほうを指す
0495132人目の素数さん垢版2018/02/04(日) 01:43:39.70ID:ABQbOPIY
>>492
例を挙げる。{a,b,c}の3つから
重複を許さず2回選ぶときの順列は3P2=6通り。具体的にはab,ac,ba,bc,ca,cb
重複を許さず2回選ぶとき組合せは3C2=3通り。具体的にはab,ac,bc (この場合例えばabとbaは同じものと考える)
重複を許して2回選ぶときの順列は3Π2=9通り。具体的にはaa,ab,ac,ba,bb,bc,ca,cb,cc
重複を許して2回選ぶとき組合せは3H2=6通り。具体的にはaa,ab,ac,bb,bc,cc
0496132人目の素数さん垢版2018/02/04(日) 02:41:00.81ID:y+GKnqZU
>>489
sin(X+a)=sin(X)cos(a)+cos(X)sin(a)≒X cos(a)+sin(a)  :Xの一次で近似
図を描いて、解が、3π/5 近辺にあることを利用するため、
X=x-3π/5 ,a=3π/5を方程式に代入すると、
x=2((x-a)cos(a)+sin(a)) → x=2(sin(a)-a*cos(a))/(1-2cos(a))

底辺1、頂角π/5、の二等辺三角形を描き、底角の二等分線を引くと、
相似の三角形ができること等を利用して、等辺の長さは(1+√5)/2と判る
これを利用して、cos(2π/5)=1/(1+√5)、sin(2π/5)=(1/4)√(10+2√5)
a=3π/5=1.885,cos(a)=-0.309,sin(a)=0.951を用いると、
x=1.8955...
0497132人目の素数さん垢版2018/02/04(日) 03:09:32.74ID:ht+YqN8a
>>493
積分愛してるとか言う割に対してできないだろ
せめて偏差値60超えてから言えアホ
この積分計算してみろ

∫[0→1] (3x^2+1)/√(x^2+1) dx
0498132人目の素数さん垢版2018/02/04(日) 03:12:43.31ID:Q7f8+AYB
計算式でsinθやcosθと書くのが面倒なんですが簡単にした書き方って有りますか?
0502132人目の素数さん垢版2018/02/04(日) 08:37:00.13ID:LmZV7Qkt
「中学生からの数学オリンピック」というの本の代数のところを
本日3問ぐらい解きたいと思います。開始すぐ解答見てしまうと
思いますが類題や難易度や参考書などが知りたいです。
知らない記号もあるので教えていただけるとありがたいです。
内容的には中高一貫校の中3以上の問題とのことです。
0503132人目の素数さん垢版2018/02/04(日) 08:54:07.47ID:LmZV7Qkt
質問
3^1024−1 が 2^n で割り切れるような、最大の整数 n を求めよ。
凡人が解くような解き方が知りたいです。
似たような問題を出す大学とかあれば教えていただけるとありがたいです。
0504132人目の素数さん垢版2018/02/04(日) 09:22:17.43ID:CxVck6NH
>>492
0123456789の10個の数字3個使って
重複を許さず並べる総数(順列)
10*9*8=720
(012,013,,,,,,987)
重複を許し並べる総数(重複順列)
10^3=1000
(000,001,,,,,999)
重複を許さず組み合わせる総数(組合せ)
720/3*2*1=120
(012,013,,,,
重複を許し取り出した組み合わせの総数(重複組合せ)
0505132人目の素数さん垢版2018/02/04(日) 09:27:40.92ID:CxVck6NH
(012=021=102=120=201=210,013=031=103=130,,,,,789=798=879=897=978=987)
重複組合せの1つの組を大小順に並べて2つめに1を足し3つめに2を足すと
重複のない組合せの1つの組となる
この対応は全単射(であることを証明できる)
よって
10個から3個取り出す重複組合せの総数は12個から3個取り出す組合せの総数と一致
12*11*10/3*2*1=220
0506132人目の素数さん垢版2018/02/04(日) 09:28:55.48ID:pIH8Vul+
その積分くらいは、暗算で出来るようになりたいと自分では思う。

この間、どこかのスレかツイッタで、これくらいの積分は暗算で出来て当然だろうとか言われてた式を
暗算できなくてちょっと落ち込んでたのでした。
0508132人目の素数さん垢版2018/02/04(日) 09:50:49.51ID:TQv5imF2
いま1a2bの復習してるから、積分の難しい問題後で解いてくるから、そしたらまた出してくれ
0510132人目の素数さん垢版2018/02/04(日) 10:16:57.42ID:LmZV7Qkt
3^1024 の数を求めることって難しいんだな。
0512132人目の素数さん垢版2018/02/04(日) 11:50:19.77ID:XRYfX+lx
今朝新聞に入ってたなんちゃら中学の入試問題です。小学生が解くやつでしかも肩慣らし問題のようですが、解法がわかりません。教えてください。算数の問題ですいません。

1.ある整数nを2回かかてできた数を10で割った余りを<n>と表すことにします。
 たとえば、2x2=4なので<2>=4
       7x7=49, 49/10=4 あまり9 なので<7>=9 です。
 このとき、1から127までの整数で、<n>=4となる整数nは[ ]個あります。
2.ある整数nを2回かけてできた数を15で割った余りを<<n>>と表すことにします。
 このとき17を17回かけた数をmとすると、<<m>>=[ ]です。

 何か法則を見つけ出すのだろうか...ちなみに答えは25と4です。
0513132人目の素数さん垢版2018/02/04(日) 12:01:07.81ID:ABQbOPIY
>>503
3^2≡1(mod 8)なので、任意の正整数mについて3^2m≡1^m≡1(mod 8)つまり、
3^2m+1=8k+2=2(4k+1)となる整数kが存在することから、3^2m+1は2と奇数の積である
一方、3^1024-1を以下のように変形する。
3^1024-1=(3^512+1)(3^512-1)=(3^512+1)(3^256+1)(3^256-1)=...
=(3^512+1)(3^256+1)(3^128+1)(3^64+1)(3^32+1)(3^16+1)(3^8+1)(3^4+1)(3^2+1)(3^2-1)
この式は(3^2m+1)となる9個の整数と3^2-1=2^3の積である
すなわち、3^1024-1は2^12と奇数の積である
最大の整数nは12
>>503が言う凡人が解けるかどうかは知らない
問題を出す大学も知らない
0514132人目の素数さん垢版2018/02/04(日) 12:19:47.12ID:trTH9dx5
合同式や文字式を使わずに厳密に議論するのは面倒くさい(中学高校入試は途中式を要求しないので何使ってもいいが)

□×△を○で割った余りは、□÷○の余りと□÷○の余りをかけた数を○で割った余りに等しい
0515132人目の素数さん垢版2018/02/04(日) 12:22:04.71ID:ABQbOPIY
>>512
法則ってことでいうと、<11>〜<20>や<21>〜<30>が<1>〜<10>と同じ
ということに気づけば<2>=4なら、<12>も<22>も4であることが言える
<1>〜<10>で4を探すだけで<1>〜<127>に4がいくつ含まれるかは計算できる
同様に<<17>>は<<2>>と等しいし、<<2×2×2×2>>=<<16>>は<<1>>と等しい
さらに<<17^17>>が<<2^17>>と等しいことに気づけば、それが<<2>>と等しいことは簡単に求められるだろう
0516132人目の素数さん垢版2018/02/04(日) 12:24:43.05ID:trTH9dx5
>>503
3^1024-1^1024を展開すると…?
0517132人目の素数さん垢版2018/02/04(日) 12:51:34.26ID:trTH9dx5
>>503
3^1024-1^1024
=(3^512+1^512)(3^256+1^256)…(3^2+1^2)(3^1+1^1)(3^1-1^1)
=(2^3)*Σ[k=1,9](3^(2^k)+1)
3^(2n)+1=9^n+1=(8+1)^n+1で二項定理を使えば4で割った余りは2
つまり3の偶数乗に1を足したものは2で高々1回割りきれる
よって全体では2で高々12回割りきれる
0518132人目の素数さん垢版2018/02/04(日) 15:13:00.97ID:3o3EFUDg
503の者です。レスありがとうございます。
こういうの初見で解ける奴いるんだろうけど、問題見せたら
 3^1024 をひたすら計算していた。
中学生でもMOD使いこなす奴るんだろうな・・・
0519132人目の素数さん垢版2018/02/04(日) 18:42:52.03ID:8PweCmGH
自殺をしたら地獄に落ちるというのは本当なのでしょうか?
0521132人目の素数さん垢版2018/02/04(日) 18:52:15.89ID:lVlsxpNH
なんとなくだけどこのスレワッチョイあればいいなあと思った
変なの排除する意味もあるけど、聞いてる相手が同じ人だってわかれば質問もスムーズになるんじゃ?って感じで
0523132人目の素数さん垢版2018/02/04(日) 19:01:24.96ID:ndb3B6XI
Sqrt[3]^100の整数部1の位の数字をもとめよ

おねがいします。
0524132人目の素数さん垢版2018/02/04(日) 19:04:40.67ID:6G7kcm9D
3を50乗
1の位を追跡
3->9->7->1->3->以下ループ
0525132人目の素数さん垢版2018/02/04(日) 19:05:44.03ID:ndb3B6XI
Sqrt[3]^99の整数部1の位の数字をもとめよ

おねがいします
0526132人目の素数さん垢版2018/02/04(日) 19:12:39.75ID:lVlsxpNH
>>523
sqrt[3]って3乗根のことでいいのかな?ちがったらすまん

適当な整数aを持ってきてa^3<100<(a+1)^3(⇔a<sqrt[3]<a+1。左右どっちかの矢印が消えるかも?ごめんわからん)になるようにaを設定
aは特に計算で求める必要はなくて、a=nとすると…って具合に宣言しちゃっておk
今回は整数部分(1の位)を求めるからaがそのまま答え。さらに(a+b/10)^3<100<(a+(b+1)/10)^3とすることで小数第1位以下も出せるよ
0528132人目の素数さん垢版2018/02/04(日) 19:15:34.24ID:ndb3B6XI
sqrt[3]って3乗根のことでいいのかな?ちがったらすまん


すみません!
Sqrt[3]=√3 です。
在りがございます。
0529132人目の素数さん垢版2018/02/04(日) 19:17:55.85ID:ndb3B6XI
>>524

Sqrt[3]^100=9なんですね
Sqrt[3]^99は?
0530132人目の素数さん垢版2018/02/04(日) 19:21:23.78ID:lVlsxpNH
√3^99=3^49×√3
√3=1.732…だったから、3^49に1.732…をかけたらその1の位がどう変化するかを見る
0531132人目の素数さん垢版2018/02/04(日) 19:27:45.61ID:1AYVpyrY
>>529
あのね、おじさん哀しいよ。

>>524 は、
(√3)^100 = 3^50,
3^n の1の位は n を追って見ていくと
(n=1で)3->(n=2で)9->(n=3で)7->(n=4で)1->(n=5で)3->
以下ループすると言っている。そのくらい読み取ろう?

(√3)^99 なら、(√3)^99 =(3^49)√3,
上の方法で 3^49 の1の位を出して、
√3の近似値を掛けてみたらどうかね?
0532132人目の素数さん垢版2018/02/04(日) 19:48:12.97ID:XwEY92Oa
数学再勉強中の理系院生です
ろくすっぽ集合論も良くわかっていない身ながら最近ルベーグ積分について独学中です
そこで質問なのですが開区間と閉区間で測度は変わるのですか?
参考書が無く、ググっても見つけられなかったのでどなたかご教授いただけるでしょうか?
0533132人目の素数さん垢版2018/02/04(日) 19:59:43.48ID:6G7kcm9D
入れる測度による
端点が(1点の測度>0)であれば当然異なる(チャージとか呼んだりする)
ただ、普通のルベーグ測度であれば上記のようなことは起こらない
厳密に言えば、『まず1点が可測であり、任意の正値より測度が小さい
こと』を示せばよい。
『』内は示せますか?もし難しく感じるのであれば、
基礎のところをみっちりやる必要があると思う。
0534132人目の素数さん垢版2018/02/04(日) 20:17:54.48ID:ht+YqN8a
xyz空間の平面z=0上の各格子点を、z軸に平行な直線が貫いている。
一辺の長さが1/√2、1つの頂点がAである正四面体を、以下の規則によりこの空間内に置く。

・正四面体の重心G(x,y,z)について、x,y,zはいずれも実数である。それぞれ独立に区間(-∞,∞)から無作為に1つの値が選ばれる(分布は一様分布とする)。
・Gが選ばれると、この正四面体の外接球となる領域Dはただひとつに定まる。D上の1点Pを無作為に選び、PとAを一致させる(分布は一様分布とする)。

このとき、この正四面体を貫く直線の本数の期待値を求めよ。
0537132人目の素数さん垢版2018/02/04(日) 20:39:56.68ID:ht+YqN8a
次の条件を満たす有理数pを1つ求めよ。
ただしnは99 以下の自然数とする。

・どのnに対しても、npは整数でない。
・f(x)をx以下の最大の整数、g(x)をx以上の最小の整数とする。このとき、min(np-f(np),g(np)-np)を最小とするnがただひとつ存在し、それはn=55である。
0540132人目の素数さん垢版2018/02/04(日) 21:53:32.52ID:v68r//Zt
俺は社会との接点を持つために「中学生からの数学オリンピック」
という本の問題を一緒に解いてるんだけど。(今日から)とりあえず3問終わった。
3^1024 を計算するためにC言語を勉強中・・・。
なんか、それぞれの立場で数学取り組んでるんだろうなと妄想中・・・。

ルベーグ積分って学部の頃に習うのだろうけど、練習問題とかあまり充実して無いんだろうな。
0541132人目の素数さん垢版2018/02/04(日) 22:28:43.91ID:6G7kcm9D
3^1024 って480桁以上あるよ
Cでdouble型使っても17桁までしか入らない
いろいろ工夫するのも含めてCなんだね
0542132人目の素数さん垢版2018/02/04(日) 22:30:58.94ID:6G7kcm9D
>>540
意外と同じような立場の人(理系分野を独学してる人)
いると思うから、経験談をネットに書いたりすると
みんなの勇気づけになるかもしれないですね。がんばってください
0543132人目の素数さん垢版2018/02/04(日) 23:01:24.04ID:PcHJ7I/R
先ほど測度について質問した者です。
レスありがとうございました。
すいません、折角レスしていただいたのに遅くなりました。

>>534
なるほど、測度によって変わるのですね。
今読んでいる教科書(といいますかネットで見つけたPDF)では、一点では測度が0になることを示すくだりがありました。
ということはその範囲であれば測度を見るのについては、開区間か閉区間かは問題ではない、という認識で大丈夫でしょうか?

それから括弧内の話は自信を持って答えられそうに無いので基礎からちゃんとやってみようと思います。
ところで、この場合の基礎は集合論でしょうか?

>>536
ルベーグ積分 pdfとやると真っ先に出てきたこのページにありました九州大学の講義資料と思しきものを読み進めています。
ttp://language-and-engineering.hatenablog.jp/entry/20140510/LebesgueIntegralMeasurePDFNoteLinks

>>539
ご助言ありがとうございます。
貴方の意図していることと同じかはわかりませんが、確かに参考書が必要ということはそもそもの基礎知識なり学力が追い着いていない状態なのかなとは感じていました。

実はそもそもルベーグ積分を勉強しているのも微分幾何を学ぼうと思ってちっともわからないので、その基礎は何かと探してみたからだったりするのです…
学部時代もう少しまじめに勉強するべきでした…
0544132人目の素数さん垢版2018/02/04(日) 23:15:53.35ID:6G7kcm9D
>>543
>その範囲であれば測度を見るのについては、開区間か閉区間かは問題ではない
そういうことです

>それから括弧内の話は自信を持って答えられそうに無いので基礎からちゃんとやってみようと思います。
>ところで、この場合の基礎は集合論でしょうか?
括弧内があやふやだとなると、やはり集合・位相はある程度(徹底的にやろうとするとキリがない)やらないと、先に進んでもちんぷんかんぷんになります(断言)
まあ正直集合論てそれ自体は面白くないので(数学基礎論的に扱う場合は別として)、なんかやってて釈然としないことがあったらこのスレとか大学レベルスレとかに書けば誰か応答してくれるかも。自分もたまに覗いてます。
0545132人目の素数さん垢版2018/02/04(日) 23:22:38.62ID:PcHJ7I/R
>>544
レスありがとうございます。
そうですね、今一度本腰を入れて勉強してみようと思います。
それでまた何かわからないところがあったらここに挙げさせてもらえれば、と思います。

ともかく初めの疑問が自分なりには消化できた気がします。
ありがとうございました。
0546132人目の素数さん垢版2018/02/04(日) 23:31:25.58ID:6G7kcm9D
がんばれ〜 数学科(卒)ならその辺(集合位相)は
嫌というほどやらされてるから、割と回答者多いよ
0547132人目の素数さん垢版2018/02/04(日) 23:39:43.10ID:QxDOhyjo
そうかな? そう奴はいそうでいないとおもうけどな
0549132人目の素数さん垢版2018/02/05(月) 00:39:29.05ID:tjkCYLNc
>>497

x = sinh(t)とおく。

x=1 のとき、t = log(1+√2)= 0.881373587… = t_1, cosh(t_1)= √2

(与式)= ∫[0,t_1]{ 3sinh(t)^2 + 1}dt

=(1/2)∫[0,t_1]{ 3sinh(t)^2 + 3cosh(t)^2 - 1}dt  *

=(1/2)[ 3 sinh(t)cosh(t)- t ](t=0→t_1)

=(1/2)(3√2 - t_1)

= 1.68063355

*)cosh(t)^2 = sinh(t)^2 + 1 を使った。

ついでながら、>>501 だと

∫{3 - 2(cosθ)^2}/(cosθ)^3 dθ

を求めることになり、楽しくない…
0550132人目の素数さん垢版2018/02/05(月) 00:45:09.24ID:A3LvedzO
Sqrt[3]^100の整数1位の9なんですが
Sqrt[3]^99の整数1位は?
0551132人目の素数さん垢版2018/02/05(月) 00:49:24.47ID:A3LvedzO
>>497
3/√2 - ArcSinh(1)/2
0553132人目の素数さん垢版2018/02/05(月) 01:12:28.01ID:A3LvedzO
>>548
99をこえる素数で宰相は101だから
n/101 が求める整数である。

宰相となるのは n=1 のとき 0.0099.。になる。
0554132人目の素数さん垢版2018/02/05(月) 01:18:04.61ID:XFLU/nHT
3^(99/2)くらいだったらWolframAlphaで計算できるぞ
0555132人目の素数さん垢版2018/02/05(月) 01:24:32.92ID:XFLU/nHT
3^(99/2)=414478596444581735990496.6…
0556132人目の素数さん垢版2018/02/05(月) 01:57:12.86ID:5YcoQeMX
>>537 >>548
ある 100以上のNに対し、
55 x ≡ ±1 mod N
を満たす二つの解の内、一方は100未満で、一方は100以上のもの
を使って、p=x/N
具体的には、p=95/201 とかかな?
0557132人目の素数さん垢版2018/02/05(月) 02:09:04.86ID:tjkCYLNc
>>503 >>513

凡人は a=2^3 とおいて二項定理を使います。

3^(2m)=(3^2)^m
=(a+1)^m
= Σ[k=0,m]C[m,k]a^k
≡Σ[k=0,4]C[m,k]a^k  (mod a^5)
= 1 + m・a + m(m-1)/2・a^2 + m(m-1)(m-2)/6・a^3 + m(m-1)(m-2)(m-3)/24・a^4 (mod a^5)
= 1 + a^4 +(m-1)(a^5)/2 +(m-1)(m-2)/6・a^6 +(m-1)(m-2)(m-3)/6・(a^7)/4 (← m=a^3 とおく)
≡ 1 + a^4,  (mod(a^5)/2)
3^(2m)- 1 = a^4 +(a^5)/2・N = 2^12 + 2^14・N,
∴ n=12

>>517
 4で割った余りは2 ⇔ 2でちょうど1回割り切れる
0558132人目の素数さん垢版2018/02/05(月) 02:21:17.56ID:XFLU/nHT
>>557
2で割る回数はこっちの自由だから「高々」
0560132人目の素数さん垢版2018/02/05(月) 03:22:00.83ID:5YcoQeMX
>>503

3^1024-1は、a[1]=2、a[n+1]=(a[n]+1)^2-1 としたときの、a[11]に当たる。

a[k]=p×2^q、ただし、pは奇数 という形であるとき、
a[k+1]=(a[k]+1)^2-1=(p×2^q+1)^2-1=p^2×2^(2q)+p×2^(q+1)=(p^2×2^(q-1)+p)×p^(q+1) だから、
(q=1のとき、(p^2×2^(q-1)+p)=p^2+p で、偶数だが、)
q≧2のとき、(p^2×2^(q-1)+p)は奇数なので、a[k+1]=p'×2^(q+1)という形になる。
つまり、a[k]が2の因数を二つ以上持っているとき、a[k]からa[k+1]へと変わるとき、2の因数の数が一つ増える。
a[2]=(2+1)^2-1=8=1×2^3 だから、a[11]は、九つ2の因数が増え、12個ある
0561132人目の素数さん垢版2018/02/05(月) 04:02:53.32ID:jmSjfyA+
大学への数学2月号の宿題1の解答教えてください
r,kを正の定数として、円x^2+y^2=r^2と曲線y=kx(x-1)(x+1)の共有点の個数をNとおく。
(1) k=2のときNを求めよ
(2) rの値によらずN=2となるようなkの範囲を求めよ。
0563132人目の素数さん垢版2018/02/05(月) 05:05:45.46ID:jmSjfyA+
>>562
締め切り前だったんですか…
失礼しました
0564132人目の素数さん垢版2018/02/05(月) 10:13:35.98ID:ZNqtFiHL
>>561
交点と言わずあえて共有点って書いてるところが渋いね
高校くらいなら人に聞かないと答えられないほど難しい問題とも思えないが…
0565132人目の素数さん垢版2018/02/05(月) 11:34:45.46ID:A3LvedzO
複素数も考えるんですねえ
0566132人目の素数さん垢版2018/02/05(月) 11:40:11.29ID:A3LvedzO
>>554
3^(99/2)くらいだったらWolframAlphaで計算できるぞ

WolframAlphga がたたしいこと証明してください。
0568132人目の素数さん垢版2018/02/05(月) 14:03:00.63ID:4YqCRdjB
WolframAphganてなニャロ?
0569132人目の素数さん垢版2018/02/05(月) 14:47:45.60ID:mvByPoEA
2円の位置関係で
Ιr-r'Ι<d<r+r'
のとき、二点で交わる
この式の意味が上手く掴めません
解説の図には直角三角形が書いてあるのですが、サッパリ
0570132人目の素数さん垢版2018/02/05(月) 14:49:22.91ID:4YqCRdjB
讃歌苦不等式じゃんか
0573132人目の素数さん垢版2018/02/05(月) 14:59:08.09ID:mvByPoEA
>>571
三角形が書いてあったのは二円が交わるのは、そこに三角形が成立するのと同値であり
三角形の一角cosAを求め、それが-1から1で挟んでできた不等式が円の中心間の距離と半径でできる三角形として捕らえられるから
と解釈しました
0574132人目の素数さん垢版2018/02/05(月) 16:26:58.07ID:OYA2sonW
「全(全て)」の大きさはどれくらいですか?
0575132人目の素数さん垢版2018/02/05(月) 17:21:50.61ID:jmSjfyA+
>>564
高2で高校の先生にやってこいって言われたんですでも分からないから聞きました
0576132人目の素数さん垢版2018/02/05(月) 17:29:42.26ID:mvByPoEA
円束でk=-1ってのは移行したら二つの円を等号で結ぶことになりそれが、2円の交点を通る直線を表すのはまだ分かるけど
k≠-1のとき、なぜ2円の交点を通る「円」になるのですか?
0577132人目の素数さん垢版2018/02/05(月) 17:44:27.67ID:nURVM6ec
エスパーした
(1) その謎の図形は2円の交点を通るか?
(2) それは円の方程式か?
のどっちが分からないんだ?
0578132人目の素数さん垢版2018/02/05(月) 18:37:05.67ID:A3LvedzO
君もう書き込まないでね
0579132人目の素数さん垢版2018/02/05(月) 19:22:12.92ID:DtCc90S/
お前がな
0580132人目の素数さん垢版2018/02/05(月) 19:32:31.43ID:mvByPoEA
>>577
二つの円の式を整数倍した等式が円を表すのが謎
kについての恒等式としてみたら定点を通ると分かるけど、なぜ上手いことそれを等式で結ぶと円の式になるのか不思議
0581132人目の素数さん垢版2018/02/05(月) 20:09:21.35ID:OUMCwV94
税込 350円の商品は いくらの商品に消費税が足されたものか教えてください

小学校くらいの計算式もお願いします。
今後計算したいのです

補足
できる計算は
かけ算、割り算、足し算、引き算、小数点 です

XとかYとか カッコ とかわかりません
0582132人目の素数さん垢版2018/02/05(月) 20:20:57.56ID:A3LvedzO
8%の消費税だと、価格は本体価格+本体価格の8%です。
つまり
1.08倍払わなければリマせん

350/1.08=324.074

で324円です。 税込みで324+324x8%=324+25.9=349.9=350円です
がんばれば349円になります。
0583581垢版2018/02/05(月) 20:31:20.88ID:OUMCwV94
>>582
回答ありがとうございます!
でもまだ難しすぎてよくわかりません 涙

〉税込みで324+324x8%=324+25.9=349.9=350円です
ここしかわからないです、、
税込349円だとして 25.9円がなぜわかったのですか?

バカですみません
0584132人目の素数さん垢版2018/02/05(月) 20:39:39.41ID:1aVeAnpR
a(1)=a
a(n+1)= pa(n)+q
で表される無限数列a(n)で、以下の条件をすべて満たすものは存在しないことを示せ。

(1)どの項の値も0以上9以下の整数である
(2)jが0以上9以下の整数であるならば、a(m)=jとなるmが必ず存在する。
0585132人目の素数さん垢版2018/02/05(月) 20:50:39.58ID:VrAypGL/
この板の最初に書いてある
「わからない問題はここに書いてね(さくらスレ)」
のさくらスレの「さくら」ってどういう意味なんですか
0586132人目の素数さん垢版2018/02/05(月) 23:07:42.81ID:A3LvedzO
本体価格x1.08=税こみ価格

本体価格=税込価格/1.08=350/1.08=324.074=324円

324円の商品に8%の消費税(324x0.08=25.9円)

25.9円は法律で25円か26円になります。 どちらかは知りません。
0588132人目の素数さん垢版2018/02/05(月) 23:20:45.34ID:P1IuWNa6
>>583
0590583垢版2018/02/06(火) 00:03:12.80ID:iqXxGxk3
>>586
すごくわかりました!
スラッシュが何のことかわからなかったのですが 割るの記号だったとは
350÷1.08 なのですね
計算する方法がわかってすっきりしました
ありがとうございます
0592132人目の素数さん垢版2018/02/06(火) 00:24:48.38ID:ePxxHYPU
高校数学解ける訳じゃないけど大学数学とかの方が見てて楽しい
デデキント切断今日しった
0593132人目の素数さん垢版2018/02/06(火) 02:12:22.56ID:Yc2MOlIU
>>547
そういう奴は位相で集合とおもうけどな

>>585
 それも「わからない問題」だな。
 ちなみにこのスレは「分かスレ」と呼ぶらしい。
0594132人目の素数さん垢版2018/02/06(火) 02:35:27.55ID:Yc2MOlIU
>>549

=[ 3sinθ/{2(cosθ)^2}+(1/2)log{tan(π/4 - θ/2)} ](θ=0→π/4)

= 3/√2 +(1/2)log{tan(π/8)}

= 3/√2 +(1/2)log(√2 - 1)

= 1.68063355
0595132人目の素数さん垢版2018/02/06(火) 07:29:54.31ID:oXmpUMOw
定積分I=∫ [0→1] (x^2+a)/√(x^2+b^2) dx
が、有理数p,qと自然数m,nを用いて
I=p√m+qln(n)
と表されるために、実数a,bが満たすべき条件を求めよ。
0598132人目の素数さん垢版2018/02/06(火) 09:50:54.66ID:z95IiXzF
すげえ
お前2000年からいたの?
0599132人目の素数さん垢版2018/02/06(火) 09:51:10.79ID:z95IiXzF
化石じゃん
0600132人目の素数さん垢版2018/02/06(火) 10:09:30.71ID:ipLnh9yt
有限集合は可算集合ではない、は正しいですか
0601132人目の素数さん垢版2018/02/06(火) 10:26:56.15ID:G0UYlGhv
有限集合は非可算集合は正しいお
0602132人目の素数さん垢版2018/02/06(火) 11:08:08.33ID:OuJKwF2p
LU分解ってL(下三角、対角成分は自由)とU(上三角かつ対角成分が1)の積として書いて、
元の行列Aの各成分と比較して未知数を求める、で合ってる?
0604132人目の素数さん垢版2018/02/06(火) 11:40:16.73ID:bxUgU7EA
>>596
>>597
CC桜。元々そんなところから来ていたのか。
ありがとうございます。
0609132人目の素数さん垢版2018/02/06(火) 19:21:17.86ID:4Ffm9GHX
>>608
qはpから平面OABにおろした垂線の足
だから、|p|^2 =|q|^2 +|p-q|^2だし、|r-p|^2 =|p-q|^2 +|r-q|^2
0610132人目の素数さん垢版2018/02/07(水) 00:13:17.91ID:47Ua7x0R
直線の角度「傾き」はなぜXの増加量分のYの増加量になるのでしょうか?そう定義しているからなのでしょうか?
0613132人目の素数さん垢版2018/02/07(水) 01:24:57.70ID:TEOVYXmJ
「全然傾いてない状態」と「いちばん傾いてる状態」を定義して、その二つの状態の間にパラメタという梯子をかける
0615132人目の素数さん垢版2018/02/07(水) 13:54:56.55ID:K14GE45G
tangentの定義に沿うから
0618132人目の素数さん垢版2018/02/07(水) 18:18:53.57ID:9DJRV39C
数列a(n)を
a(1)=a
a(n+1)=sa(n)+t
と定める。

(1)a,s,tが実数のとき、以下の命題(P)が成り立つことを示せ。
(P):a(k+1)>a(k)となるkが無数に存在するならば、lim[n→∞] a(n)=∞ である。

(2)a,s,tが複素数のとき、次の命題(Q)は成り立つか。
(Q):|a(k+1)|>|a(k)|となるkが無数に存在するならば、lim[n→∞] |a(n)|=∞ である。
0619132人目の素数さん垢版2018/02/07(水) 18:47:33.52ID:VdBVCcFo
>>618
a=1,s=-1,t=0だったら?
0620132人目の素数さん垢版2018/02/07(水) 19:11:35.54ID:8FfSWI/V
>>609
ありがとうございます!
0622132人目の素数さん垢版2018/02/07(水) 19:50:21.52ID:TEOVYXmJ
初項が a(0) = 1 である数列 a(n) にたいする二項間漸化式(mは自然数の定数)が

a(n) = a(n-1) * ( m^( a(n-1) ) )

と定まってるとき、一般項は綺麗にまとまりますか?
0623132人目の素数さん垢版2018/02/07(水) 20:28:36.66ID:WpqZ2RSO
>>622

Ackermann関数というらしい。(primitive-recursive level-2 function)

・参考書
数セミ増刊「数学の問題」日本評論社
 第1集 No.112 (1977)
 第2集 No.82 および付録-1 (1978)
0624132人目の素数さん垢版2018/02/07(水) 20:57:26.60ID:TEOVYXmJ
>>623
調べてみましたが、Ackermann関数は二変数関数でした
m、nのどちらか一方を一定値にしても得られた結果は
a(n) = a(n-1) * ( m^( a(n-1) ) )
で得られた結果と異なりました
0625132人目の素数さん垢版2018/02/07(水) 21:19:54.37ID:VOUy9rG6
開写像と連続の違いを教えて
特になぜ開写像が連続の定義じゃダメなのかも
0626132人目の素数さん垢版2018/02/07(水) 21:29:42.88ID:IXE90lwy
開集合の像が開集合なのが開写像、開集合の逆像が開集合なのが連続写像

開写像で駄目な理由は微積か位相の本に書いてあるだろ
一変数関数R→R、特に2次関数ですら開集合を開集合に移さないから連続のイメージに合致しないだろ
0628132人目の素数さん垢版2018/02/07(水) 23:03:42.99ID:WpqZ2RSO
>>622

m=2 のとき
Ack(0,n)= n+1,
k≧1 に対して
Ack(k,0)= Ack(k-1,1)
Ack(k,n)= Ack(k-1,Ack(k,n-1))   (n≧1)
から、
Ack(4,0)= Ack(3,1)= 2^4 -3 = 13
Ack(4,n)= Ack(3,Ack(4,n-1))= 2^(2^(…(2^(2^2))…))−3
0630132人目の素数さん垢版2018/02/07(水) 23:36:41.95ID:XKWKU6b0
>>627
見たことのない応用問題がなくなるまで問題を解きまくれば、応用問題と認識できるものは無くなります
0631132人目の素数さん垢版2018/02/08(木) 01:14:19.92ID:bWLJ5iCe
>>628
m = 2 のとき、
a(0) = 1
a(1) = 1*(2^1) = 2^1 = 2
a(2) = 2*(2^2) = 2^3 = 8
a(3) = 8*(2^8) = 2^11 = 2048
a(4) = 2048*(2^2048) = 2^(2059)
となった
ここで指数の肩に着目すると0 , 1 , 3 , 11 , 2059 , ... となってる
この数列n項がどうやら遺伝的有限集合で、これを頂点とするグラフに対してAckermann符号化を施したときに、辞書的にもっとも早くnクリークのグラフを形成できるらしい

肝心のAckermann 関数との関係は2の累乗があること以外には未だに分からないけれど
0632132人目の素数さん垢版2018/02/08(木) 03:34:31.82ID:uObjvkD8
環でab=aかつbは乗法単位元でないものが存在する例を教えて下さい
0633132人目の素数さん垢版2018/02/08(木) 03:38:44.58ID:uObjvkD8
>>632
冪等元以外でお願いします
0634132人目の素数さん垢版2018/02/08(木) 04:06:18.80ID:uObjvkD8
>>632
a≠零因子でお願いします
0635132人目の素数さん垢版2018/02/08(木) 04:16:10.40ID:bvcT7Tdn
以下の条件をすべて満たす関数f(x)を1つ求めよ。
(1)f(|x|)は区間(-∞,∞)で微分可能
(2)f(|x|)は定数関数でない
0636132人目の素数さん垢版2018/02/08(木) 04:20:45.40ID:uObjvkD8
>>635
f(x)=x^2
0637132人目の素数さん垢版2018/02/08(木) 04:20:47.68ID:bvcT7Tdn
xy平面上の原点をO(0,0)とする。
定点A(1,0)と、領域y>0を動く点P(a,b)がある。
△OAPの外心をQ、内心をIとするとき、OQ>OIとなるようなPが存在する領域を図示せよ。
0642132人目の素数さん垢版2018/02/08(木) 12:11:17.09ID:HI72Ukak
192と150の変化率で質問があります。
比率が分かりません。%で知りたいのですが
192:150=100:xだとするとx=78.125
192が100%だとすると150は78.125%
−約22%変化なのでしょうか?

しかし
192:150=x:100で150を基準に考えると
x=128
150を基準に考えると192は+28%
ものすごく初歩的な質問な気がしますが分かりません
よろしくお願いします
0644132人目の素数さん垢版2018/02/08(木) 13:00:02.64ID:fM335lCU
積分範囲の書き方について質問します。
下記問題と解答の赤枠の部分の積分範囲は何と書いてありますか?
https://imgur.com/a/q9ZAw?
0646132人目の素数さん垢版2018/02/08(木) 13:10:34.58ID:DJohvmqw
>>642
たとえば
100の2割増は120だけど
120の2割引は100でなく96
どっちを1にするかで比率が違うのは不思議でもなんでもない
0647644垢版2018/02/08(木) 13:36:27.67ID:fM335lCU
>>645
>画像が荒くて読めません

元々の画像がこの画質なんです。
ポインタを虫眼鏡のマークにし、クリックして拡大
すると、赤枠の右側はtで積分してあり、積分範囲は
t=0からt=voになっているのがかろうじて見えますが、
赤枠の部分はuで積分しているのに、積分範囲がそれら
しい書き方にはなっていないように見えます。
理論的に考えるとこの部分の積分範囲は
どのように書いてあると考えられますか?
0648132人目の素数さん垢版2018/02/08(木) 14:18:09.41ID:DJohvmqw
>>644
おそらく
上がt=t0
下がt=0
uについて積分してるからuの値で積分範囲を示すのが普通だが
tの値で積分範囲を示しているからあえてt=をつけている
0649132人目の素数さん垢版2018/02/08(木) 14:26:49.00ID:jlbeuFah
単に、座標変換して、積分しやすくしてるだけじゃんかよなぁと思うけど。
0650644垢版2018/02/08(木) 15:08:27.68ID:fM335lCU
t=0からt=vo 訂正→t=0からt=to 

∫[t=0)→(t=to)]{πv^2}du のような表記が可能なんですか? 
0651132人目の素数さん垢版2018/02/08(木) 15:17:02.91ID:jlbeuFah
u=u(t) じゃんか。問題なすし
0652132人目の素数さん垢版2018/02/08(木) 15:57:27.95ID:fM335lCU
ありがとうございます。

∫[0→to)]{πv^2}du は駄目だけど∫[(t=0)→(t=to)]{πv^2}du ならば
t=0に対応するuからt=toに対応するuまでの定積分という意味に解釈
できるというわけですね。
どうせtで積分するなら、この段階でいちいちuの具体的な値を求める必
要はないから、このように書いているというわけですね。
0653132人目の素数さん垢版2018/02/08(木) 17:12:12.67ID:/z1RsvOQ
すいませんこれの(3)どなたか教えてください
ちなみに(1)の答えは a=1/4 で(2)は y=-x+3 です(ここまでは間違いないです)

条件から直線lは直線ABと平行なので y=-x+b (b<0) で、
△ABP=20 なので三平方で AB 出してから直線lと直線ABの距離を求めてこれは 5√2/2 ととりあえず求めたのですが
そこからどうすればよいか分からず詰まってしまいました
https://i.imgur.com/0U8lhae.jpg
0654132人目の素数さん垢版2018/02/08(木) 17:42:25.73ID:bvcT7Tdn
>>653
ダラダラ書いたから読みにくいと思う、わからなかったら誰かに聞いて

距離を測るときに、直線AB上の点Cから
lに向かって垂線を下ろしたことにしよう
lの切片を求めるには、lがABからどれだけ下に下がってるかを考えればいいので、それを求める。
つまりCからy軸に平行に線を引き、それとlとの交点をDとして、CDを計算してその分をABの切片から引けばいい

その下ろした垂線とlの交点をPとすると、あなたの計算ではCP=5√2/2になる
ABの傾きが-1ということは、△CPDは直角二等辺三角形なので、PDも5√2/5

ちなみに、lをABから持ち上げてるのではなく下げているのは、問題文にy軸との交点が負って書いてあるから
0657132人目の素数さん垢版2018/02/08(木) 19:22:46.06ID:jlbeuFah
かけて同じ形になることをしめせばいいだけじゃん。
0658132人目の素数さん垢版2018/02/08(木) 21:43:23.95ID:bvcT7Tdn
ある整数係数の3次方程式は、以下の条件を満たす(0も有理数に含める)。
・有理数解を持たない。
・有理数p,qを用いて√p+√qで表される解を持たない。
このとき、この3次方程式は
Σ[i=0,n]√ai (ただし各aiは正の整数であり、各√aiは無理数で、どの2つのaiとajも異なる)
の形の解を持たないことを示せ。
0659132人目の素数さん垢版2018/02/08(木) 22:34:37.87ID:Ul5afKAE
>>656
なんやこれ
過去問?
問5はサービス問題?
0660132人目の素数さん垢版2018/02/08(木) 22:40:48.45ID:CbA+2eQz
>>658
a:=Σ[i=0,n]√ai
与えられた3次方程式は、aのQ上の最小方程式だから
[Q(a):Q]=3 ところが [Q(a):Q]=[Q(a):Q(√ai)]*[Q(√ai):Q]
の右辺は2で割り切れるので矛盾 □
0661132人目の素数さん垢版2018/02/08(木) 22:44:40.60ID:CbA+2eQz
>>658
だから十分条件が過剰な気がする
『ある整数係数の3次方程式は、以下の条件を満たす(0も有理数に含める)。
・有理数解を持たない。
このとき、この3次方程式は
Σ[i=0,n]√ai (ただし各aiは正の整数であり、各√aiは無理数)
の形の解を持たないことを示せ。』
でも正しい
0662132人目の素数さん垢版2018/02/09(金) 01:29:43.02ID:slQVd0T9
【平面において】

2点A,Bから
等距離の点:無限個(ABの垂直二等分線上の点)
等距離の線:無限本(ABの垂直二等分線、ABに平行な直線)

同一直線上にある3点A,B,Cから
等距離の点:0個
等距離の線:無限本(ACに平行な直線)

同一直線上にない3点A,B,Cから
等距離の点:1個(AB,BC,CAのそれぞれの垂直二等分線の交点(必ず1点で交わる)=外心)
等距離の線:3本

4点、5点、…


【空間において】

2点、3点、…


↑の等距離点/線/面の個数って何かに対応してますか?もしくは一般化した理論が存在しますか?
連立1次方程式の解の自由度と関係してそうなんですけど
0663132人目の素数さん垢版2018/02/09(金) 01:34:00.59ID:slQVd0T9
もともとは平成19年の近大数コンの「四面体の各頂点から等距離の直線は存在するか」という問題なんですけど
0665132人目の素数さん垢版2018/02/09(金) 02:29:12.39ID:hF/Mtwh+
>>658

〔アーベルの第一公式〕
pは素数とする。
もしp次の既約方程式
 x^p + a_1 x^(p-1)+ a_2 x^(p-2)+ …… + a_p = 0,
が代数的に解けるなら、その根は
 x = - a_1/p + (R_1)^(1/p) + (R_2)^(1/p) + …… + (R_{p-1})^(1/p),
という形をもつ。
ここで R_1, R_2, ……, R_{p-1} はある (p-1) 次方程式の根であり、
その方程式の係数は、与えられた係数(a_1, a_2, ……, a_p)を用いて有理的に組み立てられる。

瀬正仁「原典味読」 数セミ, Vol.37, No.10, p.78-79 (1998/Oct)

C.J.Malmsten: Crelle's Journal, Vol.34, No.1, p.46-74 (1847)
「代数方程式の解法の研究」の定理IX
0666132人目の素数さん垢版2018/02/09(金) 02:54:36.71ID:K0NTDvdv
立方体ABCD-EFGHの底面の正方形EFGHにおいて、対角線EGの中点をMとする。
Mを中心としEGを直径とする半円で、長方形AEGCの周および内部にあるものをKとする。
Kの周上(EGは含まない)に点Eと点Gを合わせた領域を点Pが動くとき、以下の問に答えよ。

(1)折れ線FPHの長さをL(P)とおく。L(P)はPの位置に関係ない定数関数であるか、あるいは定数関数でなく最大値と最小値が存在するか。前者であればその定数の値を求め、後者であればその最大値と最小値を求めよ。
(2)折れ線EPFの場合はどうか。
0668132人目の素数さん垢版2018/02/09(金) 05:12:19.54ID:NHlgywcO
>>667
頭に鈴をつける
とってもよく鳴ります

只の駄洒落と侮るなかれ
古来より学力増進や疾病治癒などの願掛けとして鈴をつける行為は広くなされており
絵馬などに鈴がついているのもその名残という
0669132人目の素数さん垢版2018/02/09(金) 14:35:27.95ID:JU2Ajph9
>>662
n次元ユークリッド空間R^nに埋め込まれたm次元ユークリッド空間R^mは
x∈R^nに対する条件としては Ax=b で表わせる
ただし A:(n-m)×n行列, b∈R^(n-m)
Aはn次元行ベクトルをn-m列並べた物と見なせるが、これをn-m個の正規直交系としても同じ条件を表わせる
さらに、その正規直交系にn-m次元内の回転を作用させても同じ条件である
従ってAの意味のある自由度は
n-m個のn次元正規直交系では (n-1)+(n-2)+…+(m)=(n-m)(n+m-1)/2
そこからn-m次元回転(n-m次元直交行列) (n-m-1)+(n-m-2)+…+(1)=(n-m)(n-m-1)/2
を差し引いた (n-m)(n+m-1)/2-(n-m)(n-m-1)/2=m(n-m)
である
これにbの自由度(m次元方向の変化は無意味)=n-m を加えた (m+1)(n-m)
が埋め込みの自由度である
例を挙げれば
2次元平面内の0次元の点の自由度は (0+1)(2-0)=2
2次元平面内の1次元直線の自由度は (1+1)(2-1)=2
3次元空間内の1次元直線の自由度は (1+1)(3-1)=4
3次元空間内の2次元平面の自由度は (2+1)(3-2)=3
となる
これに一般位置のk個の点から等距離という条件をつけると
自由度は k-1 減るから
2次元平面内で2点(k=2)から等距離の点や線は自由度2-1=1(連続無限個)
2次元平面内で3点(k=3)から等距離の点や線は自由度2-2=0(連続無限ではない)
3次元空間内で4点(k=4)から等距離の線は自由度4-3=1(連続無限個)
0670132人目の素数さん垢版2018/02/09(金) 17:48:01.99ID:Vl1wDcPm
問題っていうか質問
実際に使う微分積分は公式とか使っても求められないって聞いたような気がするんですよ
じゃあ公式必至に覚える理由って何?
いま微分方程式勉強してて公式の導き方わかった、問題も解けた
でもこの方程式がなんなの?どこのどういうところに使われてんの?
実際使うことないって
どういうことですか?数学が工学にどう繋がるんでしょうか
0672132人目の素数さん垢版2018/02/09(金) 18:02:42.48ID:Vl1wDcPm
>>671
等加速度運動する物体のT秒での速度しか今のところ使ったことありません
受験終わったら忘れていいんですか??
今大学生なんだけどわからない
0674132人目の素数さん垢版2018/02/09(金) 18:20:01.81ID:a7Kw+UdX
受験テクまで形骸化した求積問題はまず現実のモデルとの対応は無いよ。
こんなのが実務上実社会実世界上必要とされてるスキルだと勘違いしちゃうと後々本人にもトラップみたいに働く可能性がかなり高い。
0675132人目の素数さん垢版2018/02/09(金) 18:36:47.86ID:G1XoOza0
微分積分は全部数値計算するっていうのでもない限り、単純な計算くらいはできた方がいいんじゃないですか?
それに理由を求めるのはナンセンスかと思います
0676132人目の素数さん垢版2018/02/09(金) 18:49:06.41ID:a7Kw+UdX
教科書レベルから無駄に乖離しまくった受験テクを擁護する奴は直接脳味噌を献体にどうぞ
0677132人目の素数さん垢版2018/02/09(金) 18:54:39.73ID:G1XoOza0
大学で習う微分方程式が受験テクなんですか?
変数分離もできない理系とか価値あるんでしょうかね
0679132人目の素数さん垢版2018/02/09(金) 18:58:37.40ID:GOMw9elA
簡単なケースをいくつか知っておくだけのことを大げさに非難する人もいるんだよねえ
なんらかの選民思想に染まってるんじゃないかと思う
0680132人目の素数さん垢版2018/02/09(金) 19:31:09.15ID:F0Wk7Jwm
楕円積分くらいできないとな
0681132人目の素数さん垢版2018/02/09(金) 20:48:27.28ID:SkNBq/Ee
質問です。よろしくお願いします。

https://i.imgur.com/MIP117Z.png

Nという数をふたつの部分に分割したとき、
片方を二乗して3倍したものと、もう片方を二乗して2倍したものの和が
極小になるような分割の仕方を考えよ

…という問題だと思うんですが、考え方がわかりません
ちなみに答えは0.4N, 0.6Nとなっています

原文はここから読めます
http://www.gutenberg.org/ebooks/33283
0683132人目の素数さん垢版2018/02/09(金) 22:16:08.00ID:Cuj5H5/O
微積はMathematicaにお任せくんですが何か?
0686132人目の素数さん垢版2018/02/09(金) 23:00:20.91ID:3vo490QE
3a^2+2(N-a)^2=5a^2-4Na+2N^2=5(a-(2/5)N)^2+(6/5)N^2
minimum is (6/5)N^2 when a=(2/5)N=0.4N
0689132人目の素数さん垢版2018/02/10(土) 00:22:11.65ID:jX1+vITx
中学生が頑張って作ってみた数学の問題なのですが、これは高校の知識だけでも解けるものですか?
「円周率が3.06より大きいことを証明しなさい。ただし2-√2の平方根の値は0.765とし、0.765<2-√2の平方根である。」
0690132人目の素数さん垢版2018/02/10(土) 00:33:12.33ID:ZEcHRU/G
大きい中学生ですか?
0691132人目の素数さん垢版2018/02/10(土) 00:33:20.11ID:O/4sWDpp
中学の問題じゃないの?
0692132人目の素数さん垢版2018/02/10(土) 00:46:33.08ID:jX1+vITx
>>691
ギリギリ中学生でも解けますけど、少なくとも中学でこんな問題はしないです…。
一応、学校で三平方の定理を習ったのでそれを応用させまくってこの問題を作ったのですが、実際これがどれくらいのレベルの問題なのか知りたくてレスしました。
0693132人目の素数さん垢版2018/02/10(土) 00:56:58.46ID:3Sd6zN8Y
円周率は3.14159...である
ところで3.14>3.06である
よって円周率は3.06より大きい
0694132人目の素数さん垢版2018/02/10(土) 01:01:55.76ID:O/4sWDpp
単にハ角形にするだけじゃん。ちょっとできるくんなら簡単に計算できる
0695132人目の素数さん垢版2018/02/10(土) 01:04:10.27ID:JLwIa9Z8
>>689

半径1の円に内接する正n角形の半周をL_nとおく。
L_n = n sin(π/n)< π
n=8 のとき
π > 8 sin(22.5゚)= 4 √(2-√2)= 3.061467
でござるな。
0696132人目の素数さん垢版2018/02/10(土) 01:14:20.75ID:JLwIa9Z8
>>689

n=12
加法公式で
sin(15゚)= sin(45゚-30゚)=(√3 -1)/(2√2)= 0.258819…
π > 12 sin(15゚)= 3.10582854

厨房には無理かな…
0697132人目の素数さん垢版2018/02/10(土) 01:20:44.35ID:O/4sWDpp
加法定理なんか必要ないよ。高校入試レベル
0698132人目の素数さん垢版2018/02/10(土) 01:35:52.84ID:3Sd6zN8Y
「円周率が3.06より大きいことを証明しなさい。ただし円周率の値は3とする。」
0700132人目の素数さん垢版2018/02/10(土) 02:51:49.92ID:M3cIeudq
円に内接する正八角形の一辺の二乗を求める問題は、早稲田実業高の入試で(ノーヒントで)出ている
そこまで難しいわけではない

AB=BC=1, ∠ABC=45°の△ABCにおいて
AからBCへ下ろした垂線の足をDとすると
AD=BD=1/√2, DC=(√2-1)/√2
CA^2=(1/2)+((3-2√2)/2)=2-√2

2π>8CA>8*0.765=6.12
∴π>3.06
0701132人目の素数さん垢版2018/02/10(土) 03:27:50.62ID:M3cIeudq
>>666
立方体の1辺の長さをaとする。
(1) 明らかにFP+PH=FE+EH=2a
(2) 右手系でM(0,0,0), E(-a/√2,0,0), F(0,0,a/√2), G(a/√2,0,0)として、∠GMP=θ (0≦θ≦π)とおくと
P((a/√2)cosθ,(a/√2)sinθ,0)
EP=√(((a/√2)(1+cosθ))^2+((a/√2)sinθ)^2)=a√(1+cosθ)
明らかにPF=EF=a
EP+PF=(1+√(1+cosθ))a
min(EP+PF)=EE+EF=a
Max(EP+PF)=EG+GF=(1+√2)a

a=1やa=√2として最後に拡大縮小してもよい
「明らかに〜」は座標で計算してもでる
0702132人目の素数さん垢版2018/02/10(土) 05:29:25.44ID:SG8Nnt8l
>>689
問題の出しかたによってはいけるんじゃない?
「円に内接する正八角形の周の長さより、もとの円の円周の方が長いことを利用して、円周率が3.06より大きいことを証明しなさい。ここで0.765<√(2-√2)の関係を使って良い」
とかね
0703132人目の素数さん垢版2018/02/10(土) 09:17:10.10ID:+87dBTix
連休中オリンピックの問題を3問ほど取り組んでみます。
いつもは公式を定着させるため反復的な問題ばかり取り組んで
(取り組ませて)いますが、こういう難しい問題を取り組む
(取り組ませると)新たな疑問が出てきて楽しいですね。
まさか昔放り投げたC言語やり始めるとは思わなんだ。
まあ、一緒に考えているだけなのですが。
0705132人目の素数さん垢版2018/02/10(土) 12:42:05.27ID:SOKs+4/e
高校のとき、常微分方程式の簡単なのは解けた、しかし証明問題は全くできず、理系大進学を諦めた。数学の得意な方を尊敬します。(法学部へ進学した)
0706132人目の素数さん垢版2018/02/10(土) 13:27:07.18ID:gfcVweLl
ガキじゃあるまいし、テレビで「死ね。」とか言っている人間の声が聞こえてきたが

馬鹿じゃねーの。お前が死ね。
0707132人目の素数さん垢版2018/02/10(土) 13:33:20.66ID:mOvYbXUO
(2-(0.765)^2)^2>2
くらいは手計算で
0708132人目の素数さん垢版2018/02/10(土) 13:46:19.69ID:O/4sWDpp
今時の若者は開平法も知らんのかね?
0709132人目の素数さん垢版2018/02/10(土) 14:02:27.36ID:eO8qQVZO
開平法(カイ・ピンファ、生年不詳-没年西暦214年)は、後漢の数学者。
「九章算術」の註釈書の断片が残存することで、その名を知られる。
弟子への書簡に、算盤による平方根の計算手順を書き残している。
0711132人目の素数さん垢版2018/02/10(土) 14:34:38.06ID:nX4M3NB+
>>672
理解してないモノなんぞ使えんし
必要なモノを勉強する邪魔にもなるから
さっさと忘れるのが良い
0712132人目の素数さん垢版2018/02/10(土) 15:01:32.04ID:VcDtRhPJ
>>708
開立も出来て然るべきよな
0713132人目の素数さん垢版2018/02/10(土) 15:24:43.97ID:XQsyCNtl
2進表記で1と0を交互にn桁並べた 101010…
この値が 「2^(n+1) を3で割った商」になることは言えますか。
0715132人目の素数さん垢版2018/02/10(土) 17:02:17.74ID:UAeKFFhu
↓の画像の式に具体的な数値をあてはめて赤丸の分散値を求めることは可能でしょうか?
http://www.42ch.net/UploaderSmall/source/1518216947.jpg

具体的な数値は、例えばサイコロの目の期待値など簡単なものが望ましいですが、そうでなくてもOKです。

この式は英Wikipedia
「Expected value」のDefine,Extremal propertyにある式です。
https://en.wikipedia.org/wiki/Expected_value

よろしくお願いいたします。
0716715垢版2018/02/10(土) 17:06:25.58ID:UAeKFFhu
すみません。可能である場合はその例を示していただきたいです。
よろしくお願いいたします。
0719715垢版2018/02/10(土) 17:14:03.44ID:UAeKFFhu
>>717
返信ありがとうございます。
つまり>>717さまにとっては>>715の式を実例で応用することが不可能という理解でよろしいでしょうか?
それとも説明が足りないという意味でしょうか?
0720132人目の素数さん垢版2018/02/10(土) 17:21:19.99ID:dTOy0zth
>>715
Xの期待値とX-Cの期待値はわかるけど分散がわからない、という状況じゃないと使えないですね
パッと例は思い浮かばないですし、普通はX-Cの期待値を、Xの期待値と分散を用いて求める公式、というように考えておけばどうでしょう
0721132人目の素数さん垢版2018/02/10(土) 17:34:27.84ID:O/4sWDpp
最近の若人はこんなんばかりか。
0722715垢版2018/02/10(土) 17:35:44.02ID:UAeKFFhu
>>718,720
返信ありがとうございます。
サイコロの例で考えた場合、V[X]は単純にサイコロの分散値
X={x1,x2,...,X6}={1,2,...,6}
V[X]={V[x1],V[x2],...,V[x6]}

V[x1]=(1/3.5)^2=6.25
V[x2]=(1/3.5)^2=4
  ・
  ・
  ・
V[x6]=(6/3.5)^2=36

という理解でよいでしょうか?(数式の表現が正しいかどうかは不明ですが)
0723132人目の素数さん垢版2018/02/10(土) 17:39:43.96ID:W+kMTk9S
>>722
期待値や分散は、確率変数に対して定義されていて、具体的な実現値に対して定義されるわけではありません
言い換えれば、データの集まり全体の期待値は出すことはできますが、個別のデータの期待値とかいう概念はありません

V[X]はおっけーですけど、V[x1]というものはありません
もう一度、基本から見直して見てください
基本的なことから色々混乱しているようにみえます
0724715垢版2018/02/10(土) 17:49:02.11ID:UAeKFFhu
>>723
返信ありがとうございます。

では具体的なデータの集まり全体の期待値の例として示していただくことは可能でしょうか?
0725715垢版2018/02/10(土) 17:57:50.93ID:UAeKFFhu
ああなんとなくわかってきました。
(c-E[X])^2+V[X]
の(c-E[X])^2は、cのE[X]に対する分散なので、
E[X-c]^2=(c-E[X])^2+V[X]
が成立するわけですかね?
例えばE[X]がサイコロの出る目の期待値で、cが実際に試行して出した目とした場合、
cの分散値は(c-E[X])^2という意味ですかね?
0727132人目の素数さん垢版2018/02/10(土) 18:06:58.67ID:dTOy0zth
>>724
サイコロでやりますね
C=1でやってみます
X={1,2,3,4,5,6}
Y=(X-C)^2={0,1,4,9,16,25}

E[X]=1×1/6+2×1/6+3×1/6+4×1/6+5×1/6+6×1/6=3.5
E[Y]=0×1/6+1×1/6+4×1/6+9×1/6+16×1/6+25×1/6=55/6

Var[X]=(1-3.5)^2×1/6+(2-3.5)^2×1/6+(3-3.5)^2×1/6+(4-3.5)^2×1/6+(5-3.5)^2×1/6+(6-3.5)^2×1/6=35/12

E[Y]=55/6=(C-E[X])^2+Var[X]=(1-3.5)^2+35/12
0728132人目の素数さん垢版2018/02/10(土) 18:07:52.18ID:dTOy0zth
>>725
Cは定数です
やはり、確率変数とは何なのかすらわかっていないようなので、一番基本的なことからやり直した方が良いかと思います
0729715垢版2018/02/10(土) 19:13:33.23ID:UAeKFFhu
cは定数じゃないです。。。
たぶんE[X-c]^2が何なのかみなさんご存じないんでしょう。。
ここで聞いたことが私の最大の誤りです。
0730都会にアクがれる田舎学生垢版2018/02/10(土) 19:20:06.18ID:Rgp9TgIs
帰郷した女子学生によりますと
平均風俗持続関数はExp[-t/tau]です。
料金は持続時間tでExp[-t/decay]サービス割引があります。

いつダウンするのがいいか
公式がありますか?
0733132人目の素数さん垢版2018/02/10(土) 19:37:18.42ID:dTOy0zth
>>729
あなたの最大の誤りは、基本をおろそかにしていることですね
それと人の話を聞かないで自分の思い込みだけで進めることです
0734132人目の素数さん垢版2018/02/10(土) 19:38:04.35ID:mOvYbXUO
素が出てて草
0735132人目の素数さん垢版2018/02/10(土) 19:59:29.78ID:MY7c6GmE
>>715
さいころの場合こうじゃないかな
E[X-c]^2=1/6((1-c)^2)+1/6((2-c)^2)+1/6((3-c)^2)+1/6((4-c)^2)+1/6((5-c)^2)+1/6((6-c)^2)
=91/6-7c+c^2
c=7/2の場合E[X-c]^2は最小値で35/12
これらの値はさいころの目の期待値・分散と一致する
0736132人目の素数さん垢版2018/02/10(土) 20:25:57.45ID:8eZMz4rW
問題じゃなくて申し訳ないんですけどバロンの公式ってなんですか?
0737132人目の素数さん垢版2018/02/10(土) 23:35:54.65ID:uhbQujra
実数0,aに対し、以下の手続きで実数r[k]を構成する。

(1)r[0]=r、ただしrは0<r<aの実数。
(2)k≥1に対して、
・a-r[k-1]≥r[k-1]ならばr[k]=(a-r[k-1])/2
・a-r[k-1]<r[k-1]ならばr[k]=(r[k-1])/2

このとき、lim[k→∞] r[k] = a/2 を示せ。
0738132人目の素数さん垢版2018/02/10(土) 23:43:04.72ID:7iMsj2rj
君の自作問?
つまんな
0739132人目の素数さん垢版2018/02/10(土) 23:43:13.72ID:uhbQujra
すみません、1つ設問を記載し忘れました

|r[5]-(a/2)|≤(1/16 )となる初期値rの範囲を求めよ。
0741132人目の素数さん垢版2018/02/11(日) 04:05:57.97ID:92QP0Rge
>>737
さらに設問を追加します。東大入試並の難問です。

0より大きくaより小さい相異なる実数s,tを初期値として構成される数列r[k]を、それぞれrs[k],rt[k]とおく。
どのようなi,jの組に対しても、
rs[i]≠rt[j]
となるためにs,tが満たすべき必要十分条件を求めよ。
0742132人目の素数さん垢版2018/02/11(日) 05:50:21.86ID:92QP0Rge
xyz空間に正四面体PABCがあり、△ABCはxy平面上にある。
PAをa:bに内分する点をQ、PBをc:dに内分する点をR、PCをe:fに内分する点をSとする。ただしa,b,c,d,e,fは正の実数である。
3つの直線QR、RS、SQはそれぞれxy平面と交点T、U、Vを持つとする。
これらの3点T、U、Vが一直線上にあるとき、a,b,c,d,e,fが満たすべき条件を求めよ。
0743132人目の素数さん垢版2018/02/11(日) 06:06:50.38ID:92QP0Rge
一辺の長さが1の正四面体PABCの底面である△ABCの外接円をKとする。
Kの劣弧BC(両端は含まない)上に点Dを∠BAD=θ(0<θ≤π/6)となるようにとり、またKの優弧BC(両端は含まない)上に点Tをとる。
2点D,Tを通り、平面ABCに垂直な平面をH、Hによるこの四面体の切断面の面積をS(θ,T)とおく。

(1)θを固定しTを動かすとき、S(θ,T)の最大値M(θ)を求めよ。
(2)(1)のM(θ)について、θを動かすときの最大値を求めよ。
0744132人目の素数さん垢版2018/02/11(日) 06:10:43.22ID:92QP0Rge
平面上にn個(n≥4)の点があり、そのうちどの4個を選んでも同一円周上にあるという。
このとき、これらn個の点は全て同一円周上にあることを示せ。
0745132人目の素数さん垢版2018/02/11(日) 06:17:50.01ID:92QP0Rge
点(1,1)を中心とする半径1の円をC、放物線y=ax^2をDとする。ただしa>0である。
CがDによって分けられる2つの部分の面積が等しくなるときのaの値を求めよ。
0746132人目の素数さん垢版2018/02/11(日) 06:26:58.62ID:92QP0Rge
半径1の円周上に相異なるn個(n≥4)の点がある。これらのうちから3点を選んで三角形Sを作り、またこれらのうちから3点を選んで三角形Tをつくる。ただしSとTは重ならないものとする。
SとTの重なる部分の面積Mについて、以下の問いに答えよ。

(1)点の選び方を変えることによりSとTを色々変えても、Mが常に同じ値になることはあるか。

(2)Mの最小値をmとする。n個の点の配置を色々と変えるとmも変化するが、どのようなmに対してもm≥pが成り立つようなpの最大値を求めよ。
0749132人目の素数さん垢版2018/02/11(日) 09:42:10.16ID:a7KOzsQQ
奇数の完全数は存在しないことの証明


奇素数をy、素数をp、pの指数をnとし、
素数p以外の積の組み合わせの合計をxとすると

p以外の素数をp1,p2,p3,…pnとし、pnの指数をqnとすると
指数の合計は
S=Σ[k=1,n]qk
であり、
素数の組み合わせの個数は、2^Sとなることから
その合計xは偶数となる

(1+p+p^2+…+p^n)x-y=y
(1+p+p^2+…+p^(n-1))x=y

yは奇数であり、xは偶数であるからこの式は成立しない
よって、奇数の完全数は存在しない
0753132人目の素数さん垢版2018/02/11(日) 13:09:43.06ID:a7KOzsQQ
奇数の完全数は存在しないことの証明


奇素数をy、その素因数のうち一つをp、pの指数をn、
p以外の素因数をp1,p2,p3,…pmとし、pkの指数をqk、
素数p以外の積の組み合わせの合計をxとすると
x=Σ[k=1,m]pk^qk

指数qmの値の合計は
S=Σ[k=1,m]qk
であり、
素数の組み合わせの個数は、2^Sとなることから

y=(1+p+p^2+…+p^n)x-yとなるから
(1+p+p^2+…+p^n)x=2y
(p^(n+1)-1)x/(2(p-1))=y
(p-1/p^n)x/(2(p-1))=y/p^n

y/p^n=Π[k=1,m]pk^qkで奇数だから
xは2p^nで割り切れなければならない

xは組み合わせの個数が2^Sであるから、S>1の場合には
4の倍数となるからxを2p^nで割ることはできない。

S=1の場合には
y=pp1であり、
1+p+p1=pp1が成り立つとすると

1+p+p1=(p+1)(p1+1)-pp1
から
pp1=(p+1)(p1+1)/2
となるが、左辺は奇数で、右辺は偶数となることから
この式は成立しない

よって、奇数の完全数は存在しない
0754132人目の素数さん垢版2018/02/11(日) 13:20:53.16ID:2UD4VuhN
>(p-1/p^n)x/(2(p-1))=y/p^n
>y/p^n=Π[k=1,m]pk^qkで奇数だから
>xは2p^nで割り切れなければならない

(p-1/p^n)が整数でないのだから、この推論は成り立たない
0755132人目の素数さん垢版2018/02/11(日) 13:41:41.70ID:a7KOzsQQ
>>754
訂正しました

奇素数をy、その素因数のうち一つをp、pの指数をn、
p以外の素因数をp1,p2,p3,…pmとし、pkの指数をqk、
素数p以外の積の組み合わせの合計をxとすると
x=Σ[k=1,m]pk^qk

指数qmの値の合計は
S=Σ[k=1,m]qk
であり、
素数の組み合わせの個数は、2^Sとなることから

y=(1+p+p^2+…+p^n)x-yとなるから
(1+p+p^2+…+p^n)x=2y
(p^(n+1)-1)x/(2(p-1))=y
(p-1/p^n)x/(2(p-1))=y/p^n

y/p^n=Π[k=1,m]pk^qkで奇数だから
左辺は分母が整数なので、分子も整数にならなければ
ならないので、xはp^nで割り切れなければならない

xは組み合わせの個数が2^Sであるから、S>0の場合には
偶数となるからxをp^nで割ることはできない。

よって、奇数の完全数は存在しない
0759132人目の素数さん垢版2018/02/11(日) 16:25:26.83ID:lE4la0BD
それ使いきってから次スレかなあ
0761132人目の素数さん垢版2018/02/11(日) 17:44:29.45ID:IufDiH0h
無限大の空間で、無限大の動く物体が、無限の速さで移動したり、無限の高さをジャンプしたりしたらどんな感じになるのでしょうか?
0762132人目の素数さん垢版2018/02/11(日) 17:52:52.67ID:zE6ZSh8U
久しぶりに質問があります。

曲面: x^2+y^2=(2-z)^2 z∈[0,1] 上をベクトル r=(x,y,z)
が動くとき

面積分:∫r・ndSを求めよという問題わかるひといますか?
nは曲面の単位法線ベクトルです。
0763132人目の素数さん垢版2018/02/11(日) 18:00:55.78ID:lE4la0BD
電磁気かな
0764132人目の素数さん垢版2018/02/11(日) 19:19:26.67ID:V+//u+oi
曲面上をベクトルが動くってどゆこと?
0765132人目の素数さん垢版2018/02/11(日) 20:01:40.26ID:M1qgJQRb
じゃあ動くじゃなくて
曲面上の値をとるときで
物理数学なので厳密じゃなくていいと思います。
計算方法をご教授していただきたいです
0769132人目の素数さん垢版2018/02/11(日) 20:29:49.71ID:HQlfx7dk
>>762
こういうのは、ガウスの定理を使って面積分を体積分に変換すると簡単になるんよ。
div r = 3 だから、頭のちょんぎれた三角錐の体積 (7/3)πにこれをかけて、 7π。
そこから上の面の面積分 πを減じて、7π-π = 6π。
0770132人目の素数さん垢版2018/02/11(日) 20:33:26.31ID:NpzD2qN4
ありがとうございます。
理解できました。
ガウスの定理がポイントなんですね。
0771132人目の素数さん垢版2018/02/11(日) 20:35:29.30ID:HQlfx7dk
まともに面積分しても、それほど大変じゃない。この面上で、r・n dS = √2 dS (定数)
になるから、側面の面積を√2倍すれば求める面積分。で、側面の面積は 3√2π。
0772132人目の素数さん垢版2018/02/11(日) 20:45:55.41ID:NpzD2qN4
皆さん賢いですね・・
いろいろありがとうございます参考にさせていただきます
0775132人目の素数さん垢版2018/02/11(日) 21:07:18.68ID:wgY+32c2
面積分が出てきたらとりあえず発散定理が思い浮かぶようになっても損はしない
物理系ならなおさら
0780132人目の素数さん垢版2018/02/12(月) 03:25:41.19ID:BjhhanPC
このスレの連中の数学力を試す
120分で解け

1.
定積分 ∫ [0→∞] (1+sin(x))^2/(1+x^2) dx を求めよ。

2.
n個の実数a1,...,anの積が素数となるとき、以下の命題の真偽を判定せよ。
(1)a1,...,anのうち整数であるものの個数の最大値は[(n/2)-1]である。ただし[x]でxを超えない最大の整数を表す。
(2)a1,...,anの中に無理数が含まれるならば、その個数は偶数である。
0781132人目の素数さん垢版2018/02/12(月) 04:01:57.45ID:sM4N6rqJ
赤玉と白玉があるそれぞれ異なる割合の割合で袋に入っている。
そして赤玉と白玉の中にはそれぞれ1割、2割の割合で小さな青玉が入っている。
ある人が小さな青玉を見せてきたとき、それが赤玉に入っていただろうと
判断し得る赤玉と白玉の割合に対して、どのくらい多く赤玉を入れるべきか?
0782132人目の素数さん垢版2018/02/12(月) 05:55:55.11ID:BX0xHGrQ
それ過去問
0783132人目の素数さん垢版2018/02/12(月) 08:10:36.80ID:BjhhanPC
かつての大学の後期の問題らしいです
難しくて手が出ません
各頂点の座標だけでも教えてください

xyz空間に一辺の長さが1である正二十面体Vが置かれており、頂点の1つは原点O(0,0,0)にある。
またもう一つの頂点はz軸上の正の部分の点A(0,0,a)にあり、Vはz軸に関して対称である。このとき以下の問いに答えよ。

(1)aを求めよ。

(2)Vの内部かつx^2+y^2≤1である部分の体積を求めよ。
0786132人目の素数さん垢版2018/02/12(月) 10:41:07.12ID:XPhbwfc3
>>778
それ過去問
0787132人目の素数さん垢版2018/02/12(月) 10:44:31.62ID:XPhbwfc3
>>780
2はバカ問
0788132人目の素数さん垢版2018/02/12(月) 10:46:34.36ID:XPhbwfc3
>>781
問題文書き直し
0789132人目の素数さん垢版2018/02/12(月) 10:52:23.43ID:IoO/5qAd
>>783
めんどくさ過ぎ
0792132人目の素数さん垢版2018/02/12(月) 13:36:11.51ID:dJ6x5l8V
12面体を求めるなら
(0,0,1),(sinθcos(2nπ/5),sinθsin(2nπ/5),cosθ) を考えて
(sinθcos(2nπ/5),sinθsin(2nπ/5),cosθ) 間の角がθになるようにすれば良いから
cosθ=(sinθ)^2 cos(2π/5)+(cosθ)^2 を解く
0793132人目の素数さん垢版2018/02/12(月) 15:01:18.07ID:JdQjoZ85
NGワードエラーとでてリンクが貼れないのですが、


32で割ってr余る整数p、32-r余る整数をqとすると
それぞれp=32k+r、q=32l-r(r,lは整数)
の形で表すことができる

でpはいいのですが、qがなぜ32-r余る式なのかわかりません。

また、
rと2016-rを32で割ったときの余りはそれぞれr、32-rなので、

というのもなぜ32-rになるのかわかりません。

高校の剰余の定理などでは理解ができないようなのでわからないのです。
初歩的なことなので呆れられるかもしれませんが、よろしくお願いします。
0794132人目の素数さん垢版2018/02/12(月) 15:03:26.01ID:JdQjoZ85
http://mathnegi.ブログ.エフシーツー.com/ブログ-entry-236.html

こちらの問題の解法1です。
エフシーツーをfc2に変えてリンク先に飛んでください。
0796793垢版2018/02/12(月) 15:50:30.27ID:JdQjoZ85
>>795
レスありがとうございます。助かりました。
0797132人目の素数さん垢版2018/02/12(月) 16:59:03.64ID:wDY0X4JU
数学の記述をするとき
ベクトルで、
「基点をOとし、位置ベクトルをOA(a),OB(b)とする」
↑文字の上には矢印があります

これって正しい記述ですか?
0801132人目の素数さん垢版2018/02/12(月) 17:24:10.31ID:mtigaFoC
昔はその位置ベクトルの書き方をしなかったような気がするけど
いつの間にかその書き方してる人が増えてんね
別にOA=aでいいと思うぞ
0802132人目の素数さん垢版2018/02/12(月) 17:39:15.47ID:v8LHrecW
OA(a)という記述
日常的な文章でそうするように括弧内で言い換えをしてるつもりなんだろうけど、
数学では数学記号の一部として括弧を多用するから、こういう紛らわしい記述はよくない
前者の意味で括弧を使うなら括弧内は記号ではなく日本語の文や単語であるべき
0804132人目の素数さん垢版2018/02/12(月) 18:23:31.39ID:KkKeT0mC
数学は定義を重視しているといいながら、実際は
かなりずぼらな表現で教科書とか書かれていたりする
難しいのではなく情報欠損ではそりゃ読めないだろうさ
教師の質が悪い
0809132人目の素数さん垢版2018/02/12(月) 20:57:45.49ID:kPPF2Oo+
5chは初めてか?力抜けよ
0810132人目の素数さん垢版2018/02/13(火) 00:45:48.74ID:u8y/1ecu
極上のサーロインステーキを食べたい。
0811132人目の素数さん垢版2018/02/13(火) 01:10:41.41ID:I9auIbGz
AB=3、BC=4、CA=5の直角三角形の周および内部の領域をDとする。
D内でPA・PB・PC=kとなる点Pを考える。kの取りうる値の範囲を求め、Pの軌跡を求めよ。
0812132人目の素数さん垢版2018/02/13(火) 01:50:17.06ID:ZOqrSE8B
>>780 >>787
1もバカ問?

∫[0,∞) 1/(1+xx) dx = [ arctan(x)] (x=0,∞)= π/2 = 1.570796326795…

∫[0,∞) 2sin(x)/(1+xx) dx = Ei(1)/e - Ei(-1)e = 1.29352224556…

∫[0,∞) sin(x)^2 /(1+xx) dx = (π/2e) sinh(1)= (π/4) {1 - e^(-2)} = 0.6791060805…

答え 3.54342465285…
0813132人目の素数さん垢版2018/02/13(火) 03:36:24.18ID:ZOqrSE8B
>>811
BAをx軸、BCをy軸とする。
A(3,0) B(0,0) C(0,4) P(x,y)

0 ≦ k ≦ 36/√5 = 16.0996894

k=0 のとき 3頂点 A,B,C のみ。
k=36/√5 のとき P(x,y)= (6/5,12/5) … 辺AC上の点
k≧36/√5 のとき Dを包含する。

Dの外部も含めて考えると…

0 < k < 9.3908 のとき A,B,C の3ループ
9.3908 < k < 14.20 のとき AB,C の2ループ
14.20 < k のとき ABC を包含する単一ループ

k = 9.3908 のとき P(x,y) = (1.61880,0.25598) で交差
k = 14.20 のとき P(x,y) = (0.40111,2.41893) で交差
0814132人目の素数さん垢版2018/02/13(火) 15:24:08.50ID:I9auIbGz
放物線y=ax^2+bx+cについて以下の問いに答えよ。ただしa>0かつb^2-4ac≥0とする。
p,qを有理数とする。xy平面上にln:x=p+nq(nは整数)の形で表される無数の直線が引かれている。
このとき、以下を示せ。
「p,qをどのような有理数にとっても、a,b,cを変化させれば、次の命題は成り立たない。『任意のlnと放物線とがy>0の領域で交点を持つならば、すべての実数xに対しy>0である。』」
0818132人目の素数さん垢版2018/02/13(火) 17:09:55.02ID:I9auIbGz
sinθ=(√3-1)/2となる角θについてご教授ください
sin45°・sinθ=sin15°ですが、このθは特殊角ですか?
0819132人目の素数さん垢版2018/02/13(火) 17:53:44.99ID:F/iVY6fq
電気回路とランダムウォークについての質問です。
平面の格子点全体のグラフZ^2において、コンダクタンス1とすれば再帰的になりますが、
適当なコンダクタンスC:E→(0,+∞)を入れて(Z^2,C)を非再帰的にするにはどうすればいいのでしょうか。
0824132人目の素数さん垢版2018/02/14(水) 03:36:27.60ID:/bHsoXtp
>>783

(1) a = 2cos(18゚)= 2sin(72゚)= √{(5+√5)/2}= 1.902113…

(2)Oから他の11頂点までの距離の2乗の和は 6aa である。
0827132人目の素数さん垢版2018/02/14(水) 16:17:28.44ID:7V6FZjpE
傑作問題

放物線C:y=ax^2+bx+cがある。ただしa>0とする。
p,qを有理数とする。xy平面上にln:x=p+nq(nは整数)の形で表される無数の直線が引かれている。
このとき、以下を示せ。
「すべてのlnはCとy>0の領域で交点を持つが、実数xに対しy<0となるようなp,qが存在する。」
0829132人目の素数さん垢版2018/02/14(水) 16:30:02.05ID:7V6FZjpE
>>828
傑作問題

放物線C:y=ax^2+bx+cがある。ただしa>0とする。
p,qを有理数とする。xy平面上にln:x=p+nq(nは整数)の形で表される無数の直線が引かれている。
このとき、以下を示せ。
「以下のようなp,qが存在する。『すべてのlnはCとy>0の領域で交点を持つが、Cにはy<0となる部分がある。』」
0831132人目の素数さん垢版2018/02/14(水) 16:42:28.57ID:7V6FZjpE
>>830
傑作問題


放物線C:y=ax^2+bx+cがある。ただしa>0とする。
p,qを有理数とする。xy平面上にln:x=p+nq(nは整数)の形で表される無数の直線が引かれている。
このとき、以下を示せ。
「すべてのlnはCとy>0の領域で交点を持つが、実数xに対しy<0となるようなp,qおよびCが存在する。」
0833132人目の素数さん垢版2018/02/14(水) 18:04:31.56ID:7V6FZjpE
>>830

傑作問題

放物線C:y=ax^2+bx+cがある。ただしa>0とする。
p,qを有理数とする。xy平面上にln:x=p+nq(nは整数)の形で表される無数の直線が引かれている。
このとき、以下を示せ。
「与えられたp,qに対しa,b,cをうまくとれば、Cはy<0の部分を持つが、すべてのlnとy>0の領域で交点を持つようにできる。」
0835132人目の素数さん垢版2018/02/14(水) 18:52:49.81ID:7V6FZjpE
傑作問題です、さらに磨き上げました

放物線C:y=ax^2+bx+cがある。ただしa>0とする。
p,qを有理数とする。xy平面上にln:x=p+nq(nは整数)の形で表される無数の直線が引かれている。
このとき、以下を示せ。
「与えられたp,qに対しa,b,cをうまくとれば、Cはy<0の部分を持ち、かつ、すべてのlnとy>0の領域で交点を持つようにできる。」
0837132人目の素数さん垢版2018/02/14(水) 19:03:48.27ID:+eR545sM
問題:
剣山の上に風船を載せると、風船全体は、剣山の針の上にあるか?
答え:
風船はいくつかの針に支えられているが、それらの針の間に位置する風船は、
風船の曲面が下に凸であるため、剣山の先端を下回る。終了
0840132人目の素数さん垢版2018/02/14(水) 22:25:58.53ID:JKDCbORz
あほにはごみを
0841132人目の素数さん垢版2018/02/15(木) 03:20:32.49ID:UgNb7y/z
初項および係数が実定数の二項間漸化式で定義される無限数列の集合をSとする。
Sに属する数列の増減はどのようであるか、分類せよ。
0842132人目の素数さん垢版2018/02/15(木) 03:31:29.01ID:tNBf7zgk
>>824

θ = 36゚ とおくと
0 = sin(3θ)- sin(2θ)
= 3sinθ -4(sinθ)^3 -2 sinθ cosθ
= -sinθ +4 sinθ (cosθ)^2 -2 sinθ cosθ
= sinθ {4(cosθ)^2 -2cosθ -1},
sinθ≠0 だから
4(cosθ)^2 -2cosθ -1 = 0,
cosθ = (1+√5)/4,
sinθ = √{(5-√5)/8},
a = 2sin(2θ) = √{(5+√5)/2},

あるいは
0 = cos(3θ)+ cos(2θ)
= 4(cosθ)^3 -3cosθ + 2(cosθ)^2 -1
=(cosθ+1){4(cosθ)^2 -2cosθ -1),
cosθ≠-1 だから
4(cosθ)^2 -2cosθ -1 = 0,
以下同様
0845132人目の素数さん垢版2018/02/15(木) 16:04:07.85ID:uoPhQNuf
記述のとき
導関数が0となるxを表記するときに
f'(x)=0 ⇔ x=2,3

みたいに、⇔の記号を書くのは間違ってないですか?
0847132人目の素数さん垢版2018/02/15(木) 22:28:28.11ID:UgNb7y/z
この問題が分かりません。球だと思うのですが、どう解いたら良いか分かりません

曲面上の任意の2点間の距離が2以下であるような閉曲面のうち、曲面積が最大であるものは何か。
0848132人目の素数さん垢版2018/02/15(木) 23:10:27.04ID:/4/K+H0+
>>847
距離というのが3次元ユークリッド距離のことなら、
球の表面をもっとぐねぐねさせれば表面積大きくならない?
絨毛 みたいに。
曲面上の測地線を考えてるなら話は別だね どっちだろう
0850132人目の素数さん垢版2018/02/16(金) 00:33:13.60ID:7BL9mIgD
>>2
x=-1で最小となりx=-3のときy=5、x=2のときy=15である2次関数の式を求めよ
教えて下さい
0852132人目の素数さん垢版2018/02/16(金) 00:51:03.02ID:xXIgzvk8
>>850
y=ax^2+bx+c
y'=2ax+b
y''=2a
0=-2a+b
a>0
5=9a-3b+c
15=4a+2b+c
(-2,1,0)(a).(0)
(9,-3,1)(b)=(5)
(4,2,1)(c) (15)
|-2,1,0|
|9,-3,1|=6+4+4-9=5
|4,2,1|
a=(15-5)/5=2>0
b=(-10+30)/5=4
c=(90+20+20-135)/5=-1
0853132人目の素数さん垢版2018/02/16(金) 00:54:14.20ID:KqaRe5q9
f=e^(-x)sin(1/x) x>0

において次の条件2つをみたす定数aを求めよ

1つめ 任意のxにたいしてa>=f
2つめ ある数列xnがあって
f(xn)がaに収束する
0854132人目の素数さん垢版2018/02/16(金) 00:54:41.50ID:MAbnZfAt
(二次関数が-1で最小) ⇔ y=a(x+1)^2+b ∧ a>0 (下に凸)

4a+b=5 ∧ 9a+b=15 ⇔ a= , b=
これはa>0を満たしている

a,bくらいは自分で
0856132人目の素数さん垢版2018/02/16(金) 13:28:36.65ID:HF3ijfpe
全ての2次関数fは以下の性質を満たすことを示せ。ただしf'はfの導関数である。
-∞<ff'<∞
0858132人目の素数さん垢版2018/02/16(金) 15:33:46.49ID:IxQut/js
m[0] , aj , bj は任意の自然数として

Σ{m[0] , j=0} (aj)*X^j = m[1]
Σ{m[1] , j=0}(bj)*X^j = m[2]
.
.
.
Σ{m[k] , j=0}(cj)*X^j = m[k+1]

としたとき、Xを超限順序数ωに変えたら不都合が生じますか?
0860132人目の素数さん垢版2018/02/16(金) 19:27:13.31ID:yGgbizft
パーw
0861DJ学術 垢版2018/02/16(金) 20:02:27.53ID:yN3n4O8g
クラメールとかコンドラチェフとかがマイ作。
書くと変なずれが出るから、数式専用版とかアートシャイエンス数学版とかに
将来アップします。
0862132人目の素数さん垢版2018/02/16(金) 23:45:52.58ID:9Ya0AtFs
集合{1/n ; nは自然数}は0を含みますか?
0863132人目の素数さん垢版2018/02/16(金) 23:47:47.77ID:9Ya0AtFs
>>862
0=1/nとなる自然数nは存在しないので0は含まれないと思っているのですが大丈夫でしょうか?
0865132人目の素数さん垢版2018/02/17(土) 00:44:38.38ID:jxTAJpLJ
先生から、実験して頂点Bが最も近くなるような領域を見つけてみなさいとヒントをもらいました。
その領域は立方体を合同な直方体に8分割したものの1つになるでしょうか?

(問題)
一辺の長さaの立方体Kの各面の重心を頂点とする正八面体Vがある。
Vの表面の点Pから最も遠いKの頂点をA、最も近いKの頂点をBと表す。このとき、積PA・PBの最大値とそれを与えるPの位置を求めよ。
ただしVの表面には、辺および頂点を含める。
0866132人目の素数さん垢版2018/02/17(土) 01:11:20.37ID:ZE5af5vu
>>865
適当にxyz空間に立方体を作ってみるといい
頂点を(±a/2,±a/2,±a/2)にとると、その「実験」は楽にできるかもしれない
0868132人目の素数さん垢版2018/02/17(土) 03:36:20.84ID:t4p1HKbt
>>865-866

PがVの1つの面、たとえば
x+y+z = a/2, x≧0,y≧0,z≧0
の上の点とすると、(*)より
A(-a/2,-a/2,-a/2)
B(a/2,a/2,a/2)

PA^2 = PC^2 + CA^2,
PB^2 = PC^2 + CB^2,
ここに、C(a/6,a/6,a/6)は正三角形の重心
PC^2 が最大 ⇔ Pが頂点にある
PA^2 ≦(0+a/2)^2 +(0+a/2)^2+(a/2+a/2)^2 = 3aa/2,
PB^2 ≦(0-a/2)^2 +(0-a/2)^2 +(a/2-a/2)^2 = aa/2,
PA・PB ≦(√3)aa/2,

(*)P(x,y,z)とすると、
x>0 ⇔ |x-|s||<|x+|s|| ⇔ P−(|s|,t,u)< P−(-|s|,t,u)
y>0 ⇔ |x-|t||<|y+|t|| ⇔ P−(s,|t|,u)< P−(s,-|t|,u)
z>0 ⇔ |x-|u||<|z+|u|| ⇔ P−(s,t,|u|)< P−(s,t,-|u|)
0869132人目の素数さん垢版2018/02/17(土) 04:08:10.54ID:GsLmqle/
Aの触点全体をclAが閉集合であることの証明を教えて下さい
0870132人目の素数さん垢版2018/02/17(土) 06:44:19.24ID:X8wlggSq
1時間半ほど考えたけど解らんので教えてください。


直径5の円のなかに, 10個の点をどのようにとっても, 必ず互いの距離が2より小さい2個の点があることを証明せよ.
0871132人目の素数さん垢版2018/02/17(土) 07:24:18.00ID:I/6qdj3n
>>870
鳩の巣原理を使うのでは?
0872132人目の素数さん垢版2018/02/17(土) 07:47:09.67ID:X8wlggSq
その使い方がわからない…
0873132人目の素数さん垢版2018/02/17(土) 07:55:33.84ID:lNZT8YJU
ラーメン食べたい。
0874132人目の素数さん垢版2018/02/17(土) 07:56:12.18ID:lNZT8YJU
ラーメン食べたい。
0875132人目の素数さん垢版2018/02/17(土) 07:56:30.01ID:lNZT8YJU
ラーメン食べたい。
0876132人目の素数さん垢版2018/02/17(土) 07:56:49.11ID:lNZT8YJU
ラーメン食べたい。
0877132人目の素数さん垢版2018/02/17(土) 07:57:12.54ID:lNZT8YJU
ラーメン食べたい。
0878132人目の素数さん垢版2018/02/17(土) 07:57:30.10ID:lNZT8YJU
ラーメン食べたい。
0879132人目の素数さん垢版2018/02/17(土) 08:00:01.70ID:lNZT8YJU
ラーメン食べたい。
0880132人目の素数さん垢版2018/02/17(土) 08:00:20.31ID:lNZT8YJU
ラーメン食べたい。
0881132人目の素数さん垢版2018/02/17(土) 08:02:21.56ID:lNZT8YJU
ラーメン食べたい。
0882132人目の素数さん垢版2018/02/17(土) 08:02:45.87ID:lNZT8YJU
ラーメン食べたい。
0883132人目の素数さん垢版2018/02/17(土) 08:03:02.77ID:lNZT8YJU
ラーメン食べたい。
0884132人目の素数さん垢版2018/02/17(土) 08:03:23.04ID:lNZT8YJU
ラーメン食べたい。
0885132人目の素数さん垢版2018/02/17(土) 08:03:31.10ID:X8wlggSq
半径1の円を真ん中に描いて、その外側を8等分して円を9個の領域に分けるので合ってる?
0886132人目の素数さん垢版2018/02/17(土) 09:46:03.03ID:16pjJLK7
八等分した1ピース内の最大距離が2未満なのは綺麗に示せるの?
0887132人目の素数さん垢版2018/02/17(土) 09:57:52.15ID:16pjJLK7
半径2.5の円は半径1の円9つ(中心1つと外側8つ)で覆えるんじゃない?
0888132人目の素数さん垢版2018/02/17(土) 10:18:12.99ID:ZE5af5vu
円周上に7つまで点を置ける
円周上に7つ置くとあと1つしか置けない
9つを置く方法があるかわからない
0889132人目の素数さん垢版2018/02/17(土) 11:44:12.43ID:bLoXKua7
単位円より少し小さい円10個で半径2.5の円は覆える(9個では足りない)
http://www2.stetson.edu/~efriedma/circovcir/
よって11個の点なら、ある2点間は2未満になる

10個の点なら実は配置できるんじゃないかなあ
0890132人目の素数さん垢版2018/02/17(土) 12:28:36.55ID:APQdN0L3
この解き方であっていますでしょうか?
(問題)
「A∪{B∩C}={A∪B}∩{A∪C}」を示せ

x∈A∪{B∩C}について
・x∈Aのときx∈A∪Bかつx∈A∪Cとなるのでx∈{A∪B}∩{A∪C}
・x∈{B∩C}のときも、x∈Bかつx∈Cからx∈A∪Bかつx∈A∪Cとなるのでx∈{A∪B}∩{A∪C}
よってA∪{B∩C}⊂{A∪B}∩{A∪C}
x∈{A∪B}∩{A∪C}についてx∈Aのときは自明なので、
・x∈Bのときにx∈{A∪B}∩{A∪C}となるにはx∈Cでなければならないのでx∈{B∩C}よりx∈A∪{B∩C}
よってA∪{B∩C}⊃{A∪B}∩{A∪C}
したがってA∪{B∩C}={A∪B}∩{A∪C}
0891132人目の素数さん垢版2018/02/17(土) 13:00:01.32ID:yPf15RxG
余弦定理使って少し考えれば直径が2以下であることはすぐ出る。
0894132人目の素数さん垢版2018/02/17(土) 13:32:06.98ID:f1AVQzS1
すいません。数学の初心者です。
4000回コイントスして、4000回中に15連続以上裏が続く確率の
解き方と答えを教えて下さい。
0896132人目の素数さん垢版2018/02/17(土) 13:33:15.08ID:dmApQDVU
>>890
だいたいは良いと思うけど
下段の⊃を示すときの場合分けはxがAの元である場合とxがAの元でない場合にした方が良いと思う
x∈Bであればx∈Cでなければならないってことはないからね(x∈Bかつx∈Aのとき)
0897132人目の素数さん垢版2018/02/17(土) 13:35:00.64ID:yPf15RxG
八等分した一つに二点A,Bがあり円の中心をCとしたら
AC,BCを固定したときABが最大になるのはCが最大のとき。
あとは「定点と線分上の点の距離が最大になるのは点が線分の端にあるとき」を使えば
ABが最大になるのはA,Bが端にあるとき。
端と端の距離は
1.91341716182544885864229992015199と
1.92729501998714830332400615640264なので
直径は2未満。
0898132人目の素数さん垢版2018/02/17(土) 13:41:43.22ID:f1AVQzS1
>>894です。
ExcelやPythonなどを使っても構いません。
どなたか解き方を教えて下さいませ。m(_ _)m
0899132人目の素数さん垢版2018/02/17(土) 13:50:28.83ID:IFXeHL1T
多面体を使った球体の再現について質問があります
以下の条件で球体を作ろうと考えています
 ・出来るだけ一様な面を持つ
 ・出来るだけ多くの面を持つ
 ・出来るだけ多くの変を持つ面を使う
 ・最悪、極の部分は再現できなくても良い
必然フラーレンやサッカーボールのような形になりますが

 ・五角形と六角形を使った場合20面体以上は何面が作れるのでしょうか?
  (何種類あるかとか理論上の最大、計算方法とか知りたいです)
 ・一様な面を使って球状の正多面体(概ね100面以上 どんなに大きくても良い)を作る事は出来ないでしょうか?
  上述の通り極の部分は欠損しててもアリです
0900132人目の素数さん垢版2018/02/17(土) 14:03:03.88ID:APQdN0L3
>>895
レスありがとうございます。
正確に、とは自明とか書いちゃったところでしょうか?
もしお手数でなければどこが正確でないかご指南いただけると幸いです。
なにぶん独学でやっていまして雰囲気で解いてしまっていると思うので…

>>896
なるほど!確かに「x∈Aでない」がなければx∈Cである必要は無いですね。
つまりx∈{A∪B}∩{A∪C}でx∈Aでないときにx∈Bかつx∈Cとなる、ということであっていますでしょうか?

それからもうひとつ窺いたいのですが集合族について例えばG={{0, 1, 2}, {3, 4, 5}, {6, 7}}みたいな時にG⊃{0, 1}、G⊃0などは成り立つのでしょうか?
0902132人目の素数さん垢版2018/02/17(土) 14:10:00.89ID:yPf15RxG
n回トスして15回連続裏が出ないで最後連続裏が出ている回数がm回である確率を漸化式を使って求める。
0903132人目の素数さん垢版2018/02/17(土) 14:48:19.54ID:ZepfIsNx
>>894
P[k]を(k+1)回目から(k+15)回目に最初の15回連続裏が現れる確率とする
全体の確率はP=Σ[j=0〜3985]P[j]
1)P[0]は1回目から15回目までが表となる確率なのでP[0]=2^-15
2)1≦k≦15について、1回目から(k-1)回目には15回連続の裏は含まれることはない。
P[k]はk回目が表で、かつ(k+1)回目から(k+15)回目が裏となる確率なのでP[k]=2^-16
3)16≦k≦3985について P[k]は、1回目から(k-1)回目に15回連続の裏が含まれず、
k回目が表で、かつ(k+1)回目から(k+15)回目が裏となる確率なので、
P[k]=(1-Σ[j=0〜k-16]P[j])(2^-16)
これらを元に計算するとP=0.0590367くらい
0905132人目の素数さん垢版2018/02/17(土) 20:05:54.15ID:kZHesKGB
ゲーム作ってるんだけどキャラの縦と横の移動速度が同じ場合(それぞれ3とする)
3÷√2で斜め移動中の縦と横の速度を求められるのは知ってるけど(約2.121)
これが移動速度縦3横2での斜め移動といった場合にはどうすれば縦横それぞれの速度を求められますかね
0906132人目の素数さん垢版2018/02/17(土) 20:41:40.99ID:kZHesKGB
補足というか書き忘れだけど上のは斜め移動時に加速してしまうのを補正する計算です
0907132人目の素数さん垢版2018/02/17(土) 20:54:27.12ID:APQdN0L3
>>905
三平方の定理を用いてあげればいいと思います。
この場合であれば斜辺の長さ(速さ)が3になってかつ縦横の速さの比が3:2になればいいので
縦:3*3/sqrt(3^2+2^2)
横:3*2/sqrt(3^2+2^2)
になるかなと思います。sqrt(x)は平方根です。
ただ僭越ながら想像するにジョイパッドかなにかで全方向に自由に動くゲームを作られるのではないですか?
もしそうであればジョイパッドの傾いた方向の角度を用いて三角関数で速度を計算するのが良いのではないかと考えます。
0908132人目の素数さん垢版2018/02/17(土) 21:33:16.05ID:Ks/+Q+uY
>>893
>試験中に知恵袋に書き込めるガバガバの京大入試
なんじゃそれw
0909132人目の素数さん垢版2018/02/17(土) 21:44:45.74ID:Ks/+Q+uY
>>899
>・五角形と六角形を使った場合20面体以上は何面が作れるのでしょうか?
20面体
0910132人目の素数さん垢版2018/02/17(土) 21:46:35.06ID:kZHesKGB
>>907
縦:3*3/sqrt(3^2+2^2) → 9/sqrt(13) = 2.496〜
横:3*2/sqrt(3^2+2^2) → 6/sqrt(13) = 1.664〜
で合ってるでしょうか

斜辺の長さ3というのがどういう事なのかよく分からないのですが

ちなみに作っているのはアナログスティックで360度に動くようなのではなく
いわゆる十字キーでの古めかしい8方向移動タイプです
0911132人目の素数さん垢版2018/02/17(土) 22:06:28.45ID:APQdN0L3
>>910
すみません、不明確な表現でした。
いま、縦横3:2の速さで進む場合を考えていますね?
そうするとあるタイミングでは縦に3マス、横に2マス移動するわけです。
すると始めに居た位置からは斜めに移動しています。
今の縦横に移動した経路と実際に移動した斜め線を図に描きますと、直角三角形になります。
この斜辺の長さはある時間当たりに移動した距離ですので速度にあたるわけです。
いま問題にしているのはこの斜辺=速度を3に固定したい、ということでしたので上記の計算を行います。

アナログスティックですか!
そうしますとプレイヤーの直接入力で移動することを意図してはいないのですね。
RPG的なものを想像していたもので。
0912132人目の素数さん垢版2018/02/17(土) 22:25:11.65ID:kZHesKGB
>>911
速度を3に固定したいというのは縦横共に速度が3の時ですね
縦3横2の場合は理想の速度がいくつになるのか計算できていません
ただそのまま縦に3マス、横に2マス分動いたのでは「斜め移動の加速」が起きてしまいます
縦横同じ速度なら検索すれば例が見つかるのですが
0913132人目の素数さん垢版2018/02/17(土) 22:38:15.51ID:APQdN0L3
>>912
ごめんなさい、少しわからなくなりました…
いま考えられているのはどの方向にも同じ速さで進むための縦横の速度の計算、であっていますか?
0914132人目の素数さん垢版2018/02/17(土) 22:48:51.86ID:kZHesKGB
>>913
どの方向にも同じではないですね
縦が少し速くなると思います
知恵が無いなりに自分でも考えてみましたが>>910は合ってるような気がしてます
0916132人目の素数さん垢版2018/02/17(土) 23:13:11.62ID:ZE5af5vu
>>905
数字でやると理解が遅くなりますので変数を使います
どのみちコーディングは変数をつかうのでしょうし
横X縦Yの比率で、速度Vでものを動かそうとする場合の、横方向の速度Vxと縦方向の速度をVyとすると、以下の式になります
Vx=V*X/√(X*X+Y*Y)
Vy=V*Y/√(X*X+Y*Y)

この√(X*X+Y*Y)は座標(0,0)から座標(X,Y)までの距離を表す式となります。暗記しておいても損はありません
0917132人目の素数さん垢版2018/02/17(土) 23:16:54.12ID:kZHesKGB
>>915
三平方の定理はC^2 = A^2 + B^2というのですよね
ベクトルは関数で出してるのでよくわからないと思います

>>916
Vというのは移動量の多い方の値で良いのでしょうか
0918132人目の素数さん垢版2018/02/17(土) 23:25:42.99ID:APQdN0L3
>>917
三平方の定理はそれのことですが、速度の分解に用いることはご存知でしょうか?
もしその辺りが微妙であれば一度速度とベクトルについて勉強されることをおすすめします。
以下のページやそのもとのページなどは参考になるのではないかと思います。
物理のかぎしっぽ
ttp://hooktail.sub.jp/vectoranalysis/restudyVector1/

ところで関数で出すとのことですが具体的にどんな計算をする関数なのでしょうか?
こちらは個人的な興味の質問です。
0919132人目の素数さん垢版2018/02/17(土) 23:39:47.54ID:kZHesKGB
>>918
リンク先を見て勉強しておきます
関数は座標x1,y1からx2,y2までの距離、角度を求めるものや
距離と角度からx成分y成分を取り出すもの等を使っています

移動量の多い方をAとして
A / (A^2 + B^2) = Q
Q * A = 速度@
Q * B = 速度A
これで求まりますでしょうか
0920132人目の素数さん垢版2018/02/17(土) 23:59:48.37ID:APQdN0L3
>>919
ちょっと惜しいです。
Q=V/sqrt(A^2+B^2)
速度@:Q*A
速度A:Q*B
このVは>>916さんのVと同じもので、移動速度の大きさを示すものです。
0921132人目の素数さん垢版2018/02/18(日) 00:07:45.35ID:ZjrR49kp
もし余裕があれば三角比、三角関数についても学びなおしてみると理解が深まるかもしれないです。
すみませんが、これで落ちさせてもらいます。
ゲーム製作、陰ながら応援しています。
頑張ってくださいね。
0922132人目の素数さん垢版2018/02/18(日) 00:12:27.87ID:nV09OLti
>>920
sqrtは書き忘れでした
縦横が異なる値だとVがいくつになるかわからないので
値の大きい方で割ってから掛ければA対Bの割合になると考えたのですがこれはやはり間違っているのでしょうか
縦横が同じ値ならVもその値にすればいいのですが
0923905垢版2018/02/18(日) 00:14:42.25ID:nV09OLti
要領を得ない質問にお付き合いいただき有難うございました
頂いたレスで掴めたものはあるのでもう少し頑張ってみます
0924924垢版2018/02/18(日) 01:45:07.98ID:R/sb9HRI
X=1/√3+√2の時X^4+1/X^4の値の求め方ってわかりますか?
0925132人目の素数さん垢版2018/02/18(日) 01:48:56.26ID:e4NqLH6n
>>870

 >>897>>886>>885>>871 で解決

なお、>>897
上は A,B共に半径R=2.5の円周上の端点の場合で
 2 R sin(π/8)= R √(2-√2)= 0.76536686 R = 1.91341716

下は 一方が半径R=2.5の円周上の端点、他方が単位円周上の端点の場合で
 √(1+RR-R√2)= 1.92729502

ですね。

>>887-889
 9つの単位円で覆うのは、チョト無理
0926132人目の素数さん垢版2018/02/18(日) 03:37:52.49ID:gU6NQ80Q
大学一年なんですが、工学部ではペアノの公理を学ばないですか?東大京大なら学びますか
数学科では学ぶと思いますが僕の行ってる大学では簡単な微積分と線形代数しか学びません
あとこれができたら線形代数だいたいマスターという問題ありますか?
0927132人目の素数さん垢版2018/02/18(日) 03:41:57.08ID:e4NqLH6n
>>924

1/X = √3 + √2,
X = √3 - √2,
より
1/X + X = 2√3,
1/X - X = 2√2,
辺々掛けて
1/XX - XX = 4√6,

1/X^4 + X^4 =(1/XX - XX)^2 + 2
=(4√6)^2 + 2
= 98,
0928132人目の素数さん垢版2018/02/18(日) 03:45:38.30ID:R/sb9HRI
>>927

わかりました。
ありがとうございました!
0929132人目の素数さん垢版2018/02/18(日) 04:03:08.15ID:1f3/Fjuc
アラン・コンヌさんは天才の中の天才ですか?
0930132人目の素数さん垢版2018/02/18(日) 04:47:08.04ID:QJPmXO6o
>>926
適当な3次正方行列のn乗を求める
0931132人目の素数さん垢版2018/02/18(日) 05:07:28.55ID:Oc6UNOb6
>>926
学ぶのは勝手にやることなので「そんなこと教えてもらえなかった!」というのはない考え方
0934132人目の素数さん垢版2018/02/18(日) 09:40:44.21ID:pl5yPBEk
>>933
y=logxの凸性に言及してなくない?
凸性に言及すればその不等式を示すことが可能
具体的にはf''を求めて凸性について述べ、そこから平均変化率がどんどん小さくなる、と記述すればいい
高校数学だしこの程度の記述で許されると思う
0935132人目の素数さん垢版2018/02/18(日) 10:24:14.86ID:qShdtbzi
>>934
やっぱり不可能ですよね...。
わざわざ中間値使わなくても両辺ともlogx上の2点の傾きを表してるのでそこに触れて証明した方が良いのですかね?
0936132人目の素数さん垢版2018/02/18(日) 13:44:36.72ID:qShdtbzi
連続ですいません

何からしていいかよく分からなくて詰まっています。教えて頂けるとありがたいです。

原点中心に回転していくから
戦略としては極座標ですかね?

dx/dtとdy/dtから増減表かいて概形もとめるグラフにか困れた扇形みたいな図形と三角形にわかれる。なので場合わけ
扇形みたいな図形は
∫ydxでこのあとtで微分するからd{∫[0→x]f(t)dt}/dx=f(x)使う。
U(n)は導関数を積分して求めて、あとは極限ですか?

https://i.imgur.com/IUGms0s.jpg
0938132人目の素数さん垢版2018/02/18(日) 20:12:19.60ID:s7QIR4Au
2次の多項式f(x)が
 任意の自然数nに対してf(n)がn(n+1)で割り切れる
を満たすとき、f(x)は多項式としてx(x+1)で割り切れるといえますか
0941132人目の素数さん垢版2018/02/18(日) 21:52:05.89ID:r36RT/qX
といて欲しい証明の問題があります
「集合AをA={K(n)|K(n)は1を除く奇数である}と定める。
また、集合Aの要素は全て異なるものとする。
この時
(3+1/K(n))を任意の回数かけた値、
すなわちその値をXの式で
X=(3+1/K(1))*(3+1/K(2))*........と表す時
Xが、かけた回数やK(n)の値が任意かつ有限なもので自然数になることはない」
というものです
よろしくお願いします
0942132人目の素数さん垢版2018/02/18(日) 21:53:30.73ID:hJBbLdKR
f(x)=ax(x+1)+bx+cとする。
f(n)/n(n+1)=a+(bn+c)/n(n+1)で
lim(bn+c)/n(n+1)→0(n→∞)より
b,cが0でないならば
十分大きい整数nで条件を満たさない
よってb=c=0より示せた

どない?
0943132人目の素数さん垢版2018/02/18(日) 23:02:05.27ID:gINNEtP1
>>941
文章書き直し
0944132人目の素数さん垢版2018/02/18(日) 23:03:53.94ID:gINNEtP1
このスレで解かれずに残ってる問題一覧:
0945132人目の素数さん垢版2018/02/18(日) 23:13:28.36ID:5M24+335
どれもこれも全然ダメじゃん。
0946132人目の素数さん垢版2018/02/18(日) 23:15:29.83ID:F58eyW5n
感情の原因はそれを感じる者自身の固定観念・価値観・自己ルール
「言葉 風紀 世相の乱れ」はそう感じる人の心の乱れの自己投影
解釈(含む誤解)の原因は情報発信者ではなく受信者
問題解決力の低い者ほど自己防衛の為に礼儀作法やマナーを要求
憤怒は狂気、無知無能の自己証明。中途半端な知識主ほど辛辣に批判
「真実は一つ」は錯誤。執着する者ほど矛盾を体験(争い煩悩)
無自覚な傲慢者に多い「己の知見こそ全で真」も錯誤。独善の典型
論理的思考力の低い者ほどデマ宗教フェイク迷信に感化傾倒陶酔洗脳
史上最も売れているトンデモ本は聖書。神は人間の創造物
全ては必然。偶然 奇跡 理不尽 不条理は思考停止 視野狭窄の産物
人生 存在に元々 意味 価値 理由 目的 義務 使命はない
宗教民族領土貧困は争いの原因ではなく「理由口実動機言訳切欠」
社会問題の根本原因は低水準教育。必要なのは適切十分な高度教育
体罰は指導力・問題解決力の乏しい教育素人の独善甘え怠慢責任転嫁
死刑は民度の低い排他的集団リンチ殺人。「死ねば償える」は偽善
核武装論は人間不信と劣等感に苛まれた臆病な外交素人の精神安定剤
投票率低下は社会成熟の徴候。奇人変人の当選は議員数過多の証左

感情自己責任論 〜学校では教えない合理主義哲学〜 m9`・ω・)
0947132人目の素数さん垢版2018/02/18(日) 23:25:31.11ID:hJBbLdKR
>>943
どこがだめなん?なんかルールあったっけ?
0948132人目の素数さん垢版2018/02/18(日) 23:30:01.48ID:dSxwtxgZ
集合じゃないものを集合というな

(3+1/83)(3+1/55)(3+1/29)(3+1/25)(3+1/19)=256.
0949132人目の素数さん垢版2018/02/18(日) 23:55:42.70ID:e4NqLH6n
>>887 >>889

n枚の単位円板で覆うことのできる円の半径 R_n は↓らしい。

n, R_n, 単位円板(中心)の配置
-----------------------------------------
1, 1,
2, 1,
3, 2/√3 = 1.15470 , 正3角形(辺長 √3)
4, √2 = 1.41421 , 正方形(辺長 √2)
5, 1.641004464 , 5角形 Kroly Bezdek (1983)
6, 1.798869 , 6角形 Karoly Bezdek (1979)
7〜10, 1+2cos(2π/(n-1)), 原点O と 正(n-1)角形(辺長 2sin(2π/n))
7, 1+2cos(π/3)= 2 ,
8, 1+2cos(2π/7)= 2.24698 , , Gabor Fejes Toth (1996)
9, 1+2cos(π/4)= 1+√2 = 2.41421, , Gabor Fejes Toth (1996)
10, 1+2cos(2π/9)= 2.53209 , , D. Nagy (1974)
11, 2.63100 , (内) 2つ と(外)9角形, Hars Melissen (1997)
12, 2.76900 , (内) 正3角形 と(外)9角形, Hans Melissen (1997)

http://ja.wikipedia.org/wiki/円板被覆問題
http://mathworld.wolfram.com/DiskCoveringProblem.html
http://www2.stetson.edu/~efriedma/circovcir/
0950132人目の素数さん垢版2018/02/18(日) 23:57:37.00ID:hJBbLdKR
>>948
どーやって見つけたん?
0952132人目の素数さん垢版2018/02/19(月) 00:08:53.60ID:VF4EpRLf
プログラムぶん回したか
0953132人目の素数さん垢版2018/02/19(月) 00:21:09.88ID:KkZd2DJX
>>942
aが整数であることを補足すればOK
0954132人目の素数さん垢版2018/02/19(月) 00:23:51.70ID:ar7lPSMz
>>953
こんなかんじの問題京大の問題にあったよな
0955132人目の素数さん垢版2018/02/19(月) 00:25:07.21ID:kgsz5ltG
>>948ありがとうございます!
0956132人目の素数さん垢版2018/02/19(月) 00:26:58.88ID:ar7lPSMz
あれID変わってるゾ
0957132人目の素数さん垢版2018/02/19(月) 00:27:27.07ID:/8jC6j7+
>>948 >>950

コラッツ(角谷)の問題(予想)ですね。
55→83→125
25→19→29→11
より
125|55(3+1/55)(3+1/83)
11|25(3+1/25)(3+1/19)(3+1/29)
辺々掛ける。
0958132人目の素数さん垢版2018/02/19(月) 00:27:27.20ID:ar7lPSMz
日替わりなんかな
0959132人目の素数さん垢版2018/02/19(月) 00:30:54.38ID:LXKSoPV8
老オカマの「阪京」(ハゲキモ)とかいうヤツは
まだ生き恥をさらしているのか?
実際に妻子あるいは親戚がいるのか?
もしもいるとしたら、阪京が家族に隠れて他人の男性器を
しゃぶったり、ケツボボを掘って貰いたがったりしている事を
どう思ってるんだろう?

それともナタが怖くて近寄らないのか?
0960132人目の素数さん垢版2018/02/19(月) 00:35:48.19ID:ar7lPSMz
うぴー
0961132人目の素数さん垢版2018/02/19(月) 00:36:18.12ID:ar7lPSMz
うぴぴー
0962132人目の素数さん垢版2018/02/19(月) 11:12:57.47ID:OuIuY8Mw
質問があります。
当選確率40%のくじを7回引いて全部外れる確率は何%ですか?
計算式も教えて下さい。
よろしくお願いします。
0964132人目の素数さん垢版2018/02/19(月) 11:37:28.25ID:OuIuY8Mw
963さんへ
どうもありがとうございます
0965132人目の素数さん垢版2018/02/19(月) 14:20:00.30ID:FejsIOhD
>>926
講義で扱ったとしても、こんなのがあると紹介する程度でしょうね。
工学部の数学で基礎論に深入りすることは、ないだろうと思います。
自分で図書館の本を読めば済む話ではないですか?
将来仕事に使うことはまずない事項なので、
教わらないことを不安がる必死はないでしょう。
0966132人目の素数さん垢版2018/02/19(月) 15:10:39.00ID:ZUVW5CIQ
おらの高校の算数の最初の授業が、ペアノの公理だったな。それから、関東の集合論で対角線論法。センセの趣味を押し付けられた感じだけど、きっちり理解できたぞ
0968132人目の素数さん垢版2018/02/19(月) 15:36:32.24ID:84K17tgR
集合族について質問です。
C={{1, 2, 3, 4, 5}, {3, 4, 5, 6}, {3}, {2, 9}}みたいなときに{1, 2, 3}∈Cや3∈C、9∈Cなどは成り立つのでしょうか?
0970132人目の素数さん垢版2018/02/19(月) 15:54:48.28ID:84K17tgR
>>969
ありがとうございます。
とするとX={1, 2, 3}としたときにG={φ, {1}, {2}, {3}, {1, 2}, {2, 3}, {1, 3}, {1, 2, 3}}は加法族になって
G'={φ,{1, 2, 3}}では加法族にならないと思うのですが、この認識は正しいでしょうか?
0972132人目の素数さん垢版2018/02/19(月) 16:09:03.42ID:84K17tgR
>>971
立て続けに質問してすみませんが、G'では加法族の条件
あるA⊂XでA⊂G'となるときX-A⊂G'が成り立たないような気がするのですが…
すみませんがご教示願えないでしょうか?
0974132人目の素数さん垢版2018/02/19(月) 16:14:01.36ID:84K17tgR
すみません、冷静に眺めていたら普通に成り立っていますね。
φ⊂Xでφ⊂G'に対しては{1, 2, 3}⊂G'ですし
{1, 2, 3}⊂Xで{1, 2, 3}⊂G'に対してはφ⊂G'になりますね。
集合族中の要素の部分集合も集合族の要素となりえると勘違いしていたのがまだ残っていたようです…
お騒がせしました。
しつこいようですがつまりはG''={φ, {1, 2, 3}, {1, 2}}みたいな場合では加法族にならないのですね?
0976132人目の素数さん垢版2018/02/19(月) 16:17:55.57ID:84K17tgR
>>975
ありがとうございました。
ここ最近のもやもやが晴れました。
あと記号の間違いについてもご指摘ありがとうございます。
0977132人目の素数さん垢版2018/02/19(月) 17:22:58.78ID:d9u9RHnn
このようにドーナツ型の図があり、外側の円(以下、外円)と内側の円(以下、内円)は対応して動くとして
2つの円周には一回しか塗れないインクがついてるとします

2つの円を右に回転させたとき外円の接地面にはインクが全てつくと思いますが、
内円の接地面にはどのようにインクがつきますか?

中学生の頃からの疑問なのですがどうしてもわからないので、教えてください


https://i.imgur.com/2nJdEr9.jpg
0978132人目の素数さん垢版2018/02/19(月) 18:01:05.96ID:v14nFkWY
風と熱の違いってなんですか?
大学の説明会で教授に聞いたのですがテンソルが関係あると言っていました
よく分からなかったのでここで聞いてみたいと思います
0979132人目の素数さん垢版2018/02/19(月) 18:04:06.53ID:CMze8r9t
風って高気圧から低気圧に空気が流れるってことだろ?

詳しいことは物理か気象板で聞いたら?
0981132人目の素数さん垢版2018/02/19(月) 19:23:01.34ID:nk+ysyni
風邪は熱です
0982132人目の素数さん垢版2018/02/19(月) 21:53:59.41ID:lqyOSah5
一理ある
0983132人目の素数さん垢版2018/02/19(月) 21:58:20.15ID:lqyOSah5
>>977
内側の円が描く線も、外側と同じ長さになる
円周の長さは関係ない
0985132人目の素数さん垢版2018/02/19(月) 23:54:48.58ID:VF4EpRLf
ペアノ曲線の方が面白くない?
0987132人目の素数さん垢版2018/02/20(火) 14:45:13.72ID:9vWhVGGm
正多面体の図形的性質がよく載っている本を教えていただけませんか
具体的には、体積や表面積の求め方、色々な切断面の形、いくつかの頂点を結んでできる立体の種類、を知りたいです
0989132人目の素数さん垢版2018/02/20(火) 15:22:27.92ID:Y1aF8iKv
ペアノ曲線か、懐かしいのう。おらが中学の時、ペアノ曲線をモチーフにデザイン画書いたが、美術のセンセには不評だった。ぐすん
0993132人目の素数さん垢版2018/02/21(水) 03:04:10.53ID:xFlMdI8t
>>989

大和絵・錦絵の「すやり霞」なんかに応用できそうですが…

書道の先生なら、わかってくれるでしょう。
0996132人目の素数さん垢版2018/02/21(水) 15:18:57.83ID:Kk32RnsU
この問題で、四面体が動く領域をxy平面に平行な面で切って捉え、積分して体積を求めようとしているのですが、切断面の概形がつかめずうまくいきません。ご教授ください。

一辺の長さが1の正四面体OABCがある。ただしOは座標空間の原点とする。
底面ABCの重心をG(p,q,r)とし、p≥q≥r≥0を満たす範囲でGを動かすとき、この正四面体(内部を含む)が通過してできる領域の体積を求めよ。
0998132人目の素数さん垢版2018/02/21(水) 17:35:25.08ID:0jBhw946
全ての向きをとりうるんでしょ
1000132人目の素数さん垢版2018/02/21(水) 18:40:46.94ID:JFIkQrIb
>>996
(1/2)πsrの球面三角形と、それに隣接するはみ出た部分の面積を、辺と角の2種類に分けて評価するといいんじゃないかな
10011001垢版Over 1000Thread
このスレッドは1000を超えました。
新しいスレッドを立ててください。
life time: 32日 5時間 3分 41秒
10021002垢版Over 1000Thread
5ちゃんねるの運営はプレミアム会員の皆さまに支えられています。
運営にご協力お願いいたします。


───────────────────
《プレミアム会員の主な特典》
★ 5ちゃんねる専用ブラウザからの広告除去
★ 5ちゃんねるの過去ログを取得
★ 書き込み規制の緩和
───────────────────

会員登録には個人情報は一切必要ありません。
月300円から匿名でご購入いただけます。

▼ プレミアム会員登録はこちら ▼
https://premium.5ch.net/

▼ 浪人ログインはこちら ▼
https://login.5ch.net/login.php
レス数が1000を超えています。これ以上書き込みはできません。

ニューススポーツなんでも実況